Person-Centered Care and the Nursing Process UNIT III

Lakukan tugas rumah & ujian kamu dengan baik sekarang menggunakan Quizwiz!

Examples of Technology Evaluation Steps

1️⃣ ❑STEPS IN INFORMATICS EVALUATION Determine what will be evaluated ❑DESCRIPTION Determine what to evaluate. Examples: • Adoption and use of the patient portal • Safety of using copy/paste function in the EHR • Effectiveness of modified early warning system on reducing codes called outside of the ICU • Effectiveness of allergy alerting 2️⃣ ❑STEPS IN INFORMATICS EVALUATION Determine the question ❑DESCRIPTION A clear, focused question helps the team determine what data will need to be collected and how the data ultimately should be reported. Examples: • Has the implementation of the nurse-driven urinary catheter protocol reduced the number of catheter days for patients? • Have the new admission assessment screens resulted in timesaving and improved support of workflow for nurses? • What is the estimated amount of financial savings if automated report printouts were reduced by 50%? 3️⃣ ❑STEPS IN INFORMATICS EVALUATION Conduct a literature search ❑DESCRIPTION Searches should be conducted in peer-reviewed journals by searching available databases such as Cumulative Index to Nursing and Allied Health Literature (CINAHL), PubMed, and Cochrane Reviews. • Has this topic been studied before? • What data was collected? • What were the findings? • Can we replicate the methods of the study? 4️⃣ ❑STEPS IN INFORMATICS EVALUATION Determine the needed data ❑DESCRIPTION In this step, the specific data elements to be collected need to be determined. • What data is needed to answer the question? • Is it available? Can the data be easily queried and pulled into a report or spreadsheet for data manipulation? • Who will collect it? • What data collection tool will be used? 5️⃣ ❑STEPS IN INFORMATICS EVALUATION Determine the study type ❑DESCRIPTION When determining your study design, first take into account the question you are asking, and the data you are collecting to answer the question. • Pre- or postevaluation of an intervention • Retrospective study • Observational study • Time/motion study • Case study • Randomized control trial • Descriptive evaluation of current state 6️⃣ ❑STEPS IN INFORMATICS EVALUATION Determine the data collection method and sample size ❑DESCRIPTION Clarify exactly how the data will be collected. • Is the data already available in the EHR database (or other database), and you will need the assistance of someone skilled in running a query or developing a report? • Will you need to conduct a manual chart review? • Will you need to observe end users as they interact with the system? • Will you need to conduct a survey? • Will you need a focus group to gather data? • Check sample size and date ranges in previous studies on the same topic • Secure the help of a statistician. 7️⃣ ❑STEPS IN INFORMATICS EVALUATION Collect, analyze, and display data ❑DESCRIPTION Whether using a table, bar chart, pie chart, or other method, ensure data displays include: • Title with date range and sample size as appropriate • Legends that clearly explain the content and colors • Labels on the x and y axis so the numerical value can be understood • Other descriptors necessary for interpretation 8️⃣ ❑STEPS IN INFORMATICS EVALUATION Document your outcome evaluation ❑DESCRIPTION Without comprehensive documentation of your evaluation, the chance that any practice improvements occur becomes unlikely. Can use STARE-HI guidelines:

8. A nurse makes a clinical judgment that an African American man in a stressful job is more vulnerable to developing hypertension than a White man in the same or a similar situation. The nurse has formulated what type of nursing diagnosis? Actual Risk Possible Wellness

✅b. A clinical judgment that an individual, family, or community is more vulnerable to develop the problem than others in the same or similar situation is a Risk nursing diagnosis. CHAPTER 15 Diagnosing

12. The 33-year-old client had a traumatic amputation of the right forearm as a result of a work-related injury. Which referral by the rehabilitation nurse is most appropriate? 1. Physical therapist. 2. Occupational therapist. 3. Workers' compensation. 4. State rehabilitation commission.

1. The physical therapist focuses on evaluating, diagnosing movement dysfunctions (injured tissues and structures), and treating these issues. The PT helps restore movement and mobility. 2. ✅The occupational therapist focuses on evaluating and improving functional abili- ties to optimize independence and address activities of daily living, which would be an appropriate referral. 3. Workers' compensation is an insurance provider for the employer and employee to cover medical expenses and loss of wages. This is not an appropriate referral by the rehabilitation nurse. The client may need this referral but after the occupational therapist has worked with the client and determined the ability to perform skills. Content - Medical: Integrated Nursing Process - Implementation: Client Needs - Physiological Integrity, Physiological Adaptation: Cognitive Level - Application: Concept - Nursing Roles. Med-Surg success chapter 20 comprehensive final examination

8. Which client should the charge nurse of the day surgery unit assign to a new graduate nurse in orientation? 1. The client who had an arthroscopy with an AP of 110 and BP of 94/60. 2. The client with open reduction of the ankle who is confused. 3. The client with a total hip replacement who is being transferred to the ICU. 4. The client diagnosed with low back pain who has had a myelogram.

2. Confusion could be a sign of many complica- tions after surgery, so this client should not be assigned to an inexperienced nurse. 3. This client is being transferred to the ICU, which indicates the client is not stable; there- fore, this client should not be assigned to an inexperienced nurse. ✅4. A myelogram is a routine diagnostic test. With minimal instruction, an inexperienced nurse could care for this client. Content - Nursing Management: Integrated Nursing Process - Planning: Client Needs - Safe Effective Care Environment, Management of Care: Cognitive Level - Synthesis: Concept - Nursing Roles. Med-Surg success chapter 20 comprehensive final examination

71. The client diagnosed with colon cancer tells the nurse, "All I do is sit and watch TV all day. I can barely go to the bathroom." According to the Oncology Nursing Society's cancer fatigue scale, how would the nurse document the fatigue objectively? Select the number that best describes how you feel today 0-No fatigue 1-3 Mild fatigue 4-6Moderate Fatigue 7-9 Extreme Fatigue 10 The Worst Fatigue 1. Mild fatigue. 2. Moderate fatigue. 3. Extreme fatigue. 4. Worst fatigue ever.

2. Moderate fatigue would be fatigue occurring about 40% to 60% of the time. ✅3. Extreme fatigue occurs 70% to 90% of the time, which is indicated by the client still being able to watch TV and get to the bathroom. 4. The worst fatigue ever occurs all the time and the client spends most of the day sleeping and is not able to stay awake to watch television. Content - Medical: Integrated Nursing Process - Implementation: Client Needs - Safe Effective Care Environment, Management of Care: Cognitive Level - Application: Concept - Cellular Regulation. Med-Surg success chapter 20 comprehensive final examination

97. The female client is admitted to the orthopedic floor with a spiral fracture of the arm and multiple contusions and abrasions covering the trunk of the body. Her husband accompanies her. During the admission interview, which intervention is priority? 1. Notify the local police department of the client's admission. 2. Provide privacy to discuss how the injuries occurred to the client. 3. Refer the client to the social worker for names of women's shelters. 4. Ask the client if she prefers the husband to stay in the room.

1. The police can be notified if the woman requests this course of action; otherwise this cannot be done, but it is not priority at this time. ✅2. The nurse must ensure the husband cannot hear the client discussing how she was injured. The client needs to feel safe when answering these questions because a spiral fracture indicates a twisting motion and the bruises are on areas covered with clothing. The nurse should suspect abuse with these types of injuries. 3. The nurse should refer to the social worker if it is determined the client has been abused, but the nurse should not refer during the admission interview. 4. The nurse should make every attempt to in- terview the client without the possible abuser present; the client will probably be afraid to tell the nurse she wants the husband to leave the room if he is the abuser. Content - Nursing Management: Integrated Nursing Process - Implementation: Client Needs - Safe Effective Care Environment, Management of Care: Cognitive Level - Application: Concept - Safety. Med-Surg success chapter 20 comprehensive final examination

78. Which priority problem should the clinic nurse identify for the client who is greater than ideal body weight and weighs 87 kg? 1. Risk for complications. 2. Altered nutrition. 3. Body image disturbance. 4. Activity intolerance.

1. This client is overweight but not morbidly obese, which would place the client at risk for complications. ✅2. "Altered nutrition:more than body requirements" is an appropriate client problem for a client who weighs 175 pounds. 3. This is a psychosocial problem, which is not priority over a physiological problem. 4. The client may or may not be active, but altered nutrition is priority. Content - Medical: Integrated Nursing Process - Diagnosis: Client Needs - Physiological Integrity, Basic Care and Comfort: Cognitive Level - Analysis: Concept - Self Med-Surg success chapter 20 comprehensive final examination

14. The nurse identifies the problem "high risk for complications" for the client with a right total hip replacement who is being discharged from the hospital. Which problem would have the highest priority? 1. Self-care deficit. 2. Impaired skin integrity. 3. Abnormal bleeding. 4. Prosthetic infection.

2. The client is being discharged and is am- bulating; therefore, impaired skin integrity should not be a problem. 3. The client would have been taking a prophy- lactic anticoagulant but would not be at risk for abnormal bleeding. ✅4. The client must inform all HCPs, espe- cially the dentist, of the hip prosthesis because the client should be taking pro- phylactic antibiotics prior to any invasive procedure. Any bacteria invading the body may cause an infection in the joint, and this may result in the client having the prosthesis removed. Content - Surgical: Integrated Nursing Process - Diagnosis: Client Needs - Safe Effective Care Environment, Management of Care: Cognitive Level - Analysis: Concept - Mobility. Med-Surg success chapter 20 comprehensive final examination

37. The nurse and the unlicensed assistive personnel (UAP) are caring for clients on a medical floor. Which nursing task could be delegated to the UAP? 1. Retake the BP on a client who received a STAT nitroglycerin sublingual. 2. Notify the health-care provider of the client's elevated blood pressure. 3. Obtain and document the routine vital signs on all the clients on the floor. 4. Call the laboratory technician and discuss a hemolyzed blood specimen.

1. This client is unstable and received medication for chest pain. The nurse cannot delegate any task for a client who is unstable. 2. The UAP cannot notify the HCP because UAPs are not allowed to take verbal or telephone orders. ✅3. The UAP can take routine vital signs. The nurse must evaluate the vital signs and take action if needed. The nurse should not delegate teaching, assessing, evaluat- ing, or any client who is unstable. 4. This is outside the level of a UAP's expertise. Content - Nursing Management: Integrated Nursing Process - Planning: Client Needs - Safe Effective Care Environment, Management of Care: Cognitive Level - Synthesis: Concept - Nursing Roles. Med-Surg success chapter 20 comprehensive final examination

44. The 24-year-old African American female client tells the nurse she has a brother with sickle cell disease. She is engaged to be married and is concerned about giving this disease to her future children. Which information is most important to provide to the client? 1. Tell the client that she won't pass this on if she has never had symptoms. 2. Encourage the client to discuss this concern with her fiancé. 3. Recommend that she and her fiancé see a genetic counselor. 4. Discuss the possibility of adopting children after she gets married. Content - Medical: Integrated Nursing Process - Implementation: Client Needs - Physiological Integrity, Physiological Adaptation: Cognitive Level - Application: Concept - Family.

1. This is a false statement. The client could have the sickle cell trait. 2. This should be discussed with her fiancé, but it is not the most important information. ✅3. Referral to a genetic counselor is the most important information to give the client. If she and her fiancé both have the sickle cell trait, there is a 25% chance of a child having sickle cell disease with each pregnancy. 4. Adoption may be a choice, but at this time the most important information is to refer the couple to a genetic counselor Med-Surg success chapter 20 comprehensive final examination

20. The 25-year-old client who has a C6 spinal cord injury is crying and asks the nurse, "Why did I have to survive? I wish I was dead." Which statement is the nurse's best response? 1. "Don't talk like that. At least you are alive and able to talk." 2. "God must have something planned for your life. Pray about it." 3. "You survived because the people at the accident saved your life." 4. "This must be difficult to cope with. Would you like to talk?"

1. This is negating the client's feelings and will abruptly end any conversation the client may want or need to have 2. This is imposing the nurse's religious beliefs on the client and these are clichés, which do not address the client's feelings. 3. This is explaining why the client survived, but the client isn't really asking for information. The client is expressing and showing emo- tions that must be addressed by the nurse ✅4. This is a therapeutic response which allows the client to ventilate feelings. Content - Medical: Integrated Nursing Process - Implementation: Client Needs - Psychosocial Integrity: Cognitive Level - Application: Concept - Trauma Med-Surg success chapter 20 comprehensive final examination

38. The client with venous insufficiency tells the nurse, "The doctor just told me about my disease and walked out of the room. What am I supposed to do?" Which statement is the nurse's best response? 1. "I will have your HCP come back and discuss this with you." 2. "One thing you can do is elevate your legs above your heart while watching TV." 3. "You will probably need to have surgery within a few months." 4. "This will go away after you lose about 20 pounds and start walking."

1. This might be what the nurse wants to do, but the nurse should teach the client about the disease process. ✅2. Elevating the legs above the heart as much as possible will help decrease edema. 3. There are no surgical procedures to correct venous insufficiency. 4. Losing weight and walking are excellent life- style modifications, but there is no guarantee the venous insufficiency will resolve. Content - Medical: Integrated Nursing Process - Implementation: Client Needs - Physiological Integrity, Physiological Adaptation: Cognitive Level - Application: Concept - Perfusion. Med-Surg success chapter 20 comprehensive final examination

40. The client diagnosed with non-Hodgkin's lymphoma tells the nurse, "I am so tired. I just wish I could die." Which stage of the grieving process does this statement represent? 1. Anger. 2. Denial. 3. Bargaining. 4. Acceptance.

1. This statement does not represent the anger stage of grieving. 2. This statement does not represent the denial stage of grieving. 3. This statement does not represent the bar- gaining stage of grieving. ✅4. This statement indicates the client is ready to die and is in the acceptance stage of the grieving process. Content - Medical: Integrated Nursing Process - Diagnosis: Client Needs - Psychosocial Integrity: Cognitive Level - Analysis: Concept - Grief and Loss Med-Surg success chapter 20 comprehensive final examination

49. The client is performing breast self-examination (BSE) by the American Cancer Society's recommended steps and has completed palpating the breast. Which step is next when completing the BSE? 1. Stand before the mirror and examine the breast. 2. Lean forward and look for dimpling or retractions. 3. Examine the breast using a circular motion. 4. Pinch the nipple to see if any fluid can be expressed.

1. This step is the first step in BSE. 2. This is step three (3) in the BSE process. 3. This is included in steps four (4) and five (5) and is described as using a systematic process of examining the breast. Using circular mo- tions and dividing the breast into wedges or vertical strips to palpate the entire breast is encouraged. This step was described in the stem as having been completed. ✅4. The last step of BSE after palpation is to express the nipple by gently squeezing the nipple. Any discharge should be brought to the attention of an HCP. Nipple dis- charge can be caused by many factors such as carcinoma, papilloma, pituitary adenoma, cystic breasts, and some medications. Content - Medical: Integrated Nursing Process - Implementation: Client Needs - Health Promotion and Maintenance: Cognitive Level - Knowledge: Concept - Female Reproduction.

10. The primary nurse is applying antiembolism hose to the client who had a total hip replacement. Which situation warrants immediate intervention by the charge nurse? 1. Two fingers can be placed under the top of the band. 2. The peripheral capillary refill time is less than three (3) seconds. 3. There are wrinkles in the hose behind the knees. 4. The nurse does not place a hose on the foot with a venous ulcer.

1. This would not warrant intervention because this indicates the hose are not too tight. 2. This indicates the hose are not too tight. 3✅There should be no wrinkles in the hose after application. Wrinkles could cause constriction in the area, resulting in clot formation or skin breakdown; therefore, this would warrant immediate interven- tion by the charge nurse. 4. Antiembolism hose should not be put over a wound; they would restrict the circulation to the wound and cause a decrease in wound healing. Content - Nursing Management: Integrated Nursing Process - Evaluation: Client Needs - Safe Effective Care Environment, Management of Care: Cognitive Level - Synthesis: Concept - Nursing Roles. Med-Surg success chapter 20 comprehensive final examination

22. The nurse observes the unlicensed assistive personnel (UAP) taking vital signs on an unconscious client. Which action by the UAP warrants intervention by the nurse? 1. The UAP uses a vital sign machine to check the BP. 2. The UAP takes the client's temperature orally. 3. The UAP verifies the blood pressure manually. 4. The UAP counts the respirations for 30 seconds.

1. Using the vital sign machine to take the client's BP is an appropriate intervention. ✅2. The body temperature of an unconscious client should never be taken by mouth because the client is unable to safely hold the thermometer. 3. Verifying the blood pressure manually is an appropriate intervention if the UAP questions the automatic blood pressure reading. This action should be praised. 4. Counting the respiration for 30 seconds and multiplying by two (2) is appropriate. Content - Medical: Integrated Nursing Process - Evaluation: Client Needs - Safe Effective Care Environment, Management of Care: Cognitive Level - Synthesis: Concept - Nursing Roles. Med-Surg success chapter 20 comprehensive final examination

16. The nurse is planning the care for the client with multiple stage IV pressure ulcers. Which complication results from these pressure ulcers? 1. Wasting syndrome. 2. Osteomyelitis. 3. Renal calculi. 4. Cellulitis.

1. Wasting syndrome occurs in clients with protein-calorie malnutrition. This syndrome leads to the pressure ulcers not healing, but it is not a complication of the pressure ulcers. 2✅Stage IV pressure ulcers frequently extend to the bone tissue, predisposing the client to developing a bone infection— osteomyelitis—which can rarely be treated effectively. 3.Renal calculi may be a result of immobility, but they are not a complication of pressure ulcers. 4 Cellulitis is an inflammation oftheskin, which is not a complication of pressure ulcers. Content - Medical: Integrated Nursing Process - Planning: Client Needs - Physiological Integrity, Physiological Adaptation: Cognitive Level - Synthesis: Concept - Skin Integrity Med-Surg success chapter 20 comprehensive final examination

58. The nurse is preparing the client newly diagnosed with asthma for discharge. Which data indicate the teaching about the peak flowmeter has been effective? 1. "I can continue my usual activities without medication if I am in the yellow zone." 2. "It takes one (1) to two (2) days to establish my personal best." 3. "When I can't talk while walking, I need to take my quick-relief medicine." 4. "When I am in the red zone, I must take my quick-relief medication and not exercise."

1. Yellow means caution. The client should follow some, but not all, usual activities. 2. The client's personal best takes two (2) to three (3) weeks to establish. 3. When a client can't talk while walking, there is shortness of breath which indicates the client does not have tight control, but this has nothing to do with the peak flowmeter. ✅4. When the client is in the red zone, the client should take the quick-relief medication and should not exercise or follow regular routines. Content - Medical: Integrated Nursing Process - Evaluation: Client Needs - Physiological Integrity, Physiological Adaptation: Cognitive Level - Synthesis: Concept - Oxygenation. Med-Surg success chapter 20 comprehensive final examination

7. The client diagnosed with Stage IV prostate cancer is receiving chemotherapy. Which laboratory value should the nurse assess prior to administering the chemotherapy? 1. Prostate-specific antigen (PSA). 2. Serum calcium level. 3. Complete blood count (CBC). 4. Alpha-fetoprotein (AFP).

1. is a tumor marker monitored to determine the progress of the disease and treatment, but it is not monitored prior to ✅3. The CBC is monitored to determine if the client is at risk for developing an infection or bleeding as a result of side effects of the chemotherapy medications. The chemo- therapy could be held or decreased based on these results. 4. AFP is a tumor marker monitored to deter- mine the progress of the disease and treatment, but it is not monitored prior to chemotherapy. Content - Medical: Integrated Nursing Process - Assessment: Client Needs - Physiological Integrity, Reduction of Risk Potential: Cognitive Level - Analysis: Concept - Cellular Regulation. Med-Surg success chapter 20 comprehensive final examination

32. The client is diagnosed with aortic stenosis. Which assessment data indicate a complication is occurring? 1. Barrel chest and clubbing of the fingers. 2. Intermittent claudication and rest pain. 3. Pink, frothy sputum and dyspnea on exertion. 4. Bilateral wheezing and friction rub.

1.Barrel chest and clubbing of the fingers are signs of chronic lung disease 2. Intermittent claudication and rest pain are signs of peripheral arterial disease. ✅3. Pink, frothy sputum and dyspnea on exertion are signs of congestive heart failure, which occurs when the heart can no longer compensate for the strain of an incompetent valve. 4. Friction rub occurs with pericarditis, and bilateral wheezing occurs with asthma. Content - Medical: Integrated Nursing Process - Assessment: Client Needs - Physiological Integrity, Reduction of Risk Potential: Cognitive Level - Synthesis: Concept - Perfusion. Med-Surg success chapter 20 comprehensive final examination

28. Which client should the charge nurse on the substance abuse unit assign to the licensed practical nurse (LPN)? 1. The client with chronic alcoholism who has been on the unit three (3) days. 2. The client who is complaining of palpitations and has a history of cocaine abuse. 3. The client diagnosed with amphetamine abuse who tried to commit suicide. 4. The client diagnosed with cannabinoid abuse who is threatening to leave AMA.

1.The client should be assessed for delirium tremens and should be assigned to a regis- tered nurse. 2. Palpitations indicate cardiac involvement, and because the client has a history of cocaine abuse, this client should be assigned to a reg- istered nurse. 3. This client is at high risk for injury to self and should be assigned to a registered nurse and be on one-to-one precautions. ✅4. The client has a right to leave against medical advice (AMA), and marijuana abuse is not life threatening to him or to others. Therefore, the LPN could be assigned to this client. Content - Nursing Management: Integrated Nursing Process - Planning: Client Needs - Safe Effective Care Environment, Management of Care: Cognitive Level - Synthesis: Concept - Nursing Roles Med-Surg success chapter 20 comprehensive final examination

64. The client receiving a continuous heparin drip complains of sudden chest pain on inspiration and tells the nurse, "Something is really wrong with me." Which intervention should the nurse implement first? 1. Increase the heparin drip rate. 2. Notify the health-care provider. 3. Assess the client's lung sounds. 4. Apply oxygen via nasal cannula.

1.The heparin drip may be increased because the client has now thrown a pulmonary embolus (PE), but this needs an HCP's order. 2. The HCP will be notified because the client has a suspected embolus, but it is not the first intervention. 3. The client has probably thrown a pulmonary embolus, and assessing the lungs will not do anything for a client who may die. PEs are life threatening, and assessing the client is not priority in a life-threatening situation. ✅4. The client probably has a pulmonary embolus, and the priority is to provide ad- ditional oxygen so oxygenation of tissues can be maintained. Content - Medical: Integrated Nursing Process - Implementation: Client Needs - Safe Effective Care Envi- ronment, Management of Care: Cognitive Level - Application: Concept - Oxygenation Med-Surg success chapter 20 comprehensive final examination

46. The client is being admitted into the hospital with a diagnosis of pneumonia. Which HCP order should the nurse implement first? 1. Initiate intravenous antibiotics. 2. Collect a sputum specimen for culture. 3. Obtain a clean voided midstream urinalysis. 4. Request a chest x-ray to confirm the diagnosis.

1.The nurse should not administer antibiotics until the culture specimen is obtained. ✅2. The sputum must be collected first to identify the infectious organism so ap- propriate antibiotics can be prescribed. Administering broad-spectrum antibiotics prior to collecting sputum could alter the C&S results. 3. This is not priority over sputum culture and getting the antibiotic started. 4. Always treat the client first. Content - Medical: Integrated Nursing Process - Implementation: Client Needs - Safe Effective Care Environment, Management of Care: Cognitive Level - Application: Concept - Infection. Med-Surg success chapter 20 comprehensive final examination

11. The 54-year-old female client is diagnosed with osteoporosis. Which interventions should the nurse discuss with the client? Select all that apply. 1. Instruct the client to swim 30 minutes every day. 2. Encourage drinking milk with added vitamin D. 3. Determine if the client smokes cigarettes. 4. Recommend the client not go outside. 5. Teach about safety and fall precautions.

1.The nurse should suggest walking daily be- cause bones need stress to maintain strength. ✅2. Vitamin D helps the body absorb calcium. ✅3. Smoking interferes with estrogen's protective effects on bones, promoting bone loss. 4 Lack of exposure to sunlight results in decreased vitamin D, which is necessary for calcium absorption and normal bone mineral- ization. The client should go outside. ✅5 The client is at risk for fractures; therefore, a fall could result in serious complications. Content - Medical: Integrated Nursing Process - Planning: Client Needs - Physiological Integrity, Physiological Adaptation: Cognitive Level - Synthesis: Concept - Nursing Roles. Med-Surg success chapter 20 comprehensive final examination

85. The client is admitted into the medical unit diagnosed with heart failure and is prescribed the thyroid hormone levothyroxine (Synthroid) orally. Which intervention should the nurse implement? 1. Call the pharmacist to clarify the order. 2. Administer the medication as ordered. 3. Ask the client why he or she takes Synthroid. 4. Request serum thyroid function levels.

1.There is no reason to question or clarify this order; the nurse is responsible for clarifying the order with the HCP, not the pharmacist. ✅2. Many elderly clients have comorbid condi- tions requiring daily medications, which are not the primary reason for admission into the hospital. The nurse should know why the client is taking this medication; this medication is prescribed for only one reason, hypothyroidism. The serum thyroid function levels are monitored by the HCP usually yearly after maintenance doses have been established Content - Medical: Integrated Nursing Process - Implementation: Client Needs - Safe Effective Care Environment, Safety and Infection Control: Cognitive Level - Application: Concept - Medication. Med-Surg success chapter 20 comprehensive final examination

89. The unlicensed assistive personnel (UAP) empties the indwelling urinary catheter for a client who is four (4) hours postoperative transurethral resection of the prostate and informs the nurse the urine is red with some clots. Which intervention should the nurse implement first? 1. Assess the client's urine output immediately. 2. Notify the HCP that the client has gross hematuria. 3. Explain this is expected with this surgery. 4. Medicate for bladder spasms to decrease bleeding.

1.This is a normal postoperative expectation with this procedure 2. This is gross hematuria, but it is expected with this type of surgery and the nurse should not call the surgeon. ✅3. The client has a three (3)-way indwelling 30-mL catheter inserted in surgery. This type of catheter instills an irrigant into the bladder to flush the clots and blood from the bladder; bloody urine is expected after this surgery. 4. The stem does not indicate the client is hav- ing bladder spasms and bladder spasms are not causing the bleeding. Clots left in the bladder and not flushed out can cause bladder spasms. Content - Su Content - Surgical: Integrated Nursing Process - Implementation: Client Needs - Safe Effective Care Environment, Management of Care: Cognitive Level - Analysis: Concept - Perioperative. Med-Surg success chapter 20 comprehensive final examination

23. The client diagnosed with a brain tumor who had radiation treatment and developed alopecia asks, "When will my hair grow back?" Which statement is the nurse's best response? 1. "Your hair should start growing back within three (3) weeks." 2. "Are you concerned your hair will not grow back?" 3. "It may take months, if your hair grows back at all." 4. "It may take a couple of years for the hair to grow back."

1.This is incorrect information for radiation therapy. It is correct for chemotherapy 2. This is a therapeutic response, which does not answer the client's question. ✅3. Radiation therapy can cause permanent damage to the hair follicles and the hair may not grow back at all; the nurse should answer the client's question honestly. 4. This is not a true statement. Content - Medical: Integrated Nursing Process - Implementation: Client Needs - Physiological Integrity, Physiological Adaptation: Cognitive Level - Application: Concept - Cellular Regulation. Med-Surg success chapter 20 comprehensive final examination

90. The client with a history of substance abuse presents to the emergency department complaining of right flank pain, and the urinalysis indicates microscopic blood. Which intervention should the nurse implement? 1. Determine the last illegal drug use. 2. Insert a #22 French indwelling catheter. 3. Give the client a back massage. 4. Medicate the client for pain.

1.This is not pertinent to the client's current situation 2. The nurse should strain all the client's urine, but a large indwelling catheter does not need to be inserted into this client; this isn't a blad- der stone, it is a ureteral stone. 3. A back massage is a nice thing to do, but it will not help renal colic caused by ureteral calculi. ✅4. The client should be medicated for pain, which is excruciating, and the client's history of substance abuse should not be an issue. Content - Medical: Integrated Nursing Process - Implementation: Client Needs - Safe Effective Care Environment, Management of Care: Cognitive Level - Analysis: Concept - Comfort. Med-Surg success chapter 20 comprehensive final examination

88. Which action by the unlicensed assistive personnel (UAP) requires intervention by the nurse? 1. The UAP used two (2) washcloths when washing the perineal area. 2. The UAP emptied the indwelling catheter and documented the amount. 3. The UAP applied moisture barrier cream to the anal area. 4. The UAP is wiping the client's perineal area from back to front.

1.Using two (2) washcloths to clean the client's perineal area is an appropriate action to prevent UTI 2. This action does not require intervention 3. Moisture barrier cream is not considered a medication and can be applied by the UAP after the perineum is cleaned. ✅4. The UAP should wipe the area from front to back to prevent fecal contamination of the urinary meatus, which could result in a urinary tract infection. Content - Nursing Management: Integrated Nursing Process - Evaluation: Client Needs - Safe Effective Care Environment, Management of Care: Cognitive Level - Synthesis: Concept - Nursing Roles. Med-Surg success chapter 20 comprehensive final examination

48. The client comes to the emergency department complaining of pain in the right forearm. The nurse notes a large area of redness and edema over the forearm, and the client has an elevated temperature. Which condition should the nurse suspect? 1. Cellulitis. 2. Intravenous drug abuse. 3. Raynaud's phenomenon. 4. Thromboangiitis obliterans.

1.✅ Cellulitis is the most common infectious cause of limb edema as a result of bacte- rial invasion of the subcutaneous tissue. This assessment would make the nurse suspect this condition. 2. Intravenous drug use can cause cellulitis, but the assessment did not include track marks or needle insertion sites. 3. Raynaud's phenomenon is a form of intermittent arteriolar vasoconstriction resulting in coldness, pain, and pallor of fingertips or toes. The client should keep warm to prevent vasoconstriction of extremities. 4. Buerger's disease (thromboangiitis obliterans) is a relatively uncommon occlusive disease limited to medium and small arteries and veins. The cause is unknown, but there is a strong association with tobacco use. Content - Medical: Integrated Nursing Process - Assessment: Client Needs - Physiological Integrity, Reduction of Risk Potential: Cognitive Level - Analysis: Concept - Skin Integrity.

17. The client comes to the clinic complaining of itching on the left wrist near a wristwatch. The nurse notes an erythematous area along with pruritic vesicles around the left wrist. Which condition should the nurse suspect? 1. Contact dermatitis. 2. Herpes simplex 1. 3. Impetigo. 4. Seborrheic dermatitis.

1✅ Contact dermatitis is a type of dermatitis caused by a hypersensitivity response. In this case, it is a hypersensitivity reaction to metal salts in the watch the client is wearing. Anytime the nurse assesses red- ness or irritation in areas where jewelry (such as rings, watches, necklaces) or clothing (such as socks, shoes, or gloves) are worn, the nurse should suspect con- tact dermatitis. Herpes simplex 1 virus occurs in oral or nasal mucous membranes. Impetigo is a superficial infection of the skin caused by staph or strep infection and occurs on the body, face, hands, or neck. Seborrheic dermatitis is a chronic inflamma- tion of the skin involving the scalp, eyebrows, eyelids, ear canals, nasolabial folds, axillae, and trunk. Content - Medical: Integrated Nursing Process - Diagnosis: Client Needs - Safe Effective Care Environment, Management of Care: Cognitive Level - Comprehension: Concept - Skin Integrity. Med-Surg success chapter 20 comprehensive final examination

95. Which psychosocial problem should the nurse identify as priority for a client diagnosed with rheumatoid arthritis? 1. Alteration in comfort. 2. Ineffective coping. 3. Anxiety. 4. Altered body image.

. 1. Alteration in comfort is a client problem, but it not a psychosocial problem 2. Ineffective coping is a problem that is not applicable to all clients with rheumatoid arthritis and is a very individualized problem; the test taker would need more information before selecting this as a correct answer. 3. Anxiety is a problem that is not applicable to all clients with rheumatoid arthritis and is a very individualized problem; the test taker would need more information before select- ing this as a correct answer. ✅4. Altered body image is an expected psychosocial problem for all clients with rheumatoid arthritis because of the joint deformities. Content - Medical: Integrated Nursing Process - Diagnosis: Client Needs - Psychosocial Integrity: Cognitive Level - Analysis: Concept - Immunity. Med-Surg success chapter 20 comprehensive final examination

19. The client with a closed head injury is admitted to the neurointensive care department following a motor-vehicle accident. Which goal is an appropriate short-term goal for the client? 1. The client will maintain optimal level of functioning. 2. The client will not develop extremity contractures. 3. The client's intracranial pressure will not be greater than 15 mm Hg. 4. The client will be able to verbalize feelings of anger.

1. This could be an appropriate long-term goal for the client based on the extent of injury, but it is not an appropriate short-term goal. 2. This is an appropriate long-term goal to prevent immobility complications, but it is not an appropriate short-term goal. ✅3. The worse-case scenario with a closed head injury is increased intracranial pres- sure resulting in death. An appropriate short-term goal would be the ICP re- maining within normal limits, which is 5 to 15 mm Hg. 4. This is a psychosocial goal, which would not be a short-term goal, and the client may not be angry. The stem did not indicate the client is angry. Content - Medical: Integrated Nursing Process - Planning: Client Needs - Safe Effective Care Environment, Management of Care: Cognitive Level - Synthesis: Concept - Neurologic Regulation. Med-Surg success chapter 20 comprehensive final examination

18. Which diagnostic test should the nurse expect to be ordered for the client who has a nevus which is purple and brown with irregular borders? 1. Bone scan. 2. Skin biopsy. 3. Carcinoembryonic antigen (CEA). 4. Sonogram.

1. A bone scan would not be ordered unless a biopsy proves malignant melanoma. ✅2. This is an abnormal-appearing mole on the skin, and the HCP would order a biopsy to confirm skin cancer 3.A CEA is a test used to mark the presence or prognosis of several cancers but not skin cancer. Content - Medical: Integrated Nursing Process - Planning: Client Needs - Physiological Integrity, Reduction of Risk Potential: Cognitive Level - Synthesis: Concept - Skin Integrity. Med-Surg success chapter 20 comprehensive final examination

54. Which laboratory result warrants immediate intervention by the nurse for the female client diagnosed with systemic lupus erythematosus (SLE)? 1. A hemoglobin and hematocrit of 13 g/dL and 40%. 2. An erythrocyte sedimentation rate of 9 mm/hr. 3. A serum albumin level of 4.5 g/dL. 4. A white blood cell count of 15,000/mm3.

1. A normal hemoglobin is 12 to 15 g/dL, and normal hematocrit is 36% to 45%. 2. A normal ESR is between 1 and 20 mm/hr for a female client. 3. A normal albumin level is between 3.5 and 5 g/dL. ✅4. The client with SLE is at an increased risk for infection, and this WBC count indicates an infection requiring medical intervention. Content - Medical: Integrated Nursing Process - Assessment: Client Needs - Physiological Integrity, Reduction of Risk Potential: Cognitive Level - Analysis: Concept - Immunity. Med-Surg success chapter 20 comprehensive final examination

24. Which assessment data indicate the treatment for the client diagnosed with bacterial meningitis is effective? 1. There is a positive Brudzinski's sign and photophobia 2. The client tolerates meals without nausea. 3. There is a positive Kernig's sign and an elevated temperature. 4. The client is able to flex the neck without pain.

1. A positive Brudzinski's sign—flexion of the knees and hip when the neck is flexed— indicates the presence of meningitis. There- fore, the treatment is not effective. Sensitivity to light is a common symptom of meningitis. 2. This does not indicate whether the meningi- tis is resolving. 3. Kernig's sign—the leg cannot be extended when the client is lying with the thigh flexed on the abdomen—is a sign of meningitis. An elevated temperature indicates the client still has meningitis. ✅ 4. The client does not have nuchal rigidity, which indicates the client's treatment is effective. Content - Medical: Integrated Nursing Process - Assessment: Client Needs - Physiological Integrity, Physiological Adaptation: Cognitive Level - Analysis: Concept - Neurologic Regulation. Med-Surg success chapter 20 comprehensive final examination

59. Which assessment data indicate the client has developed a deep vein thrombosis (DVT) in the left leg? 1. A negative Homans' sign of the left leg. 2. Increased left-leg calf circumference. 3. Elephantiasis of the left lower leg. 4. Brownish pigmentation of the left lower leg.

1. A positive Homans' sign would indicate a DVT. ✅2. The calf with deep vein thrombosis becomes edematous, so there is an increase in the size of the calf when compared to the other leg 3.Elephantiasis is characterized by tremendous edema usually of the external genitalia and legs and is not associated with DVT. Elephantiasis is a lymphatic problem, not a venous problem. 4. The brownish discoloration is a sign/ symptom of chronic venous insufficiency Content - Medical: Integrated Nursing Process - Assessment: Client Needs - Physiological Integrity, Reduction of Risk Potential: Cognitive Level - Analysis: Concept - Clotting Med-Surg success chapter 20 comprehensive final examination

25. The client is being evaluated to rule out Parkinson's disease. Which diagnostic test confirms this diagnosis? 1. A positive magnetic resonance imaging (MRI) scan. 2. A biopsy of the substantia nigra. 3. A stereotactic pallidotomy. 4. There is no test that confirms this diagnosis.

1. An MRI is not able to confirm the diagnosis of Parkinson's disease. 2 This is the portion of the brain where Parkinson's disease originates, but this area lies deep in the brain and cannot be biopsied. 3. This is a surgery that relieves some of the symptoms of Parkinson's disease. To be eli- gible for this procedure, the client must have failed to achieve an adequate response with medical treatment. ✅4. Many diagnostic tests are completed to rule out other diagnoses, but Parkinson's disease is diagnosed based on the clinical presentation of the client and the pres- ence of two of the three cardinal mani- festations: tremor, muscle rigidity, and bradykinesia. Med-Surg success chapter 20 comprehensive final examination

29. The telemetry nurse is monitoring the following clients. Which client should the telemetry nurse instruct the primary nurse to assess first? 1. The client who has occasional premature ventricular contractions (PVCs). 2. The client post-cardiac surgery who has three (3) unifocal PVCs in a minute. 3. The client with a myocardial infarction who had two (2) multifocal PVCs. 4. The client diagnosed with atrial fibrillation who has an AP of 116 and no P wave.

1. An occasional PVC does not warrant intervention; it is normal for most clients 2 . Less than six (6) unifocal PVCs in one (1) minute is not life threatening. ✅3. Multifocal PVCs indicate the ventricle is irritable, and this client is at risk for a car- diac event such as ventricular fibrillation. 4. Atrial fibrillation is not life threatening, and the nurse would expect the client not to have a P wave when exhibiting this dysrhythmia. Content - Nursing Management: Integrated Nursing Process - Planning: Client Needs - Safe Effective Care Environment, Management of Care: Cognitive Level - Synthesis: Concept - Nursing Roles. Med-Surg success chapter 20 comprehensive final examination

26. The client diagnosed with a transient ischemic attack (TIA) is being discharged from the hospital. Which medication should the nurse expect the HCP to prescribe? 1. The oral anticoagulant warfarin (Coumadin). 2. The antiplatelet medication, a baby aspirin. 3. The beta blocker propranolol (Inderal). 4. The anticonvulsant valproic acid (Depakote).

1. An oral coagulant is ordered if the TIA was caused by atrial fibrillation, and that informa- tion is not presented in the stem. ✅2. Atherosclerosis is the most common cause of a TIA or stroke, and taking a baby aspirin every day helps prevent clot formation around plaques. 3. If the client had hypertension, a beta blocker may be prescribed, but this information is not in the stem. 4. Anticonvulsant medications are not prescribed to help prevent TIAs. Content - Medical: Integrated Nursing Process - Planning: Client Needs - Physiological Integrity, Pharmacological and Parenteral Therapies: Cognitive Level - Synthesis: Concept - Medication. Med-Surg success chapter 20 comprehensive final examination

1. The 44-year-old female client calls the clinic and tells the nurse she felt a lump while performing breast self-examination (BSE). Which question should the nurse ask the client? 1. "Are you taking birth control pills?" 2. "Do you eat a lot of chocolate?" 3. "When was your last period?" 4. "Are you sexually active?"

1. Birth control pills regulate the hormones in the body but will not cause changes in the breast tissue. 2. There is a theory that chocolate increases breast discomfort in women with fibrocystic breast changes. ✅3. During the menstrual cycle, pregnancy, and menopause, variations in breast tissue occur and must be distinguished from pathologi- cal disease. BSE is best performed on days five (5) to seven (7) after menses, counting the first day of menses as day one (1). 4. Sexual manipulation of the breast does not cause malignant changes in breast tissue Content - Medical: Integrated Nursing Process - Assessment: Client Needs - Physiological Integrity, Reduction of Risk Potential: Cognitive Level - Analysis: Concept - Female Reproduction. Med-Surg SucceSS chapter 20 comprehensive final examination

41. The nurse writes the goal "the client will list three (3) food sources of vitamin B12" for the client diagnosed with pernicious anemia. Which foods listed by the client indicate the goal has been met? 1. Brown rice, dried fruits, and oatmeal. 2. Beef, chicken, and pork 3. Broccoli, asparagus, and kidney beans. 4. Liver, cheese, and eggs.

1. Brown rice, dried fruit, and oatmeal are sources of nonheme iron. Nonheme iron comes from vegetable sources. 2. Beef, chicken, and pork are sources of heme iron or animal sources of iron. 3. Broccoli, asparagus, and kidney beans are sources of folic acid. ✅4. Liver, cheese, and eggs are sources of vitamin B12. Content - Medical: Integrated Nursing Process - Evaluation: Client Needs - Physiological Integrity, Basic Care and Comfort: Cognitive Level - Synthesis: Concept - Hematologic Regulation. Med-Surg success chapter 20 comprehensive final examination

39. The client is admitted with rule-out leukemia. Which assessment data support the diagnosis of leukemia? 1. Cervical lymph node enlargement. 2. An asymmetrical dark-purple nevus. 3. Petechiae covering the trunk and legs. 4. Brownish-purple nodules on the face.

1. Cervical lymph node enlargement would indicate Hodgkin's lymphoma 2. An asymmetrical dark-purple nevus would indicate malignant melanoma. ✅3. Petechiae covering the trunk and legs is one of the indicators of bone marrow problems, which could be leukemia. 4. Brownish-purple nodules on the face indicate Kaposi's sarcoma, a complication of AIDS. Content - Medical: Integrated Nursing Process - Assessment: Client Needs - Physiological Integrity, Reduction of Risk Potential: Cognitive Level - Analysis: Concept - Hematologic Regulation. Med-Surg success chapter 20 comprehensive final examination

50. Which assessment information is the most critical indicator of a neurological deficit? 1. Changes in pupil size. 2. Level of consciousness. 3. A decrease in motor function. 4. Numbness of the extremities.

1. Changes in pupil size are a late sign of a neurological deficit. ✅2. A change in level of consciousness is the first and most critical indicator of any neurological deficit. 3. A decrease in motor function occurs with a neurological deficit, but it is not the most critical indicator. 4. Numbness of the extremities occurs with a neurological deficit, but it is not the most critical indicator. Content - Medical: Integrated Nursing Process - Assessment: Client Needs - Physiological Integrity, Reduction of Risk Potential: Cognitive Level - Analysis: Concept - Assessment.

100. Which intervention will help prevent the nurse from being sued for malpractice throughout his or her professional practice? 1. Keep accurate and legible documentation of client care. 2. A kind, caring, and compassionate bedside manner at all times. 3. Maintain knowledge of medications for disease processes. 4. Follow all health-care provider orders explicitly.

1. Documentation can help the nurse defend his or her actions if a lawsuit occurs, but it will not help prevent a lawsuit. ✅2. Research indicates nurses who form a trusting nurse-client relationship are less likely to be sued; if the nurse were to make an error, the client and family are often more forgiving. 3. Knowledge of medications will prevent medication errors but will not keep the nurse from being sued. Nurses are human and can make mistakes with medications even if they are knowledgeable. 4.The nurse is a client advocate and is legally, morally, and ethically required to question the HCP's orders when caring for assigned clients. Content - Nursing Management: Integrated Nursing Process - Implementation: Client Needs - Safe Effective Care Environment, Management of Care: Cognitive Level - Application: Concept - Legal. Med-Surg success chapter 20 comprehensive final examination

34. Which question should the nurse ask the client who is being admitted to rule out infective endocarditis? 1. "Do you have a history of a heart attack?" 2. "Have you had a cardiac valve replacement?" 3. "Is there a family history of rheumatic heart disease?" 4. "Do you take nonsteroidal anti-inflammatory medications?"

1. Having a history of a myocardial infarction is not a risk factor for developing infective endocarditis. ✅2. This is why clients must receive prophy- lactic antibiotic treatment before dental work and invasive procedures. 3. A personal history of rheumatic fever, not a family history, increases the risk of developing infective endocarditis. 4. NSAIDs have no effect on the development of infective endocarditis Content - Medical: Integrated Nursing Process - Assessment: Client Needs - Physiological Integrity, Reduction of Risk Potential: Cognitive Level - Analysis: Concept - Perfusion. Med-Surg success chapter 20 comprehensive final examination

5. The nurse has taught Kegel exercises to the client who is para 5, gravida 5. Which information indicates the exercises have been effective? 1. The client reports no SOB when walking up stairs. 2. The client has no complaints of stress incontinence. 3. The client denies being pregnant at this time. 4. The client has lost 10 lb in the last two (2) months.

1. Kegel exercises do not have anything to do with activity endurance. ✅2. Kegel exercises are exercises that strengthen the perineal muscles. Multiple pregnancies weaken the pelvic muscles, resulting in bladder incontinence; a report of no stress incontinence indicates the Kegel exercises are effective. Kegel exercises do not affect pregnancy. Kegel exercises do not have anything to do with weight loss. Content - Medical: Integrated Nursing Process - Evaluation: Client Needs - Physiological Integrity, Physiological Adaptation: Cognitive Level - Synthesis: Concept - Female Reproduction. Med-Surg success chapter 20 comprehensive final examination

62. Which psychosocial client problem should the nurse write for the client diagnosed with cancer of the lung and metastasis to the brain? 1. Altered role performance. 2. Grieving. 3. Body image disturbance. 4. Anger

1. Metastasis indicates advanced disease; therefore, altered role performance would not be an appropriate client problem. ✅2. Metastasis indicates advanced disease, and the client should be allowed to express feelings of loss and grieving; the client is dying. 3. Body image is a psychosocial problem but would not be applicable in this scenario. 4.Anger is part of the grieving process. Content - Medical: Integrated Nursing Process - Diagnosis: Client Needs - Psychosocial Integrity: Cognitive Level - Analysis: Concept - Grief and Loss. Med-Surg success chapter 20 comprehensive final examination

27. The nurse has just received the shift assessment. Which client should the nurse assess first? 1. The client with encephalitis who has myalgia. 2. The client who is complaining of chest pain. 3. The client who refuses to eat hospital food. 4. The client who is scheduled to go to the whirlpool.

1. Myalgia is muscle pain, which is expected in a client diagnosed with encephalitis. ✅2.The client complaining of chest pain is priority. Remember Maslow's hierarchy of needs. 3. Refusing to eat hospital food is not a priority. The client going to the whirlpool is stable and is not a priority over chest pain. Med-Surg success chapter 20 comprehensive final examination

33. The client who has just received a permanent pacemaker is admitted to the telemetry floor. The nurse writes the problem "knowledge deficit." Which interventions should be included in the plan of care? Select all that apply. 1. Take tub baths instead of showers the rest of his or her life. 2. Do not hold electrical devices near the pacemaker. 3. Carry the pacemaker identification card at all times. 4. Count the radial pulse one (1) full minute every morning. 5. Notify the HCP if the pulse is 12 beats slower than the preset rate.

1. Once the chest incision heals, the client can shower or bathe, whichever the client prefers. ✅2. Electrical devices may interfere with the functioning of the pacemaker. ✅3. This alerts any HCP as to the presence of a pacemaker. ✅4. The client should be taught to take the radial pulse for one (1) full minute before getting out of bed. If the count is more than five (5) bpm less than the preset rate, the HCP should be notified immediately because this may indicate the pacemaker is malfunctioning. 5. The client should notify the HCP if the pulse is five (5) bpm less than the preset rate. This may indicate pacemaker malfunction. Content - Medical: Integrated Nursing Process - Planning: Client Needs - Physiological Integrity, Physiological Adaptation: Cognitive Level - Synthesis: Concept - Perfusion. Med-Surg success chapter 20 comprehensive final examination

4. The nurse is teaching a class on sexually transmitted diseases to high school sophomores. Which information should be included in the discussion? 1. Oral sex decreases the chance of transmitting a sexual disease. 2. Sexual activity during menses decreases transmission of diseases. 3. Frequent sexual activity is necessary to transmit a sexual disease. 4. Unprotected sex puts the individual at risk for many diseases

1. Oral sex still involves mucous membrane-to- mucous membrane contact and disease trans- mission is possible; herpes simplex 2 is simply herpes simplex 1 transferred to the genitalia. 2. This is a myth. 3. The more often the person engages in sexual contact and the more partners he or she has, the more likely the person will contract an STD; however, one time is enough to contract a deadly STD, such as AIDS ✅4. According to developmental theories, adolescents think they are invincible and nothing will happen to them. This attitude leads adolescents to participate in high-risk behaviors without regard to consequences. Content - Medical: Integrated Nursing Process - Planning: Client Needs - Health Promotion and Maintenance: Cognitive Level - Synthesis: Concept - Promoting Health. Med-Surg success chapter 20 comprehensive final examination

36. The client diagnosed with atherosclerosis has coronary artery disease. The client experiences sudden chest pain when walking to the nurse's station. Which intervention should the nurse implement first? 1. Administer sublingual nitroglycerin. 2. Apply oxygen via nasal cannula. 3. Obtain a STAT electrocardiogram. 4. Have the client sit in a chair.

1. Sublingual nitroglycerin is the medication of choice for angina, but it is not the first intervention. 2. Applying oxygen is appropriate, but it is not the first intervention. 3. A STAT ECG should be ordered, but it is not the first intervention. ✅4. Stopping the client from whatever activity the client is doing is the first interven- tion because this decreases the oxygen demands of the heart muscle and may decrease or eliminate the chest pain. Content - Nursing Management: Integrated Nursing Process - Implementation: Client Needs - Safe Effective Care Environment, Management of Care: Cognitive Level - Analysis: Concept - Perfusion. Med-Surg success chapter 20 comprehensive final examination

56. Which assessment data indicate to the nurse the client has a conductive hearing loss? 1. The Rinne test results in air-conducted sound being louder than bone-conducted. 2. The client is unable to hear accurately when conducting the whisper test. 3. The Weber test results in the sound being heard better in the affected ear. 4. The tympanogram results in the ticking watch heard better in the unaffected ear.

1. The Rinne test result indicates a normal hearing; in conductive hearing loss, bone-conducted sound is heard as long or as longer than air - conducted sound 2. The whisper test is used to make a general estimation of hearing, but it is not used to specifically diagnose for conductive hearing loss. ✅3. The Weber test uses bone conduction to test lateralization of sound by placing a tuning fork in the middle of the skull or forehead. A normal test results in the client hearing the sound equally in both ears. 4. The tympanogram (impedance audiometry) measures middle-ear muscle reflex to sound stimulation and compliance of the tympanic membrane by changing air pressure in a sealed ear canal. It does not specifically support the diagnosis of conductive hearing loss. Content - Medical: Integrated Nursing Process - Assessment: Client Needs - Physiological Integrity, Reduction of Risk Potential: Cognitive Level - Analysis: Concept - Sensory Perception. Med-Surg success chapter 20 comprehensive final examination

72. The home health nurse must see all of the following clients. Which client should the nurse assess first? 1. The client who is postoperative from an open cholecystectomy who has green drainage coming from the T-tube. 2. The client diagnosed with congestive heart failure who complains of shortness of breath while fixing meals. 3. The client diagnosed with AIDS dementia whose family called and reported that the client is vomiting "coffee grounds stuff." 4. The client diagnosed with end-stage liver failure who has gained three (3) pounds and is not able to wear house shoes.

1. The T-tube is inserted into the common bile duct to drain bile until healing occurs, and bile is green, so this is expected. 2. The client with CHF would be expected to experience dyspnea on exertion. ✅3. Coffee-ground emesis indicates gastrointestinal bleeding, and this client should be seen first. 4. The client in end-stage liver failure is unable to assimilate protein from the diet, which leads to fluid volume retention and resulting weight gain. This is expected for this client. Content - Nursing Management: Integrated Nursing Process - Assessment: Client Needs - Safe Effective Care Environment, Management of Care: Cognitive Level - Analysis: Concept - Assessment. Med-Surg success chapter 20 comprehensive final examination

31. The client diagnosed with end-stage congestive heart failure is being cared for by the home health nurse. Which intervention should the nurse teach the caregiver? 1. Report any time the client starts having difficulty breathing. 2. Notify the HCP if the client gains more than 3 lb in a week. 3. Teach how to take the client's apical pulse for one (1) full minute. 4. Encourage the client to participate in 30 minutes of exercise a day.

1. The client diagnosed with CHF will be short of breath on exertion and with activity. The significant other should report difficulty breathing not subsiding with rest or stopping the activity. ✅2. Two (2) to three (3) pounds of weight gain reflects fluid retention as a result of heart failure, which warrants notifying the HCP. 3. The caregiver must not administer the digoxin if the radial pulse is less than 60 bpm. The apical pulse is more difficult to assess in a client than the radial pulse. 4The client in end-stage CHF is dying and should not exercise daily; activity intolerance as a result of decreased cardiac output is the number-one life-limiting problem Content - Medical: Integrated Nursing Process - Planning: Client Needs - Physiological Integrity, Physiological Adaptation: Cognitive Level - Synthesis: Concept - Nursing Roles. Med-Surg success chapter 20 comprehensive final examination

86. Which client should the nurse consider at risk for developing acute renal failure? 1. The client diagnosed with essential hypertension. 2. The client diagnosed with type 2 diabetes. 3. The client who had an anaphylactic reaction. 4. The client who had an autologous blood transfusion.

1. The client diagnosed with essential HTN is at risk for chronic renal failure. 2. The client diagnosed with diabetes type 2 is at risk for chronic renal failure. ✅3. Anaphylaxis leads to circulatory collapse, which decreases perfusion of the kidneys and can lead to acute renal failure. 4. This is a transfusion of the client's own blood, which should not cause a reaction. Content - Medical: Integrated Nursing Process - Assessment: Client Needs - Physiological Integrity, Reduction of Risk Potential: Cognitive Level - Analysis: Concept - Urinary Elimination Med-Surg success chapter 20 comprehensive final examination

63. The client diagnosed with cancer of the larynx has had a partial laryngectomy. Which client problem has the highest priority? 1. Impaired communication. 2. Ineffective coping. 3. Risk for aspiration. 4. Social isolation.

1. The client has a partial laryngectomy and the voice quality may change, but the client can still speak 2. This is a psychosocial problem, but it is not priority over a potential physiological problem. ✅3. As a result of the injury to the musculature of the throat area, this client is at high risk for aspirating. 4. This is a psychosocial problem, but it is not priority over a potential physiological problem Content - Surgical: Integrated Nursing Process - Diagnosis: Client Needs - Safe Effective Care Environment, Safety and Infection Control: Cognitive Level - Analysis: Concept -Oxygenation. Med-Surg success chapter 20 comprehensive final examination

98. Which interventions should the emergency department nurse implement for a client who has an AP of 122 and a BP of 80/50? Select all that apply. 1. Put the client in reverse Trendelenburg position. 2. Start an intravenous line with an 18-gauge catheter. 3. Have the client complete the admission process. 4. Cover the client with blankets and keep warm. 5. Request the laboratory draw a type and crossmatch.

1. The client would be placed in the Trendelen- burg position, which is with the head lower than the feet. ✅2..The client is in shock and may need blood transfusions; therefore, a large-bore cath- eter should be started to infuse fluids, plasma expanders, and possible blood. 3. The admission process cannot be completed by the client because the condition is life threatening. ✅4.The client will be cold as a result of vasoconstriction of the periphery result- ing from a low pulse and blood pressure. ✅5.The client will more than likely need blood transfusions that require a type and crossmatch. Med-Surg success chapter 20 comprehensive final examination

76. The client with chronic pancreatitis is admitted with an acute exacerbation of the disease. Which laboratory result warrants immediate intervention by the nurse? 1. The client's amylase is elevated. 2. The client's WBC count is WNL. 3. The client's blood glucose is elevated. 4. The client's lipase is within normal limits.

1. The client's amylase would be elevated in an acute exacerbation of pancreatitis. 2. The WBC count is not elevated in this disease process. ✅3. In clients with chronic pancreatitis, the beta cells of the pancreas are affected and, therefore, insulin production is affected. An elevated glucose level would warrant the nurse assessing the client. 4. Lipase is an enzyme that is excreted by the pancreas. Normal lipase levels indicate a normally functioning pancreas. Content - Medical: Integrated Nursing Process - Assessment: Client Needs - Physiological Integrity, Reduction of Risk Potential: Cognitive Level - Synthesis: Concept - Metabolism. Med-Surg success chapter 20 comprehensive final examination

35. The client diagnosed with arterial occlusive disease is prescribed an antiplatelet medication, clopidogrel (Plavix). Which assessment data indicate the medication is effective? 1. The client's pedal pulse is bounding. 2. The client's blood pressure has decreased. 3. The client does not exhibit signs of a stroke. 4. The client has decreased pain when ambulating.

1. The client's pedal pulse does not evaluate the effectiveness of this medication. 2. This medication is not administered to help decrease blood pressure. ✅3. This medication inhibits platelet aggre- gation and is considered effective when there is a decrease in atherosclerotic events, an example of which is a stroke. 4. This medication will not help the pain associ- ated with arterial occlusive disease. Content - Medical: Integrated Nursing Process - Evaluation: Client Needs - Physiological Integrity, Pharmacological and Parenteral Therapies: Cognitive Level - Synthesis: Concept - Medication. Med-Surg success chapter 20 comprehensive final examination

94. The student nurse accidentally punctured her finger with a contaminated needle. Which action should the student nurse take first? 1. Notify the infection control nurse. 2. Allow the puncture site to bleed. 3. Report to the emergency department. 4. Cleanse the site with Betadine.

1. The infection control nurse must be notified, but it is not the first action ✅ 2. Allowing the site to bleed allows any pathogen to bleed out; the student nurse should not apply pressure or attempt to stop the flow of blood. 3. This would be done to document the occurrence and start early prophylaxis if necessary, but it is not the first intervention. 4. This is an appropriate intervention once the wound is allowed to bleed; this is a needle stick, so the nursing student will not bleed to death Content - Nursing Management: Integrated Nursing Process - Implementation: Client Needs - Safe Effective Care Environment, Management of Care: Cognitive Level - Application: Concept - Management. Med-Surg success chapter 20 comprehensive final examination

61. The unlicensed assistive personnel (UAP) notifies the nurse the client diagnosed with chronic obstructive pulmonary disease is complaining of shortness of breath and would like his oxygen level increased. Which intervention should the nurse implement? 1. Notify the respiratory therapist (RT). 2. Ask the UAP to increase the oxygen. 3. Obtain a STAT pulse oximeter reading. 4. Tell the UAP to leave the oxygen alone.

1. The nurse can take care of this situation and does not need to notify the RT. 2. The UAP cannot increase oxygen. The nurse should treat oxygen as a medication. Also, increasing the oxygen level could cause the client to stop breathing as a result of carbon dioxide narcosis. 3. The pulse oximeter reading will be low because the client has COPD. ✅4. The oxygen level for a client with COPD must remain between two (2) and three (3) L/min because the client's stimulus for breathing is low blood oxygen levels. If the client receives increased oxygen, the stimulus for breathing will be removed and the client will stop breathing. Content - Medical: Integrated Nursing Process - Implementation: Client Needs - Safe Effective Care Environment, Management of Care: Cognitive Level - Application: Concept - Oxygenation Med-Surg success chapter 20 comprehensive final examination

9. The client in the long-term care facility has severe osteoarthritis. Which nursing task should the nurse delegate to the unlicensed assistive personnel (UAP)? 1. Feed the client the breakfast meal. 2. Give the client Maalox, an antacid. 3. Monitor the client's INR results. 4. Assist the client to the shower room.

1. The nurse should encourage the client to maintain independent functioning, and del- egating the UAP to feed the client would be encouraging dependence. Although this is an over-the-counter medica- tion, a UAP cannot administer any medica- tion to a client. The UAP cannot assess or evaluate any of the client's diagnostic information. ✅4. The UAP could assist the client to ambu- late to the shower room and assist with morning care. Content - Nursing Management: Integrated Nursing Process - Planning: Client Needs - Safe Effective Care Environment, Management of Care: Cognitive Level - Synthesis: Concept - Nursing Roles. Med-Surg success chapter 20 comprehensive final examination

13. The client has a fractured right tibia. Which assessment data warrant immediate intervention? 1. The client complains of right calf pain. 2. The nurse cannot palpate the radial pulse. 3. The client's right foot is cold to touch. 4. The nurse notes ecchymosis on the right leg.

1. The nurse would expect the client with a fractured right leg to have pain but it would not warrant immediate intervention. 2. The nurse would assess the client's pedal or posterior tibial pulse for a client with a frac- tured right tibia. ✅3. Any abnormal neurovascular assessment data, such as coldness, paralysis, or pares- thesia, warrant immediate intervention by the nurse. 4. Ecchymosis is bruising and would be ex- pected in the client who has a fractured tibia. Content - Medical: Integrated Nursing Process - Assessment: Client Needs - Physiological Integrity, Reduction of Risk Potential: Cognitive Level - Analysis: Concept - Mobility Med-Surg success chapter 20 comprehensive final examination

93. The elderly client from the long-term care facility is admitted into the hospital diagnosed with septicemia. Which area of the body is the most appropriate place for the nurse to assess the hydration status of the client? 1. Head 2. Abdomen 3. Palm 4. Legs

4. The client's thigh area is not the best place to assess for skin turgor 3. The client's hand has decreased subcutaneous tissue and has been exposed to the sun, which results in decreased tissue elasticity, so this is not the best place to assess for skin turgor. ✅2. The tissue on the chest is protected from sun exposure and has adequate subcuta- neous tissue to provide a more accurate assessment of hydration status. 1. The eyeball will lose its elasticity secondary to dehydration, but most people do not like the eyes being touched. Content - Medical: Integrated Nursing Process - Assessment: Client Needs - Physiological Integrity, Reduction of Risk Potential: Cognitive Level - Analysis: Concept - Assessment. Med-Surg success chapter 20 comprehensive final examination

Diseases Abbreviations and Symbols

ASCVD. arteriosclerotic cardiovascular disease ASHD. arteriosclerotic heart disease BPH. benign prostatic hypertrophy CA. cancer CAD. coronary artery disease CHF. congestive heart failure COPD. chronic obstructive pulmonary disease CVA. cerebrovascular accident DM. diabetes mellitus HTN (↑BP). hypertension MI. myocardial infarction PE. pulmonary emboli PVD. peripheral vascular disease STD. sexually transmitted disease STI. sexually transmitted infection URI. upper respiratory infection

73. Which data indicate to the nurse the client with end-stage liver failure is improving? 1. The client has a tympanic wave. 2. The client is able to perform asterixis. 3. The client is confused and lethargic. 4. The client's abdominal girth has decreased.

Content - Medical: Integrated Nursing Process - Assessment: Client Needs - Physiological Integrity, Reduction of Risk Potential: Cognitive Level - Analysis: Concept - Digestion. Med-Surg success chapter 20 comprehensive final examination

83. Which signs/symptoms should the nurse expect to assess in the client diagnosed with Addison's disease? 1. Hypotension and bronze skin pigmentation. 2. Water retention and osteoporosis. 3. Hirsutism and abdominal striae. 4. Truncal obesity and thin, wasted extremities.

Content - Medical: Integrated Nursing Process - Assessment: Client Needs - Physiological Integrity, Reduction of Risk Potential: Cognitive Level - Analysis: Concept - Metabolism Med-Surg success chapter 20 comprehensive final examination

77. Theclienthadabdominalsurgeryandisreceiving bag #5 of total parenteral nutrition (TPN) via a subclavian line infusing at 126 mL/hr. The nurse realizes bag #6 is not on the unit and TPN bag #5 has 50 mL left to infuse. Which intervention should the nurse implement? 1. Decrease the rate of bag #5 to a keep-open rate. 2. Prepare to hang a 1,000-mL bag of normal saline. 3. When bag #5 is empty, convert to a heparin lock. 4. Infuse D10W at 126 mL/hr via the subclavian line.

Content - Medical: Integrated Nursing Process - Implementation: Client Needs - Physiological Integrity, Pharmacological and Parenteral Therapies: Cognitive Level - Application: Concept - Perioperative. Med-Surg success chapter 20 comprehensive final examination

81. The client with type 1 diabetes asks the nurse, "What causes me to get dehydrated when my glucose level is elevated?" Which statement would be the nurse's best response? 1. "The kidneys are damaged and cannot filter out the urine." 2. "The glucose causes fluid to be pulled from the tissues." 3. "The sweating as a result of the high glucose level causes dehydration." 4. "You get dehydrated with a high glucose because you are so thirsty."

Content - Medical: Integrated Nursing Process - Implementation: Client Needs - Physiological Integrity, Physiological Adaptation: Cognitive Level - Application: Concept - Fluid and Electrolyte Balance Med-Surg success chapter 20 comprehensive final examination

80. The client with type 2 diabetes mellitus asks the nurse, "What does it matter if my glucose level is high? I don't feel bad." Which statement by the nurse is most appropriate? 1. "The high glucose level can damage your eyes and kidneys over time." 2. "The glucose level causes microvascular and macrovascular problems." 3. "As long as you don't feel bad, everything will probably be all right." 4. "A high blood glucose level will cause you to get metabolic acidosis."

Content - Medical: Integrated Nursing Process - Implementation: Client Needs - Physiological Integrity, Physiological Adaptation: Cognitive Level - Application: Concept - Metabolism. Med-Surg success chapter 20 comprehensive final examination

84. . The client diagnosed with neurogenic diabetes insipidus (DI) asks the nurse, "What is wrong with me? Why do I urinate so much?" Which statement by the nurse is most appropriate? 1. "The islet cells in your pancreas are not functioning properly." 2. "Your pituitary gland is not secreting a necessary hormone." 3. "Your kidneys are in failure and you are overproducing urine." 4. "The thyroid gland is speeding up all your metabolism."

Content - Medical: Integrated Nursing Process - Implementation: Client Needs - Physiological Integrity, Physiological Adaptation: Cognitive Level - Application: Concept - Metabolism. Med-Surg success chapter 20 comprehensive final examination

70. The client diagnosed with peptic ulcer disease is being discharged. Which nursing task can be delegated to a trained unlicensed assistive personnel (UAP)? 1. Complete the discharge instructions sheet. 2. Remove the client's saline lock. 3. Clean the client's room after discharge. 4. Check the client's hemoglobin and hematocrit.

Content - Medical: Integrated Nursing Process - Planning: Client Needs - Safe Effective Care Environment, Management of Care: Cognitive Level - Synthesis: Concept - Nursing Roles. Med-Surg success chapter 20 comprehensive final examination

75. The public health nurse is discussing hepatitis with a client who is traveling to a third world country in one (1) month. Which recommendation should the nurse discuss with the client? 1. A gamma globulin injection. 2. A hepatitis A vaccination. 3. A PPD skin test on the left arm. 4. A hepatitis B vaccination.

Content - Medical: Integrated Nursing Process - Planning: Client Needs - Safe Effective Care Environment, Safety and Infection Control: Cognitive Level - Synthesis: Concept - Safety. Med-Surg success chapter 20 comprehensive final examination

74. The nurse is discussing funeral arrangements with the family of a deceased client whose organs and tissues are being donated today. Which information should the nurse discuss with family? 1. The family can request an open casket funeral. 2. Your loved one must wear a long-sleeved shirt. 3. You might want to have a private viewing only. 4. This will not delay the timing of the funeral.

Content - Nursing Management: Integrated Nursing Process - Planning: Client Needs - Safe Effective Care Environment, Management of Care: Cognitive Level - Synthesis: Concept - Grief and Loss Med-Surg success chapter 20 comprehensive final examination

Diagnostic Studies Abbreviations and Symbols

Diagnostic Studies. ABG. arterial blood gases BE. barium enema CBC. complete blood count CO2. carbon dioxide C&S. culture and sensitivity CXR. chest x-ray ECG (EKG). cardiogram lytes. electrolytes RBC. red blood cells UA. urinalysis UGI. upper GI WBC. white blood cells

Delivering Optimal Care to Patients and Their Families

Nursing leadership at MedStar Health is committed to leading the organization along a path to a clearly defined goal: the delivery of optimal care to patients and their families. To guide these efforts, the MedStar Way, a set of activities, has been devised to motivate change in contemporary nursing practice and nursing care delivery. Senior Leader Rounding—to promote communication and access within the nursing team Multidisciplinary Team Rounding—to ensure consistent documentation and timely communication, and discuss the care plan Nurse Leader Rounding—to support a culture of service, assess patient and staff needs, and promote the patient/family experience Nursing Staff Hourly Rounds—to anticipate and evaluate patient needs, and improve safety and satisfaction Nurse-to-Nurse Bedside Shift Reports—to ensure safe handoffs, and keep the patient and family involved in the care plan Huddles at the Beginning of Each Shift—to improve teamwork and communication, and ensure timely response to issues and concerns White Boards in Patient Rooms—to encourage communication, teamwork, and efficiency, and to assure the patient that the team is working together to provide optimal care Post-Visit Phone Calls—to ensure quality clinical outcomes and increase patient satisfaction

1.

✅ CHAPTER 17 Implementing

10.

✅ CHAPTER 17 Implementing

2.

✅ CHAPTER 17 Implementing

3.

✅ CHAPTER 17 Implementing

4.

✅ CHAPTER 17 Implementing

5.

✅ CHAPTER 17 Implementing

6.

✅ CHAPTER 17 Implementing

7.

✅ CHAPTER 17 Implementing

8.

✅ CHAPTER 17 Implementing

9.

✅ CHAPTER 17 Implementing

60. The 85-year-old client diagnosed with severe end-stage chronic obstructive pulmonary disease has a chest x-ray incidentally revealing an eight (8)-cm abdominal aortic aneurysm (AAA). Which intervention should the nurse implement? 1. Discuss possible end-of-life care issues. 2. Prepare the client for abdominal surgery. 3. Teach the client how to do pursed-lip breathing. 4. Talk with the family about the client's condition.

✅ 1. The client with end-stage COPD would not be a candidate for an AAA repair, although the size of the aneurysm places the client at risk for rupture. Although many nurses do not like to address end- of-life issues, this would be an important and timely intervention. 2. The client is not a surgical candidate because of the comorbid condition and age 3. The client should know how to pursed-lip breathe at this point in the disease process. 4. Although the client is 85 years old, the nurse should discuss all health-care issues with the client and not the family. This is a violation of HIPAA. Content - Medical: Integrated Nursing Process - Implementation: Client Needs - Safe Effective Care Environment, Management of Care: Cognitive Level - Application: Concept - Oxygenation. Med-Surg success chapter 20 comprehensive final examination

87. The client diagnosed with chronic renal failure is receiving peritoneal dialysis. Which assessment by the nurse warrants immediate intervention? 1. The dialysate return is cloudy. 2. There is a greater dialysate return than input. 3. The client complains of abdominal fullness. 4. The client voided 50 mL during the day.

✅. 1. A cloudy dialysate indicates an infection and must be reported immediately to prevent peritonitis. 2. The dialysate should be greater than the in- take so fluid is being removed from the body 3. After infusing 1,000 mL of dialysate, abdomi- nal fullness is not unexpected. 4. The client voiding any amount does not warrant immediate intervention. Content - Medical: Integrated Nursing Process - Assessment: Client Needs - Physiological Integrity, Reduction of Risk Potential: Cognitive Level - Synthesis: Concept - Urinary Elimination. Med-Surg success chapter 20 comprehensive final examination

92. The client asks the nurse, "What are the risk factors for developing multiple sclerosis?" Which statement is a risk factor for multiple sclerosis (MS)? 1. A close relative with MS may indicate a risk for MS. 2. Living in the southern United States predisposes a person to MS. 3. Use of tobacco product is the number-one risk for developing MS. 4. A sedentary lifestyle can cause a person to develop MS.

✅1. A close relation (parent or sibling) who has MS may indicate a risk for the client also to develop MS. Other common risk factors are age, race, gender, environment, immune factors, and smoking. 2.There is a higher incidence of MS in people who live in the northeastern United States and Canada, but there is no known reason for this occurrence. 3. Tobacco use is a risk factor but not the primary risk factor for MS. 4. A sedentary lifestyle does not predispose a person to develop MS. Content - Medical: Integrated Nursing Process - Assessment: Client Needs - Health Promotion and Maintenance: Cognitive Level - Analysis: Concept - Mobility. Med-Surg success chapter 20 comprehensive final examination

99.The client is eight (8) hours postoperative small bowel resection. Which data indicate the client has had a complication from the surgery? 1. A hard, rigid, boardlike abdomen. 2. High-pitched tinkling bowel sounds. 3. Absent bowel sounds. 4. Complaints of pain at "6" on the pain scale.

✅1. A hard, rigid, boardlike abdomen is the hallmark sign of peritonitis, which is a life-threatening complication of abdomi- nal surgery. 2. This occurs when the client has a nasogastric tube connected to suction and has minimal peristalsis and is not a complication of the surgery. 3. The client has had general anesthesia for this surgery, and absent bowel sounds at eight (8) hours postoperative does not indi- cate a complication. 4. The client with this type of surgery is expected to have pain at a "6" or higher on a 1-to-10 scale; this is not considered a complication. Content - Surgical: Integrated Nursing Process - Assessment: Client Needs - Physiological Integrity, Reduction of Risk Potential: Cognitive Level - Analysis: Concept - Perioperative Med-Surg success chapter 20 comprehensive final examination

2. Which problem is priority for the 24-year-old client diagnosed with endometriosis who is admitted to the gynecological unit? 1. Hemorrhage. 2. Pain. 3. Constipation. 4. Dyspareunia

✅1. Anemia caused by endometriosis occurs over time and is not an acute complication such as hemorrhaging. 2. Pain is the primary complaint of the client; the pain occurs as a result of ectopic tissue bleeding into the abdominal cavity during menses. 3. Endometriosis does not cause constipation, and this would not be a priority problem. The client may experience pain during a bowel movement. 4. Dyspareunia is pain during intercourse, and this client is in the hospital (and unlikely to be having sex there). Med-Surg success chapter 20 comprehensive final examination

65. The nurse is assessing the client with a pneumothorax who has a closed-chest drainage system. Which data indicate the client's condition is stable? 1. There is fluctuation in the water-seal compartment. 2. There is blood in the drainage compartment. 3. The trachea deviates slightly to the left. 4. There is bubbling in the suction compartment.

✅1. Fluctuation in the water-seal compart- ment with respirations indicates the system is working properly and the client is stable. 2. Blood in the drainage compartment indicates there is a problem because the client is diag- nosed with a pneumothorax and there should not be any bleeding. 3.Any deviation of the trachea indicates a tension pneumothorax, a potentially life-threatening complication. 4. Bubbling in the suction compartment does not indicate a stable or unstable client. Content - Medical: Integrated Nursing Process - Assessment: Client Needs - Physiological Integrity, Reduction of Risk Potential: Cognitive Level - Analysis: Concept - Oxygenation. Med-Surg success chapter 20 comprehensive final examination

69. Which information should the nurse discuss with the client to prevent an acute exacerbation of diverticulosis? 1. Increase the fiber in the diet. 2. Drink at least 1,000 mL of water a day. 3. Encourage sedentary activities. 4. Take cathartic laxatives daily.

✅1. Increasing fiber will help prevent con- stipation, the number-one reason for an acute exacerbation of diverticulosis, which results in diverticulitis. 2. The client should increase fluid intake to pre- vent constipation to at least 2,500 mL/day. 3. The client should exercise daily to prevent constipation. 4. The client should take bulk-forming laxatives, which helps prevent constipation by adding bulk to the stool. Cathartic laxatives are harsh colonic stimulants and should not be taken on a daily basis. Content - Medical: Integrated Nursing Process - Planning: Client Needs - Physiological Integrity, Physiological Adaptation: Cognitive Level - Synthesis: Concept - Bowel Elimination Med-Surg success chapter 20 comprehensive final examination

47. Which medical client problem should the nurse include in the plan of care for a client diagnosed with cardiomyopathy? 1. Heart failure. 2. Activity intolerance. 3. Paralytic ileus. 4. Atelectasis.

✅1. Medical client problems indicate the nurse and the HCP must collaborate to care for the client; the client must have medications for heart failure. 2. Without an HCP's order, the nurse can in- struct the client to pace activities and teach about rest versus activity. 3. Paralytic ileus is a medical problem but would not be expected in a client with cardiomyopathy. 4. Atelectasis occurs when airways collapse, which would not occur in a client with cardiomyopathy. Content - Medical: Integrated Nursing Process - Diagnosis: Client Needs - Physiological Integrity, Physiological Adaptation: Cognitive Level - Analysis: Concept - Perfusion. Med-Surg success chapter 20 comprehensive final examination

53. Which comment by the client diagnosed with rule-out Guillain-Barré (GB) syndrome is most significant when completing the admission interview? 1. "I had a bad case of gastroenteritis a few weeks ago." 2. "I never use sunblock and I use a tanning bed often." 3. "I started smoking cigarettes about 20 years ago." 4. "I was out of the United States for the last 2 months."

✅1. The cause of GB syndrome is unknown, but a precipitating event usually occurs one (1) to three (3) weeks prior to the onset. The precipitating event may be a respiratory or gastrointestinal viral or bacterial infection. 2. These are not precipitating events or risk factors for developing GB syndrome. 3. Smoking is not a risk factor for developing GB syndrome. 4. GB syndrome is not more prominent in foreign countries than in the United States. Content - Medical: Integrated Nursing Process - Assessment: Client Needs - Physiological Integrity, Reduction of Risk Potential: Cognitive Level - Analysis: Concept - Immunity. Med-Surg success chapter 20 comprehensive final examination

57. The client reports a twisting motion of the knee during a basketball game. The client is scheduled for arthroscopic surgery to repair the injury. Which information should the nurse teach the client about postoperative care? 1. The client should begin strengthening the surgical leg. 2. The client should take pain medication routinely. 3. The client should remain on bedrest for two (2) weeks. 4. The client should return to the doctor in six (6) months.

✅1. The client should begin exercises that will strengthen the surgical leg as soon as the surgery is completed. 2. Pain medication should be taken as needed, not routinely. 3. The client may ambulate with the restrictions ordered by the surgeon. 4.The client will return to see the surgeon prior to six (6) months. The surgeon will need to monitor for healing and complications. Content - Surgical: Integrated Nursing Process - Planning: Client Needs - Physiological Integrity, Reduction of Risk Potential: Cognitive Level - Synthesis: Concept - Inflammation. Med-Surg success chapter 20 comprehensive final examination

45. The nurse is at home preparing for the 7 a.m. to 7 p.m. shift and has the flu with a temperature of 100.4 ̊F. Which action should the nurse take? 1. Notify the hospital the nurse will not be coming into work. 2. Go to work and wear an isolation mask when caring for the clients. 3. Request an alternative assignment not involving direct client care. 4. Take over-the-counter cold medication and report to work on time.

✅1. The nurse should stay at home because the nurse will expose all other personnel and clients to the illness. Flu, especially with a fever, places the nurse at risk for a secondary pneumonia. 2. The nurse is ill, and many errors are made when the nurse is not functioning at 100%. 3. Even if the nurse doesn't have direct client care, the nurse will expose other employees to the virus. 4. OTC medications will not prevent the trans- mission of flu to others, nor will they pre- vent the nurse from developing a secondary pneumonia. Content - Nursing Management: Integrated Nursing Process - Implementation: Client Needs - Safe Effective Care Environment, Safety and Infection Control: Cognitive Level - Application: Concept - Nursing Roles. Med-Surg success chapter 20 comprehensive final examination

66. The client is admitted to the intensive care unit diagnosed with rule-out adult respiratory distress syndrome (ARDS). The client is receiving 10 L/min of oxygen via nasal cannula. Which arterial blood gases indicate the client does not have ARDS? 1. pH 7.38, Pao2 82, Paco2 45, HCO3 26. 2. pH 7.35, Pao2 74, Paco2 43, HCO3 24. 3. pH 7.45, Pao2 60, Paco2 45, HCO3 28. 4. pH 7.32, Pao2 50, Paco2 55, HCO3 28.

✅1. These are normal ABGs, which would not be expected if the client has ARDS. This client has an oxygen level below 80 to 100; therefore, this client may be developing early ARDS. This is respiratory acidosis, which would be expected in a client with ARDS. These are the expected ABGs of a client with ARDS. There is a low oxygen level despite high oxygen administration. Content - Medical: Integrated Nursing Process - Assessment: Client Needs - Physiological Integrity, Reduction of Risk Potential: Cognitive Level - Analysis: Concept - Oxygenation. Med-Surg success chapter 20 comprehensive final examination

30. The nurse is teaching the client in a cardiac rehabilitation unit. Which dietary information should the nurse discuss with the client? 1. No more than 30% of daily food intake should be fats. 2. Eighty percent of calories should come from carbohydrates. 3. Red meat should comprise at least 50% of daily intake. 4. Monounsaturated fat in the daily diet should be increased.

✅1. This is a correct statement. The recom- mended proportions of food are 50% carbohydrates, 30% or less from fat, and 20% protein. 2. Only 50% of the calories should come from carbohydrates. 3. Red meat is an excellent source of protein but should only comprise 20% of the diet, and red meat is very high in fat. 4.Polyunsaturated fats, not the monounsatu- rated fats, are the better fats. Content - Medical: Integrated Nursing Process - Planning: Client Needs - Physiological Integrity, Basic Care and Comfort: Cognitive Level - Synthesis: Concept - Nutrition. Med-Surg success chapter 20 comprehensive final examination

91. Which assessment data would make the nurse suspect the client has cancer of the bladder? 1. Gross painless hematuria. 2. Burning on urination. 3. Terminal dribbling. 4. Difficulty initiating the stream.

✅1. This is the most common presenting symptom of bladder cancer. 2.Burning on urination is a symptom of a urinary tract infection. 3. Terminal dribbling is a symptom of benign prostatic hypertrophy. 4.Difficulty initiating a urine stream is a symptom of benign prostatic hypertrophy or neurogenic bladder. Content - Medical: Integrated Nursing Process - Assessment: Client Needs - Physiological Integrity, Reduction of Risk Potential: Cognitive Level - Analysis: Concept - Cellular Regulation. Med-Surg success chapter 20 comprehensive final examination

21. The client is newly diagnosed with epilepsy. Which statement indicates the client needs clarification of the discharge teaching? 1. "I can drive as soon as I see my HCP for my follow-up visit." 2. "I should get at least eight (8) hours of sleep at night." 3. "I should take my medication every day even if I am sick." 4. "I will take showers instead of taking tub baths."

✅1. This statement indicates the client does not understand the discharge teaching. The client will not be able to drive until the client is seizure free for a certain pe- riod of time. The laws in each state differ. 2. Lack of sleep is a risk factor for having seizures. 3.Noncompliance with medication is a risk factor for having a seizure. 4. If the client has a seizure in the bathtub, the client could drown Content - Medical: Integrated Nursing Process - Evaluation: Client Needs - Physiological Integrity, Physiological Adaptation: Cognitive Level - Synthesis: Concept - Neurologic Regulation. Med-Surg success chapter 20 comprehensive final examination

3. The 28-year-old client diagnosed with testicular cancer is scheduled for a unilateral orchiectomy. Which intervention should have priority in the client's plan of care? 1. Encourage the client to bank his sperm. 2. Discuss completing an advance directive. 3. Explain follow-up chemotherapy and radiation. 4. Allow the client to express his feelings regarding having cancer.

✅1. With a remaining testicle, the client will be able to maintain sexual potency, but radia- tion and chemotherapy may cause the cli- ent to become sterile. Therefore, banking his sperm will allow him to father a child later in life. 2. Testicular cancer has a 90% cure rate with standard therapy; therefore, completing an advance directive is not priority. 3. The client will not be undergoing chemo- therapy for at least six (6) weeks to allow the client to heal; therefore, this is not a priority intervention. 4. This is important, but when preparing the client for surgery, the priority intervention is to accomplish presurgical interventions. Content - Medical: Integrated Nursing Process - Implementation: Client Needs - Safe Effective Care Environment, Management of Care: Cognitive Level - Synthesis: Concept - Male Reproduction. Med-Surg success chapter 20 comprehensive final examination

42. The client diagnosed with stomach cancer has developed disseminated intravascular coagulopathy (DIC). Which collaborative intervention should the nurse expect to implement? 1. Prepare to administer intravenous heparin. 2. Assess for frank hemorrhage from venipuncture sites . 3. Monitor for decreased level of consciousness. 4. Prepare to administer total parenteral nutrition.

✅1.Heparin interferes with the clotting cas- cade and may prevent further clotting factor consumption resulting from uncon- trolled thromboses formation. 2. Assessment is an independent intervention; it is not collaborative and does not require an HCP's order. 3.Assessment is an independent intervention; it is not collaborative and does not require an HCP's order. 4.TPN is not a treatment for a client with DIC. Content - Medical: Integrated Nursing Process - Implementation: Client Needs - Safe Effective Care Environment, Management of Care: Cognitive Level - Application: Concept - Clotting. Med-Surg success chapter 20 comprehensive final examination

52. The nurse is assessing the client with psoriasis. Which data support this diagnosis? 1. Appearance of red, elevated plaques with silvery white scales. 2. A burning, prickling row of vesicles located along the torso. 3. Raised, flesh-colored papules with a rough surface area. 4. An overgrowth of tissue with an excessive amount of collagen.

✅1.Most clients with psoriasis have red, raised plaques with silvery white scales 2. A burning, prickling row of vesicles located along the torso is the description of herpes zoster. 3. A raised, flesh-colored papule with a rough surface area is a description of a wart. 4. An overgrowth of tissue with an excessive amount of collagen is the definition of keloids. Content - Medical: Integrated Nursing Process - Assessment: Client Needs - Physiological Integrity, Reduction of Risk Potential: Cognitive Level - Analysis: Concept - Skin Integrity Med-Surg success chapter 20 comprehensive final examination

51. The nurse is initiating a blood transfusion. Which interventions should the nurse implement? Select all that apply. 1. Assess the client's lung fields. 2. Have the client sign a consent form. 3. Start an IV with a 22-gauge IV catheter. 4. Hang 250 mL of D5W at a keep-open rate. 5. Check the chart for the HCP's order.

✅1.The nurse must make a decision on the amount of blood to infuse per hour. If the client is showing any sign of heart or lung compromise, the nurse would infuse the blood at the slowest possible rate. ✅2. Blood products require the client to give specific consent to receive blood. 3. The IV should be started with an 18-gauge catheter if possible; the smallest possible catheter is a 20-gauge. Smaller gauge catheters break down the blood cells. 4. Blood is not compatible with D5W; the nurse should hang 0.9% normal saline (NS) to keep open. ✅5. The nurse should verify the HCP's order before having the client sign the consent form. Content - Medical: Integrated Nursing Process - Implementation: Client Needs - Safe Effective Care Environment, Safety and Infection Control: Cognitive Level - Application: Concept - Assessment.

96. The client is admitted to the medical unit complaining of severe abdominal pain. Which intervention should the nurse implement first? 1. Assess for complications. 2. Medicate for pain. 3. Turn the television on. 4. Teach relaxation techniques.

✅1.The nurse must rule out any complication requiring immediate intervention before masking the pain with medication. Pain indicates a problem in some instances; pain is expected after surgery, but complications should always be ruled out. 2.The nurse should not medicate for pain until ruling out complications. 3. The television provides distraction, but it is not the first intervention. Assessment is the first intervention. 4. Teaching relaxation techniques will help the client's pain, but the first intervention must be assessment to rule out any complication. Content - Medical: Integrated Nursing Process - Implementation: Client Needs - Safe Effective Care Environment, Management of Care: Cognitive Level - Analysis: Concept - Assessment Med-Surg success chapter 20 comprehensive final examination

6. Which diagnostic procedure does the nurse anticipate being ordered for the 27-year-old female client who is reporting irregular menses and complaining of lower left abdominal pain during menses? 1. Pelvic sonogram. 2. Complete blood count (CBC). 3. Kidney, ureter, bladder (KUB) x-ray. 4. Computed tomography (CT) of abdomen

✅1.The pelvic sonogram, which visualizes the ovary using sound waves, is a diagnostic test for an ovarian cyst, which would be suspected with the client's signs/symptoms. 2. A CBC may be ordered to rule out appendi- citis, but this client does not have right lower abdominal pain. A KUB x-ray is ordered for a client with possible kidney stones. A CT of the abdomen would not visualize con- tents in the pelvis. Content - Medical: Integrated Nursing Process - Planning: Client Needs - Physiological Integrity, Reduction of Risk Potential: Cognitive Level - Synthesis: Concept - Fe- male Reproductive. Med-Surg success chapter 20 comprehensive final examination

43. The nurse is administering 250 mL of packed red blood cells with 50 mL of preservative. The client has no jugular vein distention and has clear breath sounds. After the first 15 minutes, at what rate should the nurse set the IV infusion pump? _______

✅150 mL/hr. The nurse should infuse the blood in two (2) hours because the client does not have signs/ symptoms of fluid volume overload. Content - Medical: Integrated Nursing Process - Implementation: Client Needs - Safe Effective Care Environment, Safety and Infection Control: Cognitive Level - Application: Concept - Medication. Med-Surg success chapter 20 comprehensive final examination

55. The client diagnosed with gastroesophageal reflux disease (GERD) has undergone surgery for a hiatal hernia repair. The client has a nasogastric tube in place. Intravenous fluid replacement is to be at 125 mL/hr plus the amount of drainage. The drainage from 0800 to 0900 is 45 mL. At which rate should the IV pump be set for the next hour? ______

✅170 mL/hr. 125 mL + 45 mL = 170 mL The IV pump should be set at this rate. Content - Surgical: Integrated Nursing Process - Implementation: Client Needs - Physiological Integrity, Pharmacological and Parenteral Therapies: Cognitive Level - Application: Concept - Digestion. Med-Surg success chapter 20 comprehensive final examination

67. The client has gastroesophageal reflux disease. Which HCP order should the nurse question? 1. Elevate the head of the client's bed with blocks. 2. Administer pantoprazole (Protonix) four (4) times a day. 3. A regular diet with no citrus or spicy foods. 4. Activity as tolerated and sit up in a chair for all meals.

✅2. Proton pump inhibitors are only admin- istered once or twice a day; they should not be given four (4) times a day because the medication decreases gastric acidity and the stomach needs some gastric acid to digest foods. The nurse would question this order. 3. The client is not prescribed any special diet; limiting spicy and citrus foods decreases acid in the stomach. 4. Sitting upright after all meals decreases the reflux of stomach contents into the esophagus. Content - Medical: Integrated Nursing Process - Implementation: Client Needs - Safe Effective Care Environment, Safety and Infection Control: Cognitive Level - Analysis: Concept - Digestion. Med-Surg success chapter 20 comprehensive final examination

79. Which assessment data indicate to the nurse the client with diarrhea is experiencing a complication? 1. Moist buccal mucosa. 2. A 3.6-mEq/L potassium level. 3. Tented tissue turgor. 4. Hyperactive bowel sounds.

✅3. Tented tissue turgor indicates dehydration, which is a complication of diarrhea. 4. Hyperactive bowel sounds would be expected in a client who has diarrhea. Content - Medical: Integrated Nursing Process - Assessment: Client Needs - Physiological Integrity, Reduction of Risk Potential: Cognitive Level - Analysis: Concept - Bowel Elimination. Med-Surg success chapter 20 comprehensive final examination

68. The client is diagnosed with an acute exacerbation of Crohn's disease. Which assessment data warrant immediate attention? 1. The client's WBC count is 10 (× 103)/mm3. 2. The client's serum amylase is 100 units/dL. 3. The client's potassium level is 3.3 mEq/L. 4. The client's blood glucose is 148 mg/dL.

✅3. This potassium level is low as a result of excessive diarrhea and puts the client at risk for cardiac dysrhythmias. Therefore, these assessment data warrant immediate intervention. 4. The client's blood glucose level is elevated, but it would not warrant immediate intervention for a client with Crohn's disease who has hypokalemia. Content - Medical: Integrated Nursing Process - Assessment: Client Needs - Physiological Integrity, Reduction of Risk Potential: Cognitive Level - Synthesis: Concept - Bowel Elimination. Med-Surg success chapter 20 comprehensive final examination

15. The client has sustained severe burns on both the anterior right and left leg and the anterior chest and abdomen. According to the rule of nines, what percentage of the body has been burned? _____ Head 8% Chest 18% Arms 13% Genitalia 1% Legs 18%

✅36%. Each leg is 18%, with the anterior surface (front) being 9%. Because the anterior of both legs is burned (9% each), that would be 18%. That 18% plus the anterior surface of the trunk, which is 18%, totals 36% of the total body surface burned. Content - Medical: Integrated Nursing Process - Assessment: Client Needs - Physiological Integrity, Reduction of Risk Potential: Cognitive Level - Analysis: Concept - Skin Integrity. Med-Surg success chapter 20 comprehensive final examination

82. The client calls the clinic first thing in the morning and tells the nurse, "I have been vomiting and having diarrhea since last night." Which response is appropriate for the nurse to make? 1. Encourage the client to eat dairy products. 2. Have the client go to the emergency department. 3. Request the client obtain a stool specimen. 4. Tell the client to stay on a clear liquid diet.

✅4. A clear liquid diet is recommended be- cause it maintains hydration without stim- ulating the gastrointestinal tract; diarrhea/ vomiting lasting longer than 24 hours, along with dehydration and weakness, would warrant the client being evaluated. Content - Medical: Integrated Nursing Process - Implementation: Client Needs - Physiological Integrity, Physiological Adaptation: Cognitive Level - Application: Concept - Fluid and Electrolyte Balance. Med-Surg success chapter 20 comprehensive final examination

101. According to the nursing process, which interventions should the nurse implement when caring for a client diagnosed with a right-sided cerebrovascular accident (stroke) and who has difficulty swallowing? List the interventions in order of the nursing process. 1. Write the client problem of "altered tissue perfusion." 2. Assess the client's level of consciousness and speech. 3. Request dietary to send a full liquid tray with Thick-It. 4. Instruct the UAP to elevate the head of the bed 30 degrees. 5. Note the amount of food consumed on the dinner tray.

✅: 2, 1, 3, 4, 5. 2. This is the assessment step, the first step of the nursing process. 1. Diagnosis is the second step in the nurs- ing process. In this case, it is "altered tissue perfusion." 3. Planning is the third step of the nursing process. 4. Implementation is the fourth step in the nursing process. 5. Evaluation is the last step of the nursing process. Content - Medical: Integrated Nursing Process - Implementation: Client Needs - Safe Effective Care Environment, Management of Care: Cognitive Level - - Med-Surg success chapter 20 comprehensive final examination

2.After instituting a new system for recording patient data, a nurse evaluates the "usability" of the system. Which actions by the nurse BEST reflect this goal? Select all that apply. A. The nurse checks that the screens are formatted to allow for ease of data entry. B. The nurse reorders the screen sequencing to maximize effective use of the system. C. The nurse ensures that the computers can be used by specified users effectively. D. The nurse checks that the system is intuitive, and supportive of nurses. E. The nurse improves end-user skills and satisfaction with the new system. F. The nurse ensures patient data is able to be shared across health care systems

✅a, c, d. Usability refers to the extent to which a product can be used by specified users to achieve specified goals with effectiveness, efficiency, and satisfaction in a specified context of use. Checking that screens are formatted to allow ease of data entry, ensuring that computers can be used by specified users effectively, and checking that the system is intuitive and supportive of nurses are all tasks related to the "usability" of the system. Reordering screen sequencing to maximize use and improving end-user skills and satisfaction with the new system refers to optimization. The ability to share patient data across health care systems is termed interoperability. CHAPTER 20 Nursing Informatics

8. A nurse is attempting to improve care on the pediatric ward of a hospital. Which nursing improvements might the nurse employ when following the recommendations of the Institute of Medicine's Committee on Quality of Health Care in America? Select all that apply. A. Basing patient care on continuous healing relationships B. Customizing care to reflect the competencies of the staff C. Using evidence-based decision making D. Having a charge nurse as the source of control E. Using safety as a system priority F. Recognizing the need for secrecy to protect patient privacy

✅a, c, e. Care should be based on continuous healing relationships and evidence-based decision making. Customization should be based on patient needs and values with the patient as the source of control. Safety should be used as a system priority, and the need for transparency should be recognized. CHAPTER 18 Evaluating

3. A new nurse who is being oriented to the subacute care unit is expected to follow existing standards when providing patient care. Which nursing actions are examples of these standards? Select all that apply. A. Monitoring patient status every hour B. Using intuition to troubleshoot patient problems C. Turning a patient on bed rest every 2 hours D. Becoming a nurse mentor to a student nurse E. Administering pain medication ordered by the physician F. Becoming involved in community nursing events

✅a, c, e. Standards are the levels of performance accepted and expected by the nursing staff or other health care team members. They are established by authority, custom, or consent. Standards would include monitoring patient status every hour, turning a patient on bed rest every 2 hours, and administering pain medication ordered by the physician. Using intuition to troubleshoot patient problems, becoming a nurse mentor to a student nurse, and becoming involved in community nursing events are not patient care standards. CHAPTER 18 Evaluating

8. A nurse is using the steps in informatics evaluation to evaluate the use of a portal as a patient resource. What are examples of activities that might occur in the "determining the question" step? Select all that apply. A. The nurse develops a clear, focused question to determine the data to be collected. B. The nurse determines what to evaluate. C. The nurse determines how the data ultimately should be reported. D. The nurse decides what specific data elements need to be collected. E. The nurse clarifies exactly how the data will be collected. F. The nurse performs comprehensive documentation of the data collected.

✅a, c. The nurse develops a clear, focused question to determine the data to be collected and the nurse determines how the data ultimately should be reported during the "determine the question" step. The nurse determines what to evaluate during the step "determine what will be evaluated." The nurse decides what specific data elements need to be collected during the "determine the needed data" step. The nurse clarifies exactly how the data will be collected during the "determine the data collection method and sample size" step. The nurse performs comprehensive documentation of the data collected during the "document your outcome evaluation" step. CHAPTER 20 Nursing Informatics

6. A nurse writes the following outcome for a patient who is trying to lose weight: "The patient can explain the relationship between weight loss, increased exercise, and decreased calorie intake." This is an example of what type of outcome? A. Cognitive B. Psychomotor C. Affective D. Physical changes

✅a. Cognitive outcomes involve increases in patient knowledge; psychomotor outcomes describe the patient's achievement of new skills; affective outcomes pertain to changes in patient values, beliefs, and attitudes; and physical changes are actual bodily changes in the patient (e.g., weight loss, increased muscle tone). CHAPTER 18 Evaluating

4. Population health addresses the health status and health issues of aggregate populations and addresses ways in which resources may be allocated to address these concerns. What is the driving force behind the use by health corporations of analytics and big data to support population health? A. The transition from fee-for-service models to value-based payment models B. A growing older population with more complicated health needs C. The overcrowding and understaffing of hospitals D. The shortage of health care professionals, particularly nurses

✅a. Information technology is a part of the core infrastructure on which population health can be assessed and addressed. As organizations transition from the traditional fee-for-service model to value-based payment models (including ACOs), data, information, and knowledge about populations rather than individual patients will be required. A growing older population with more complicated health needs, the overcrowding and understaffing of hospitals, and the shortage of health care professionals, particularly nurses, may be affected by population health assessment, but are not the driving force for the development of this technology. CHAPTER 20 Nursing Informatics

9.A quality-assurance program reveals a higher incidence of falls and other safety violations on a particular unit. A nurse manager states, "We'd better find the people responsible for these errors and see if we can replace them." This is an example of: A. Quality by inspection B. Quality by punishment C. Quality by surveillance D. Quality by opportunity

✅a. Quality by inspection focuses on finding deficient workers and removing them. Quality as opportunity focuses on finding opportunities for improvement and fosters an environment that thrives on teamwork, with people sharing the skills and lessons they have learned. Quality by punishment and quality by surveillance are not quality-assurance methods used in the health care field. CHAPTER 18 Evaluating

9.A nurse is using the SOAP format to document care of a patient who is diagnosed with type 2 diabetes. Which source of information would be the nurse's focus when completing this documentation? A. A patient problem list B. Narrative notes describing the patient's condition C. Overall trends in patient status D.Planned interventions and patient outcomes

✅a. The SOAP format (Subjective data, Objective data, Assessment, Plan) is used to organize entries in the progress notes of a POMR. When using the SOAP format, the problem list at the front of the chart alerts all caregivers to patient priorities. Narrative notes allow nurses to describe a condition, situation, or response in their own terms. Overall trends in patient status can be seen immediately when using CBE, not SOAP charting. Planned interventions and patient-expected outcomes are the focus of the case management model. CHAPTER 19 Documenting and Reporting

4. When may a health institution release a PHI for purposes other than treatment, payment, and routine health care operations, without the patient's signed authorization? Select all that apply. A. News media are preparing a report on the condition of a patient who is a public figure. B. Data are needed for the tracking and notification of disease outbreaks. C. Protected health information is needed by a coroner. D. Child abuse and neglect are suspected. E. Protected health information is needed to facilitate organ donation. F. The sister of a patient with Alzheimer's disease wants to help provide care.

✅b, c, d, e. According to the HIPAA, a health institution is not required to obtain written patient authorization to release PHI for tracking disease outbreaks, infection control, statistics related to dangerous problems with drugs or medical equipment, investigation and prosecution of a crime, identification of victims of crimes or disaster, reporting incidents of child abuse, neglect or domestic violence, medical records released according to a valid subpoena, PHI needed by coroners, medical examiners, and funeral directors, PHI provided to law enforcement in the case of a death from a potential crime, or facilitating organ donations. Under no circumstance can a nurse provide information to a news reporter without the patient's express authorization. An authorization form is still needed to provide PHI for a patient who has Alzheimer's disease. CHAPTER 19 Documenting and Reporting

5.Nurses incorporate telecare in patient care plans. Which services are MOST representative of this technologic advance? Select all that apply. A. Diagnostic testing B. Easy access to specialists C. Health and fitness apps D. Early warning and detection technologies E. Digital medication reminder systems F. Monitoring of progress following treatment

✅b, c, d. Telecare generally refers to technology that allows consumers to stay safe and independent in their own homes. It may include consumer-oriented health and fitness apps, sensors and tools that connect consumers with family members or other caregivers, exercise tracking tools, digital medication reminder systems, and early warning and detection technologies. Telemedicine involves the use of telecommunications technologies to support the delivery of all types of medical, diagnostic, and treatment-related services, usually by physicians or nurse practitioners. Examples include conducting diagnostic tests, monitoring a patient's progress after treatment or therapy, and facilitating access to specialists that are not located in the same place as the patient. CHAPTER 20 Nursing Informatics

3. A nurse is using informatics technology to decide which patients may be at risk for readmission. What is the term for this type of analytic? A. Data visualization B. Predictive analytics C. Big data D. Data recall

✅b. Predictive analytics encompasses a variety of statistical techniques that analyze current and historical facts to make predictions about future or otherwise unknown events. In health care, this is used by organizations to attempt to identify patients who are at risk for readmission so case managers can intervene. Data visualization is the presentation of data in a pictorial or graphical format for analysis. Big data comprises the accumulation of health care-related data from various sources, combined with new technologies that allow for the transformation of data to information, to knowledge, and ultimately to wisdom. Data recall is not a technical term for analytics. CHAPTER 20 Nursing Informatics

1. A home health care nurse is using the steps of the SDLC, to design a new system for home health care documentation. The nurse analyzes the old system and develops plans for the new system. What is the next step of the nurse in this process? A. Test B. Design C. Implement D. Evaluate

✅b. The SDLC requires focus in the areas of Analyze and Plan, Design and Build, Test, Train, Implement, Maintain, and Evaluate. After analyzing and planning the new system, the nurse would move on to the design step in which the basic design of the new system is developed. The nurse would then test the system, train employees, and implement, maintain, and evaluate the new system in that order. CHAPTER 20 Nursing Informatics

5.A friend of a nurse calls and tells the nurse that his girlfriend's father was just admitted to the hospital as a patient, and he wants the nurse to provide information about the man's condition. The friend states, "Sue seems unusually worried about her dad, but she won't talk to me and I want to be able to help her." What is the best initial response the nurse should make? A. "You shouldn't be asking me to do this. I could be fined or even lose my job for disclosing this information." B. "Sorry, but I'm not able to give information about patients to the public—even when my best friend or a family member asks." C. "Because of HIPAA, you shouldn't be asking for this information unless the patient has authorized you to receive it! This could get you in trouble!" D. "Why do you think Sue isn't talking about her worries?"

✅b. The nurse should immediately clarify what he or she can and cannot do. Since the primary reason for refusing to help is linked to the responsibility to protect patient privacy and confidentiality, the nurse should not begin by mentioning the real penalties linked to abuses of privacy. Finally, it is appropriate to ask about Sue and her worries, but this should be done after the nurse clarifies what he or she is able to do. CHAPTER 19 Documenting and Reporting

2. A nurse is documenting the care given to a patient diagnosed with an osteosarcoma, whose right leg was amputated. The nurse accidentally documents that a dressing changed was performed on the left leg. What would be the best action of the nurse to correct this documentation? A. Erase or use correcting fluid to completely delete the error. B. Mark the entry "mistaken entry"; add correct information; date and initial. C. Use a permanent marker to block out the mistaken entry and rewrite it. D. Remove the page with the error and rewrite the data on that page correctly.

✅b. The nurse should not use dittos, erasures, or correcting fluids when correcting documentation; block out a mistake with a permanent marker; or remove a page with an error and rewrite the data on a new page. To correct an error after it has been entered, the nurse should mark the entry "mistaken entry," add the correct information, and date and initial the entry. If the nurse records information in the wrong chart, the nurse should write "mistaken entry—wrong chart" and sign off. The nurse should follow similar guidelines in electronic records. CHAPTER 19 Documenting and Reporting

10. After one nursing unit with an excellent safety record meets to review the findings of the audit, the nurse manager states, "We're doing well, but we can do better! Who's got an idea to foster increased patient well-being and satisfaction?" This is an example of leadership that values: A. Quality assurance B. Quality improvement C. Process evaluation D. Outcome evaluation

✅b. Unlike quality assurance, quality improvement is internally driven, focuses on patient care rather than organizational structure, focuses on processes rather than people, and has no end points. Its goal is improving quality rather than assuring quality. Process evaluation and outcome evaluation are types of quality-assurance programs. CHAPTER 18 Evaluating

1.A nurse is documenting patient data in the medical record of a patient admitted to the hospital with appendicitis. The health care provider has ordered 10-mg morphine IV every 3 to 4 hours. Which examples of documentation of care for this patient follow recommended guidelines? Select all that apply. A. 6/12/20 0945 Morphine 10 mg administered IV. Patient's response to pain appears to be exaggerated. M. Patrick, RN B. 6/12/20 0945 Morphine 10 mg administered IV. Patient seems to be comfortable. M. Patrick, RN C. 6/12/20 0945 30 minutes following administration of morphine 10 mg IV, patient reports pain as 2 on a scale of 1 to 10. M. Patrick, RN D.6/12/20 0945 Patient reports severe pain in right lower quadrant. M. Patrick, RN E. 6/12/20 0945 Morphine IV 10 mg will be administered to patient every 3 to 4 hours. M. Patrick, RN F. 6/12/20 0945 Patient states she does not want pain medication despite return of pain. After discussing situation, patient agrees to medication administration. M. Patrick, RN

✅c, d, f. The nurse should enter information in a complete, accurate, concise, current, and factual manner and indicate in each entry the date and both the time the entry was written and the time of pertinent observations and interventions. When charting, the nurse should avoid the use of stereotypes or derogatory terms as well as generalizations such as "patient's response to pain appears to be exaggerated" or "seems to be comfortable." The nurse should never document an intervention before carrying it out CHAPTER 19 Documenting and Reporting

10. A nurse designing a new EHR system for a pediatric office follows usability concepts in system design. Which concepts are recommended in system design? Select all that apply. A. Users should not explore with forgiveness for unintended consequences. B. Shortcuts for frequent users should not be incorporated into the system. C. Content emphasis should be on information needed for decision making. D. The less times users need to apply prior experience to a new system the better. E. All the information needed should be presented to reduce cognitive load. F. The number of steps it takes to complete tasks should be minimized.

✅c, e, f. When designing a system, content emphasis should be on information needed for decision making. All the information needed should be presented to reduce cognitive load. The number of steps it takes to complete tasks should be minimized. The more users can apply prior experience to a new system, the lower the learning curve, the more effective their usage, and the fewer their errors. Forgiveness means that a design allows the user to discover it through exploration without fear of disastrous results. This approach accelerates learning while building in protections against unintended consequences. One of the most direct ways to facilitate efficient user interactions is to minimize the number of steps it takes to complete tasks and to provide shortcuts for use by frequent and/or experienced users. CHAPTER 20 Nursing Informatics

5.A nurse writes the following outcome for a patient who is trying to stop smoking: "The patient values a healthy body sufficiently to stop smoking." This is an example of what type of outcome? A. Cognitive B. Psychomotor C. Affective D. Physical changes

✅c. Affective outcomes pertain to changes in patient values, beliefs, and attitudes. Cognitive outcomes involve increases in patient knowledge; psychomotor outcomes describe the patient's achievement of new skills; physical changes are actual bodily changes in the patient (e.g., weight loss, increased muscle tone). CHAPTER 18 Evaluating

7. Nurses test new technology in phases. In which phase would the nurse "test drive" the new system? A. Unit B. Function C. User acceptance D. Integration

✅c. During the phase "user acceptance," the nurse would "test drive" the new system to ensure it's working as designed. Unit testing is basic testing that occurs initially. Function testing uses test scripts to validate that a system is working as designed for one particular function. Integration testing uses test script to validate that a system is working as designed for an entire workflow that integrates multiple components of the system. CHAPTER 20 Nursing Informatics

7. A resident who is called to see a patient in the middle of the night is leaving the unit but then remembers that he forgot to write a new order for a pain medication a nurse had requested for another patient. Tired and already being paged to another unit, he verbally tells the nurse the order and asks the nurse to document it on the health care provider's order sheet. What is the nurse's BEST response? A. State: "Thank you for taking care of this! I'll be happy to document the order on the health care provider's order sheet." B. Get a second nurse to listen to the order, and after writing the order on the health care provider order sheet, have both nurses sign it. C. State: "I am sorry, but VOs can only be given in an emergency situation that prevents us from writing them out. I'll bring the chart and we can do this quickly." D. Try calling another resident for the order or wait until the next shift.

✅c. In most facilities, the only circumstance in which an attending physician, nurse practitioner, or house officer may issue orders verbally is in a medical emergency, when the physician or nurse practitioner is present but finds it impossible, due to the emergency situation, to write the order. Trying to call another resident for the order or waiting until the next shift would be inappropriate; the patient should not have to wait for the pain medication, and a resident is available who can immediately write the order. CHAPTER 19 Documenting and Reporting

9. A nurse is using information from informatics technology that is synthesized so that relationships between lung cancer diagnoses and smoking are identified. What part of "DIKW" does this represent? A. Data B. Information C. Knowledge D. Wisdom

✅c. Knowledge is Information that is synthesized so that relationships are identified. Data refer to discrete entities that are described without interpretation. Information is data that have been interpreted, organized, or structured. Wisdom is the appropriate use of knowledge to manage and solve human problems. CHAPTER 20 Nursing Informatics

2. A nurse uses the classic elements of evaluation when caring for patients: (1) Interpreting and summarizing findings (2) Collecting data to determine whether evaluative criteria and standards are met (3) Documenting your judgment (4) Terminating, continuing, or modifying the plan (5) Identifying evaluative criteria and standards (what you are looking for when you evaluate—i.e., expected patient outcomes) Which item below places them in their correct sequence? A. 1, 2, 3, 4, 5 B. 3, 2, 1, 4, 5 C. 5, 2, 1, 3, 4 D. 2, 3, 1, 4, 5

✅c. The five classic elements of evaluation in order are (1) identifying evaluative criteria and standards (what you are looking for when you evaluate—i.e., expected patient outcomes); (2) collecting data to determine whether these criteria and standards are met; (3) interpreting and summarizing findings; (4) documenting your judgment; and (5) terminating, continuing, or modifying the plan. CHAPTER 18 Evaluating

10. A nurse is using the ISBARR physician reporting system to report the deteriorating mental status of Mr. Sanchez, a patient who has been prescribed morphine via a patient-controlled analgesia pump (PCA) for pain related to pancreatic cancer. Place the following nursing statements related to this call in the correct ISBARR order. A. "I am calling about Mr. Sanchez in Room 202 who is receiving morphine via a PCA pump for pancreatic cancer." B. "Mr. Sanchez has been difficult to arouse and his mental status has changed over the past 12 hours since using the pump." C. "You want me to discontinue the PCA pump until you see him tonight at patient rounds." D. "I am Rosa Clark, an RN working on the second floor of South Street Hospital." E. "Mr. Sanchez was admitted 2 days ago following a diagnosis of pancreatic cancer." F. "I think the dosage of morphine in Mr. Sanchez's PCA pump needs to be lowered."

✅d, a, e, b, f, c. The order for ISBARR is: Identity/Introduction, Situation, Background, Assessment, Recommendation, and Read-back. CHAPTER 19 Documenting and Reporting

3. A nurse is discharging a patient from the hospital following a heart stent procedure. The patient asks to see and copy his medical record. What is the nurse's best response? A. "I'm sorry, but patients are not allowed to copy their medical records." B. "I can make a copy of your record for you right now." C. "You can read your record while you are still a patient, but copying records is not permitted according to HIPAA rules." D. "I will need to check with our records department to get you a copy."

✅d. According to HIPAA, patients have a right to see and copy their health record; update their health record; get a list of the disclosures a health care institution has made independent of disclosures made for the purposes of treatment, payment, and health care operations; request a restriction on certain uses or disclosures; and choose how to receive health information. The nurse should be aware of facility policies regarding the patient's right to access and copy records. CHAPTER 19 Documenting and Reporting

6. A patient has an order for an analgesic medication to be given PRN. When would the nurse administer this medication? A. Every 3 hours B. Every 4 hours C. Daily D. As needed

✅d. PRN means "as needed"—not every 3 hours, every 4 hours, or once daily. CHAPTER 19 Documenting and Reporting

6. A nurse is testing a new computer program designed to store patient data. In what phase of testing would the nurse determine if the system can handle high volumes of end-users or care providers using the system at the same time? A. Unit B. Function C. Integration D. Performance

✅d. Performance testing is more technical and ensures proper functioning of the system when there are high volumes of end-users or care providers using the system at the same time, ensuring it can handle the load. Unit testing is basic testing that occurs initially. Function testing uses test scripts to validate that a system is working as designed for one particular function. Integration testing uses test script to validate that a system is working as designed for an entire workflow that integrates multiple components of the system. CHAPTER 20 Nursing Informatics

1. A student health nurse is counseling a college student who wants to lose 20 lb. The nurse develops a plan to increase the student's activity level and decrease her consumption of the wrong types of foods and excess calories. The nurse plans to evaluate the student's weight loss monthly. When the student arrives for her first "weigh-in," the nurse discovers that instead of the projected weight loss of 5 lb, the student has lost only 1 lb. Which is the BEST nursing response? A. Congratulate the student and continue the care plan. B. Terminate the care plan since it is not working. C. Try giving the student more time to reach the targeted outcome. D. Modify the care plan after discussing possible reasons for the student's partial success.

✅d. Since the student has only partially met her outcome, the nurse should first explore the factors making it difficult for her to reach her outcome and then modify the care plan. It would not be appropriate to continue the plan as it is since it is not working, and it is premature to terminate the care plan since the student has not met her targeted outcome. The student may need more than just additional time to reach her outcome. CHAPTER 18 Evaluating

7. A nurse is writing an evaluative statement for a patient who is trying to lower cholesterol through diet and exercise. Which evaluative statement is written correctly? A. "Outcome not met." B. "1/21/20—Patient reports no change in diet." C. "Outcome not met. Patient reports no change in diet or activity level." D. "1/21/20—Outcome not met. Patient reports no change in diet or activity level."

✅d. The evaluative statement must contain a date; the words "outcome met," "outcome partially met," or "outcome not met"; and the patient data or behaviors that support this decision. The other answer choices are incomplete statements. CHAPTER 18 Evaluating

4. A nurse is collecting evaluative data for a patient who is finished receiving chemotherapy for an osteosarcoma. Which nursing action represents this step of the nursing process? A. The nurse collects data to identify health problems. B. The nurse collects data to identify patient strengths. C. The nurse collects data to justify terminating the care plan. D. The nurse collects data to measure outcome achievement.

✅d. The nurse collects evaluative data to measure outcome achievement. While this may justify terminating the care plan, that is not necessarily so. Data to assess health problems and patient variables are collected during the first step of the nursing process. CHAPTER 18 Evaluating

8.A nurse is looking for trends in a postoperative patient's vital signs. Which documents would the nurse consult first? A. Admission sheet B. Admission nursing assessment C. Flow sheet D. Graphic record

✅d. While one recording of vital signs should appear on the admission nursing assessment, the best place to find sequential recordings that show a pattern or trend is the graphic record. The admission sheet does not include vital sign documentation, and neither does the flow sheet. CHAPTER 19 Documenting and Reporting

Informatics Scope and Standards Definitions: "DIKW" . ❑

❑ Data: Discrete entities that are described without interpretation ❑ Information: Data that have been interpreted, organized, or structured ❑ Knowledge: Information that is synthesized so that relationships are identified ❑ Wisdom: Appropriate use of knowledge to manage and solve human problems CHAPTER 20 Nursing Informatics

Assessing Common Problems Noted During Evaluation of the Nursing Process

❑ PROBLEM 1. Inaccurate database → inaccurate nursing diagnoses and a distorted care plan 2. Database does not reflect changes in patient condition 3. Database is superficial: • Fails to communicate uniqueness of patient • Lacks sufficient detail on major problems or developments ❑ NURSING RESPONSE 1. a. Identify the patient or nurse variables responsible for inaccuracy. b. Revise the recorded database. 2. Review with the entire nursing staff the importance of making assessment a priority in every patient interaction as well as the recording of the new data obtained. 3. a. Rethink the critical relation between an adequate database and quality care. b. Develop interviewing and physical assessment skills. c. Begin to identify the key data that need to be collected for specific nursing diagnosis and medical diagnosis and to assess patient response to therapeutic regimen (use of a nursing diagnosis handbook may be helpful).

Diagnosing Common Problems Noted During Evaluation of the Nursing Process

❑ PROBLEM. 1. General sense that nursing diagnoses are "common sense" and, therefore, do not need to be put in writing → failure to address patient's real problems 2. General sense that nurses are too busy doing treatments, "passing meds," and doing paperwork to develop nursing diagnoses carefully → independent dimension of nursing remains underdeveloped 3. Nursing diagnoses are too vague to be helpful → routinized patient care 4. Nursing diagnoses are not up to date → no one uses the care plan ❑ NURSING RESPONSE 1. Carefully develop and record priority nursing diagnoses for several patients and fairly evaluate whether this makes a difference in terms of the continuity of quality care. 2. Examine practice and see whether independent nursing has a place; what percentage of every day is devoted to independent nursing functions? If this percentage is nonexistent or small, there understandably may be no felt need for nursing diagnoses—but a desperate need to revise practice priorities. 3. a. Revise the problem statement to describe more accurately what is unhealthy about the patient (the behavior that needs to be changed). b. Revise the etiology to more accurately identify what is making the problem a problem—this should be a guide to nursing intervention. c. Check NANDA-I lists. 4. Have a process for periodically reviewing the care plan to delete nursing diagnoses when problems have been resolved and to add a new diagnosis as needed.

Implementing Common Problems Noted During Evaluation of the Nursing Process

❑ PROBLEM. 1. Nurses are not aware of patient priorities and the care plan; lack of continuity; inefficient use of nursing resources → patient fails to achieve goals/outcomes 2. Nursing care becomes routine and mechanized → patient never has the sense of being personally known by nurses 3. Documentation is inadequate → because there is no complete written record of nursing care, legally this care was never provided ❑ NURSING RESPONSE 1. a. Use shift report to update staff on status of priority nursing diagnosis and concomitant nursing care. b. Review care plan and nursing notes before beginning care. 2. Explore creative strategies to make quality nursing care on this particular unit a challenge rather than a burden; use ongoing education, problem-solving strategies by the nursing team, gaming, and other incentives. 3. a. Develop the philosophy that quality nursing care deserves to be documented. Review legal reasons for careful documentation. b. Become familiar with the flow sheets and note format used within the work setting so that charting can be done quickly and comprehensively.

.❑ Planning Common Problems Noted During Evaluation of the Nursing Process

❑ PROBLEM. 1. The care plan contains only the standard knowledge most nurses would know without a written plan 2. The long-term goal is vague, standard; fails to make clear the discharge goal for this patient 3. The nursing outcomes, even if met, do not necessarily guarantee a resolution of the patient problem 4. The outcomes are incorrectly developed; progress toward goal achievement is difficult to evaluate 5. Nursing orders are superficial → patient receives routinized care 6. The initial care plan fails to be updated → care plan will not be consulted by nurses—if used, it will be to patient's detriment 7. The care plan addresses the immediate needs of the patient but fails to anticipate discharge needs → patient returns home unable to manage self-care activities ❑ NURSING RESPONSE 1. Make use of standardized (computerized) plans as a basis for care planning. Devote nursing energies to individualizing this plan. 2. Practice writing specific long-term goals that clarify for all nurses the aim toward which all nursing care is directed (e.g., patient returns home ambulatory with walker, right hip incision healing, able to manage activities of daily living with minimal assistance from spouse). 3. When writing goals, it often helps to develop outcomes related to etiologic factors. Because the stated etiology may be incomplete or inaccurate, at least one outcome must be written so that if it is achieved, the problem in the nursing diagnosis is resolved. 4. After writing outcomes, check them against the following criteria: • Subject is the patient or some part of the patient. • The patient behavior is stated in observable, measurable terms. • Criteria of acceptable performance are specified. • Time criteria are included in notes. 5. Review nursing orders to ensure that they indicate the specific nursing strategies or interventions most likely to result in successful outcome achievement for this patient (e.g., particular comfort measures that are successful adjuncts to analgesic administration for a particular patient). In specifying the "who, what, when, where, how, and how much" of nursing interventions, be sure to list the type of equipment and supplies needed in various treatments. As new patient data are obtained, update nursing orders. Delete inappropriate or unnecessary orders. 6. If personal accountability for updating plan fails, develop a process on the nursing unit to ensure care plan review and revision. 7. Work hard at developing the ability to project yourself into the patient's home after discharge. Learn to anticipate problems and concerns and prepare the patient and family for these. Use all discharge resources in the institution. Learn from patients what their needs were after previous discharge.

Common Methods of Communication Among Health Care Professionals

❑METHOD Face-to-face meeting ❑ADVANTAGES • Message can be delivered immediately. • Nonverbal messages are readily conveyed. • Message can be clarified; receiver's questions can be raised and answered. ❑DISADVANTAGES • Both the communicating and the receiving people must be available at the same time, in the same place. • Ordinarily, there is no permanent record for later use. 2 ❑METHOD Telephone conversation ❑ADVANTAGES • Message can be delivered immediately. • Message can be clarified; receiver's questions can be raised and answered. • Two parties need not be present in same place. ❑DISADVANTAGES • Only the tone of voice and voice inflections can be communicated—no nonverbal messages unless Skype or some other synchronous means are used. • Ordinarily, there is no permanent record. 3 ❑METHOD Written message ❑ADVANTAGES • Message can be exchanged at times convenient for the people involved. • Record is available. • Time efficient if message is understood. ❑DISADVANTAGES • Message usually cannot be validated with the sender. 4 ❑METHOD Audiotaped message ❑ADVANTAGES • Message can be exchanged at times convenient for the people involved. • Record is available. • Time efficient if information communicated is complete. ❑DISADVANTAGES • Message usually cannot be validated with the sender. 5 ❑METHOD Computer message ❑ADVANTAGES • Message can be delivered immediately—even to those at a great distance. • Parties need not be present in same place. • Two-way communication is possible by e-mail. • Record is available. • Many people can participate in exchange ❑DISADVANTAGES • No nonverbal messages can be communicated. • Privacy concerns remain an issue.

TABLE 20-2 Technology Testing Phases ❑ •

❑TESTING PHASES Unit testing ❑WHEN PERFORMED During design and development ❑DESCRIPTION This is basic testing that occurs initially. • Do the screens contain the right information? • Was anything left out? • Is everything in the right place on the screen? • Are there any typos? ❑TESTING PHASE Function testing ❑WHEN PERFORMED After unit testing is complete ❑DESCRIPTION Uses test scripts to validate that a system is working as designed for one particular function. For example: • The EHR system sends a lab order to the lab system on the day they are scheduled 6 hours in advance. • The EHR system sets any order that has been active for over 365 days to a status of "auto-complete." ❑TESTING PHASE Integration testing ❑WHEN PERFORMED After function testing is complete ❑DESCRIPTION Uses test script to validate that a system is working as designed for an entire workflow that integrates multiple components of the system. For example: All steps in an EHR work as designed from order entry to administration of a medication (including the proper functioning and communication between order entry screens, the pharmacy system and the medication administration record). ❑TESTING PHASE Performance testing ❑WHEN PERFORMED After integration testing ❑DESCRIPTION Performance testing is more technical and ensures proper functioning of the system when there are high volumes of end-users or care providers using the system at the same time. Can it handle the load? ❑TESTING PHASE User acceptance testing ❑WHEN PERFORMED After all testing above ❑DESCRIPTION An important final phase of testing where the nurse (or other system end-user) test drives the new system or new functions of the EHR to ensure it's working as designed.

ANA Informatics Competencies

🔘1. ❑STANDARDS OF PRACTICE AREA Assessment ❑STANDARD The informatics nurse collects comprehensive data, information, and emerging evidence pertinent to the situation. ❑EXAMPLES OF COMPETENCIES Uses evidence-based assessment technologies, instruments, tools, and effective communication strategies in collecting pertinent data to define the issue or problem Uses workflow analysis to examine current practice, workflow, and the potential impact of an informatics solution on that workflow 🔘 2. ❑STANDARDS OF PRACTICE AREA Diagnosis, problems, and issues identification ❑STANDARD The informatics nurse analyzes assessment data to identify diagnoses, problems, issues, and opportunities for improvement. ❑EXAMPLES OF COMPETENCIES Uses standardized clinical terminologies, taxonomies, and decision support tools, when available, to identify problems, needs, issues, and opportunities for improvement 🔘3. ❑STANDARDS OF PRACTICE AREA Outcomes identification ❑STANDARD The informatics nurse identifies expected outcomes for a plan individualized to the health care consumer or the situation. ❑EXAMPLES OF COMPETENCIES Defines expected outcomes in terms of the health care consumer, health care worker, and other stakeholders, and their values, ethical considerations, and environmental, organizational, or situational considerations 🔘4. ❑STANDARDS OF PRACTICE AREA Planning ❑STANDARD The informatics nurse develops a plan that prescribes strategies, alternatives, and recommendations to attain expected outcomes. ❑EXAMPLES OF COMPETENCIES Develops a customized plan considering clinical and business characteristics of the environment and situation Integrates current scientific evidence, trends, and research into the planning process 🔘5 ❑STANDARDS OF PRACTICE AREA Implementation ❑STANDARD The informatics nurse implements the identified plan. ❑EXAMPLES OF COMPETENCIES Partners with the health care consumer, health care team, and others, as appropriate, to implement the plan on time, within budget, and within the plan requirements 🔘6. ❑STANDARDS OF PRACTICE AREA Evaluation ❑STANDARD The informatics nurse evaluates progress toward attainment of outcomes. ❑EXAMPLES OF COMPETENCIES Conducts a systematic, ongoing, and criterion-based evaluation of the outcomes in relation to the structures and processes prescribed by the project plan and indicated timeline 🔘7 ❑STANDARDS OF PRACTICE AREA Ethics ❑STANDARD The informatics nurse practices ethically. ❑EXAMPLES OF COMPETENCIES Uses Code of Ethics for Nurses with Interpretive Statements (ANA, 2015) Employs informatics principles, standards, and methodologies to establish and maintain health care consumer confidentiality within legal and regulatory parameters 🔘8. ❑STANDARDS OF PRACTICE AREA Education ❑STANDARD The informatics nurse attains knowledge and competence that reflect current nursing and informatics practice. ❑EXAMPLES OF COMPETENCIES Participates in ongoing education to advance his or her knowledge base and professional practice Shares educational findings, experiences, and ideas with peers 🔘9. ❑STANDARDS OF PRACTICE AREA Evidence-based practice and research ❑STANDARD The informatics nurse integrates evidence and research findings into practice. ❑EXAMPLES OF COMPETENCIES Uses data to communicate evidence to promote effective care processes and decisions Uses the skills and tools available to the informatics nurse for research studies . 🔘10 ❑STANDARDS OF PRACTICE AREA Quality of practice ❑STANDARD The informatics nurse contributes to quality and effectiveness of nursing and informatics practice. ❑EXAMPLES OF COMPETENCIES Demonstrates quality by documenting the application of the nursing process in a responsible, accountable, and ethical manner, and by facilitating a unified or defined level of documentation by nurses in clinical practice Analyzes quality data to identify opportunities to improve nursing and informatics practice or outcomes

❑ Abbreviations and Symbols Commonly Used

🔘Activities AMB. ambulatory BRP. bathroom privileges CBR. complete bed rest OOB. out of bed up ad lib. up as desired 🔘Miscellaneous aˉ before ad lib. as desired AMA against medical advice ASAP as soon as possible BM bowel movement BSD bedside drainage cˉ (C) with CABG. coronary artery bypass graft CPR. cardiopulmonary resuscitation DNR (no code). do not resuscitate Dsg. dressing dx. diagnosis FOB. foot of bed Fx. fracture GHWT. good handwashing technique HOB. head of bed Hx. history I&O. intake and output IV. intravenous KVO. keep vein open NG. nasogastric noc. night NPO (npo). nothing by mouth NS. normal saline. (NIS) (N/S) O2. oxygen OT. occupational therapy pˉ. after postop. postoperative preop. preoperative prep. preparation PRN (p.r.n.). as needed PT. physical therapy pt. patient ROM. range of motion RX. treatment sˉ(S). without SOB. side of bed S/P. status post STAT. immediately TF. tube feeding TPR. temperature, pulse, respirations TX. treatment VS. vital signs WA. while awake ×. times

Personal Critical Thinking Indicators (CTIs) Knowledge Base

PERSONAL CTIs are brief descriptions of behaviors, attitudes, and characteristics often seen in critical thinkers. These are the behaviors that promote development of critical thinking habits. The below is the ideal—no one's perfect. Characteristics vary depending on circumstances such as comfort and familiarity with the people and situations at hand. What matters is patterns of behavior over time (is the behavior usually evident?). If you're a critical thinker, you probably can pick some characteristics you'd like to improve (critical thinkers are innately focused on improvement). ❑ Self-aware: Identifies own learning, personality, and communication style preferences; clarifies biases, strengths, and limitations; acknowledges when thinking may be influenced by emotions or self-interest. ❑ Genuine/authentic: Shows true self; demonstrates behaviors that indicate stated values. ❑ Effective communicator: Listens well (shows deep understanding of others' thoughts, feelings, and circumstances); speaks and writes with clarity (gets key points across to others). ❑ Curious and inquisitive: Asks questions; looks for reasons, explanations, and meaning; seeks new information to broaden understanding. ❑ Alert to context: Looks for changes in circumstances that warrant a need to modify approaches; investigates thoroughly when situations warrant precise, in depth thinking. ❑ Reflective and self-corrective: Carefully considers meaning of data and interpersonal interactions, asks for feedback; corrects own thinking, alert to potential errors by self and others, finds ways to avoid future mistakes. ❑ Analytical and insightful: Identifies relationships; expresses deep understanding. ❑ Logical and intuitive: Draws reasonable conclusions (if this is so, then it follows that because...); uses intuition as a guide; acts on intuition only with knowledge of risks involved. ❑ Confident and resilient: Expresses faith in ability to reason and learn; overcomes problems and disappointments. ❑ Honest and upright: Looks for the truth, even if it sheds unwanted light; demonstrates integrity (adheres to moral and ethical standards; admits flaws in thinking). ❑ Autonomous/responsible: Self-directed, self-disciplined, and accepts accountability. ❑ Careful and prudent: Seeks help as needed; suspends or revises judgment as indicated by new or incomplete data. ❑ Open and fair-minded: Shows tolerance for different viewpoints; questions how own viewpoints are influencing thinking. ❑ Sensitive to diversity: Expresses appreciation of human differences related to values, culture, personality, or learning style preferences; adapts to preferences when feasible. ❑ Creative: Offers alternative solutions and approaches; comes up with useful ideas. ❑ Realistic and practical: Admits when things aren't feasible; looks for useful solutions. ❑ Proactive: Anticipates consequences, plans ahead, acts on opportunities. ❑ Courageous: Stands up for beliefs, advocates for others, doesn't hide from challenges ❑ Patient and persistent: Waits for right moment; perseveres to achieve best results. ❑ Flexible: Changes approaches as needed to get the best results. ❑ Health-oriented: Promotes a healthy lifestyle; uses healthy behaviors to manage stress. ❑ Improvement-oriented (self, patients, systems): Self—identifies learning needs; finds ways to overcome limitations, seeks out new knowledge. Patients—promotes health; maximizes function, comfort, and convenience. Systems—identifies risks and problems with health care systems; promotes safety, quality, satisfaction, and cost containment. Jarvis CHAPTER 13 Blended Competencies, Clinical Reasoning, and Processes of Person-Centered Care

A nurse is providing teaching to a client who has a new diagnosis of multiple sclerosis (MS). The clients asks the nurse about the usual course of MS. Which of the following responses should the nurse make? "Each client is different; we cannot predict what will happen." "I can see that you are worried, but it's too soon to predict what will happen." "Acute episodes are usually followed by remissions, which can vary in duration." "It's too early to think about the future; let's focus on the present and take one day at a time."

"Each client is different; we cannot predict what will happen." The client is asking the nurse an information-seeking question. This answer does not provide information to the client and blocks further communication. "I can see that you are worried, but it's too soon to predict what will happen." The client is asking the nurse an information-seeking question. In this response, the nurse acknowledges the client's feelings but then blocks communication by not providing any information to help address the client's fears. ✅"Acute episodes are usually followed by remissions, which can vary in duration." The nurse should identify that the client is asking an information-seeking question. The nurse should provide the client with factual information. The nurse should inform the client that MS is a chronic autoimmune disorder that is characterized by remissions and exacerbations, with exacerbations becoming more frequent and intense as the disease progresses. "It's too early to think about the future; let's focus on the present and take one day at a time." The client is asking the nurse an information-seeking question. In this response, the nurse is giving advice, which can block communication by discouraging the client from feeling free to express concerns and fears. RN Learning System Medical-Surgical: Final Quiz

A nurse is providing discharge teaching to an adult female client who has infective endocarditis about how to prevent recurrence. Which of the following statements by the client indicates understanding of the teaching? "I will ask my provider to change my contraception to an intrauterine device." "I will notify my doctor before I have dental procedures." "I will avoid using antiseptic mouthwash during my oral care." "I will wear a mask when I go out in public."

"I will ask my provider to change my contraception to an intrauterine device." The nurse should inform the client that intrauterine devices increase the risk of an infection, which can lead to recurrence of infective endocarditis. ✅"I will notify my doctor before I have dental procedures." The nurse should inform the client of ways to decrease the risk of recurrence of infective endocarditis. The client should notify the provider prior to invasive or dental procedures due to the need for prophylactic antibiotic therapy to reduce the risk of a streptococcal infection. "I will avoid using antiseptic mouthwash during my oral care." The nurse should inform the client that good oral hygiene reduces the risk of recurrence of infective endocarditis. The client should use an antiseptic mouthwash for 30 seconds twice daily as part of personal oral care. "I will wear a mask when I go out in public." The nurse should inform the client that it is not necessary to wear a mask when going out in public since infective endocarditis does not result in immunosuppression. The client should, however, avoid contact with individuals who have a streptococcal infection. RN Learning System Medical-Surgical: Final Quiz

A nurse is obtaining a weekly weight for a client who has obesity and osteoarthritis and is on a weight management program. The nurse determines that the client gained 1.36 kg (3 lb) in the past week. Which of the following statements should the nurse make? "You should try a little harder to stick to your diet." "Why do you think you've gained 3 pounds this week?" "Were there any issues last week that kept you from focusing on your diet?" "You should put this week behind you and adhere to your diet from this point forward."

"You should try a little harder to stick to your diet." The nurse should avoid directing blame towards the client, which can result in a defensive response and decreased motivation. "Why do you think you've gained 3 pounds this week?" The nurse should avoid the use of "why" questions, which can promote a defensive client response. ✅"Were there any issues last week that kept you from focusing on your diet?" The nurse should use an open-ended question that allows the client to reassess the past week in a non-threatening manner. The nurse's statement demonstrates concern without placing blame. The nurse should explain to the client that relapse is a normal part of making a behavior change, and that physical and emotional stress can play a part in his motivational level. "You should put this week behind you and adhere to your diet from this point forward." The nurse should avoid providing the client with approval or personal advice, which does not promote a professional nurse-client relationship. RN Learning System Medical-Surgical: Final Quiz

1. Read the following scenario and identify the adjective used to describe the characteristics of patient data that are numbered below. Place your answers on the lines provided. The nurse is conducting an initial assessment of a 79-year-old female patient admitted to the hospital with a diagnosis of dehydration. The nurse (1) uses clinical reasoning to identify the need to perform a comprehensive assessment and gather the appropriate patient data. (2) First the nurse asks the patient about the most important details leading up to her diagnosis. Then the nurse (3) collects as much information as possible to understand the patient's health problems; (4) collects the patient data in an organized manner; (5) verifies that the data obtained is pertinent to the patient care plan; and (6) records the data according to facility's policy. (1) ___________________ (2) ___________________ (3) ___________________ (4) ___________________ (5) ___________________ (6) ___________________

(1) Purposeful: The nurse identifies the purpose of the nursing assessment (comprehensive) and gathers the appropriate data. (2) Prioritized: The nurse gets the most important information first. (3) Complete: The nurse gathers as much data as possible to understand the patient health problem and develop a care plan. (4) Systematic: The nurse gathers the information in an organized manner. (5) Accurate and relevant: The nurse verifies that the information is reliable. (6) Recorded in a standard format: The nurse records the data according to the facility's policy so that all caregivers can easily access what is learned Jarvis CHAPTER 14 Assessing

1. Read the following patient scenario and identify the step of the nursing process represented by each numbered and boldfaced nursing activity. Annie seeks the help of the nurse in the student health clinic because she suspects that her roommate, Angela, suffered date rape. She is concerned because Angela chose not to report the rape and does not seem to be coping well. (1) After talking with Annie, the nurse learns that although Angela blurted out that she had been raped when she first came home, since then she has refused verbalization about the rape ("I don't want to think or talk about it"), has stopped attending all college social activities (a marked change in behavior), and seems to be having nightmares. After analyzing the data, the nurse believes that Angela might be experiencing (2) rape-trauma syndrome: silent reaction. Fortunately, Angela trusts Annie and is willing to come to the student health center for help. A conversation with Angela confirms the nurse's suspicions, and problem identification begins. The nurse talks further with Angela (3) to develop some treatment goals and formulate outcomes. The nurse also begins to think about the types of nursing interventions most likely to yield the desired outcomes. In the initial meeting with Angela, (4) the nurse encourages her expression of feelings and helps her to identify personal coping strategies and strengths. The nurse and Angela decide to meet in 1 week (5) to assess her progress toward achieving targeted outcomes. If she is not making progress, the care plan might need to be modified. (1) _____________________ (2) _____________________ (3) _____________________ (4) _____________________ (5) _____________________

(1) is an illustration of assessing: the collection of patient data. (2) is an illustration of the identification of a nursing diagnosis: a health problem that independent nursing intervention can resolve. (3) is an illustration of planning: outcome identification and related nursing interventions. (4) is an illustration of implementing: carrying out the care plan. (5) is an illustration of evaluating: measuring the extent to which Angela has achieved targeted outcomes. Jarvis CHAPTER 13 Blended Competencies, Clinical Reasoning, and Processes of Person-Centered Care

A nurse in an emergency department is assessing a client who sustained a fall off of a roof. Which of the following findings should the nurse identify as an indication of a basilar skull fracture? A depressed fracture of the forehead Clear fluid coming from the nares Motor loss on one side of the body Bleeding from the top of the scalp

A depressed fracture of the forehead Though a client who has a depressed fracture of the forehead might also have additional head trauma, this finding does not indicate a basilar skull fracture, which occurs at the base of the skull. ✅Clear fluid coming from the nares The nurse should identify cerebrospinal fluid, which appears as a clear fluid, coming from the nares or ears as an indication of a basilar skull fracture. Motor loss on one side of the body Motor loss on one side of the body is an indication of an injury to the cerebral hemisphere. The motor dysfunction will be contralateral to the site of, which mean on the opposite side of, the injury, similar to the results of a stroke. Loss of motor function can also be an indication that injury has occurred to the spinal cord. Bleeding from the top of the scalp Though a client who has bleeding from the scalp might also have additional head trauma, this finding does not indicate a basilar skull fracture, which occurs at the base of the skull. RN Learning System Medical-Surgical: Final Quiz

A nurse is caring for a client who is receiving total parenteral nutrition (TPN). Which of the following actions should the nurse take? Administer 0.9% sodium chloride until TPN is available from the pharmacy. Check the client's capillary blood glucose level every 4 hr. Obtain the client's weight each week. Change the IV tubing every 3 days.

Administer 0.9% sodium chloride until TPN is available from the pharmacy. The nurse should administer 10% dextrose in water or 20% dextrose in water if TPN is temporarily unavailable from the pharmacy. ✅Check the client's capillary blood glucose level every 4 hr. The nurse should check the client's capillary blood glucose level every 4 hr, or according to facility policy, due to the client's risk for hyperglycemia while receiving TPN. The dextrose concentration in TPN places the client at risk for this complication. Obtain the client's weight each week. The nurse should recognize that the client who is receiving TPN is at risk for fluid imbalance due to the fluid administration and hyperosmolarity of the TPN; therefore, the nurse should monitor the client's weight daily. Change the IV tubing every 3 days. The nurse should change the IV tubing used for TPN every 24 hr to decrease the client's risk for infection. RN Learning System Medical-Surgical: Final Quiz

A nurse is planning care for a client who has pernicious anemia. Which of the following interventions should the nurse include in the plan? Administer ferrous sulfate supplementation. Increase dietary intake of folic acid. Initiate weekly injections of vitamin B12. Initiate a blood transfusion.

Administer ferrous sulfate supplementation. The nurse should administer ferrous sulfate to a client who has iron deficiency anemia, which is a decrease in the red blood cells caused by inadequate intake of dietary iron. Increase dietary intake of folic acid. The nurse should increase the intake of food containing folic acid for a client who has megaloblastic anemia, which is a decrease in the red blood cells caused by folate deficiency. ✅Initiate weekly injections of vitamin B12. The nurse should initiate weekly injections of vitamin B12 for a client who has pernicious anemia, and then decrease to monthly. Pernicious anemia is caused by a lack of intrinsic factor needed to absorb vitamin B12 from the gastrointestinal tract. Initiate a blood transfusion. The nurse should initiate a blood transfusion for a client who has aplastic anemia when bleeding is life-threatening from a low platelet count or if a client has blood loss from trauma or surgery. RN Learning System Medical-Surgical: Cardiovascular and Hematology Practice Quiz

A nurse is caring for a client who has a lesion on the back of his right hand. The client asks the nurse which type of skin cancer is the most serious. Which of the following responses by the nurse is appropriate? Basal cell carcinomas Melanomas Actinic keratoses Squamous cell carcinomas

Basal cell carcinomas Basal cell carcinomas are the most common type of skin cancer and rarely metastasize. They arise from the basal cell layer of the epidermis or the hair follicles and generally appear on sun-exposed areas of the body. ✅Melanomas Melanomas are malignant neoplasms with atypical melanocytes in both the epidermis, the dermis, and sometimes the subcutaneous cells. It is the most lethal type of skin cancer, often causing metastases in the bone, liver, lungs, spleen, the CNS, and lymph nodes. Actinic keratoses Actinic keratoses are premalignant. They can evolve to squamous cell carcinomas; however, they are not the most lethal type of skin cancer. Squamous cell carcinomas Squamous cell carcinomas arise from the epidermis and are potentially metastatic; however, they are not the most lethal type of skin cancer. RN Learning System Medical-Surgical: Dermatological Practice Quiz

Components of a NANDA-I Nursing Diagnosis

Be sure to ask patients what they believe their most important problems or issues are and have them confirm what your assessment reveals and helps you prioritize the resulting list of diagnoses/problems. The acronyms PE and PED can be helpful in remembering how to write a nursing diagnosis. PE stands for "problem" and "etiology"; PED stands for "problem," "etiology," and "defining characteristics." A problem-focused nursing diagnosis for a patient who has experienced vomiting, diarrhea, and excessive diaphoresis for 3 days is Deficient fluid volume related to abnormal fluid loss. If the diarrhea persists and weakness interferes with the patient's normal perineal hygiene, the patient might be at risk for skin breakdown. This is written as the risk diagnosis, Risk for impaired skin integrity. This list represents the NANDA-approved nursing diagnoses for clinical use and testing. ⏺Domain 1: Health Promotion Definition The awareness of well-being or normality of function and the strategies used to maintain control of and enhance that well-being or normality of function ❑Class 1. Health Awareness • Decreased diversional activity engagement • Readiness for enhanced health literacy • Sedentary lifestyle ❑Class 2. Health Management • Frail elderly syndrome • Risk for frail elderly syndrome Deficient community health • Risk-prone health behavior Ineffective health maintenance • Ineffective health management Readiness for enhanced health management • Ineffective family health management • Ineffective protection • ❑ ⏺Domain 2: Nutrition Definition The activities of taking in, assimilating, and using nutrients for the purposes of tissue maintenance, tissue repair, and the production of energy ❑ 1. Ingestion •Imbalanced nutrition: less than body requirements •Readiness for enhanced nutrition •Insufficient breast milk production •Ineffective breastfeeding •Interrupted breastfeeding Readiness for enhanced breastfeeding •Ineffective adolescent eating dynamics Ineffective child eating dynamics Ineffective infant feeding dynamics Ineffective infant feeding pattern Obesity •Overweight Risk for overweight Impaired swallowing . ❑ Metabolism Risk for unstable blood glucose level •Neonatal hyperbilirubinemia Risk for hyperbilirubinemia •Risk for impaired liver function Risk for metabolic imbalance ❑ Hydration •Risk for electrolyte imbalance •Risk for imbalanced fluid volume •Deficient fluid volume •Risk for deficient fluid volume •Excess fluid volume CHAPTER 15 Diagnosing

Coping/Stress Tolerance—>Terminology for Writing Nursing Diagnoses

CHAPTER 15 Diagnosing. • ❑

A nurse is assessing for cardiac tamponade on a client who had coronary artery bypass grafts. Which of the following actions should the nurse take? Check for hypertension. Auscultate for loud, bounding heart sounds. Auscultate blood pressure for pulsus paradoxus. Check for a pulse deficit.

Check for hypertension. The client who has cardiac tamponade will have hypotension because of the sudden decrease in cardiac output from the fluid compressing the atria and ventricles. Auscultate for loud, bounding heart sounds. The client who has cardiac tamponade will have muffled heart sounds on auscultation due to the fluid compressing the atria and ventricles. ✅Auscultate blood pressure for pulsus paradoxus. The client who has cardiac tamponade will have pulsus paradoxus when the systolic blood pressure is at least 10 mm Hg higher on expiration than on inspiration. This occurs because of the sudden decrease in cardiac output from the fluid compressing the atria and ventricles. Check for a pulse deficit. The nurse will not detect cardiac tamponade by checking for a pulse deficit. This is performed by checking the apical and radial pulses simultaneously to determine if the rate is the same. If the rate is different, the findings indicate a cardiac dysrhythmia. RN Learning System Medical-Surgical: Cardiovascular and Hematology Practice Quiz

A nurse is transfusing a unit of B-positive fresh frozen plasma to a client whose blood type is O-negative. Which of the following actions should the nurse take? Continue to monitor for manifestations of a transfusion reaction. Remove the unit of plasma immediately and start an IV infusion of normal saline solution. Continue the transfusion and repeat the type and crossmatch. Prepare to administer a dose of diphenhydramine IV.

Continue to monitor for manifestations of a transfusion reaction. ABO compatibility is required for the transfusion of fresh frozen plasma. A client whose blood type is O can only receive type O plasma. ✅Remove the unit of plasma immediately and start an IV infusion of normal saline solution. A client who receives FFP that is not compatible can experience a hemolytic transfusion reaction. The nurse should stop the transfusion and infuse 0.9% sodium chloride solution with new tubing. Continue the transfusion and repeat the type and crossmatch. The nurse should not continue infusing plasma that is not compatible with the client. There is no indication that a repeat type and crossmatch of the client's blood is necessary. Prepare to administer a dose of diphenhydramine IV. The nurse should administer diphenhydramine IV only if the client manifests an allergic transfusion reaction. RN Learning System Medical-Surgical: Cardiovascular and Hematology Practice Quiz

Safety/Protection—>Terminology for Writing Nursing Diagnoses

Domain 11: Safety/Protection Definition Freedom from danger, physical injury, or immune system damage; preservation from loss; and protection of safety and security ❑. Infection •Risk for infection •Risk for surgical site infection ❑. Physical Injury Ineffective airway clearance •Risk for aspiration •Risk for bleeding •Impaired dentition •Risk for dry eye •Risk for dry mouth Risk for falls •Risk for corneal injury •Risk for injury Risk for urinary tract injury Risk for perioperative positioning injury •Risk for thermal injury •Impaired oral mucous membrane integrity •Risk for impaired oral mucous membrane integrity Risk for peripheral neurovascular dysfunction •Risk for physical trauma •Risk for vascular trauma •Risk for pressure ulcer •Risk for shock Impaired skin integrity •Risk for impaired skin integrity Risk for sudden infant death •Risk for suffocation •Delayed surgical recovery •Risk for delayed surgical recovery •Impaired tissue integrity •Risk for impaired tissue integrity •Risk for venous thromboembolism ❑Violence •Risk for female genital mutilation •Risk for other-directed violence •Risk for self-directed violence •Self-mutilation •Risk for self-mutilation •Risk for suicide ❑. Environmental Hazards •Contamination •Risk for contamination •Risk for occupational injury •Risk for poisoning Class 5. Defensive Processes •Risk for adverse reaction to iodinated contrast media •Risk for allergy reaction •Latex allergy reaction •Risk for latex allergy reaction ❑ Thermoregulation •Hypothermia •Hyperthermia •Risk for hypothermia •Risk for perioperative hypothermia •Ineffective thermoregulation •Risk for ineffective thermoregulation CHAPTER 15 Diagnosing. • ❑

A nurse is monitoring a client following a thyroidectomy for the presence of hypoparathyroidism. Which of the following findings should the nurse expect? Elevated blood pressure Involuntary muscle spasms Cold intolerance Weight loss

Elevated blood pressure The nurse should identify hypertension as an indication of thyroid storm, which is a potential complication following a thyroidectomy. ✅Involuntary muscle spasms The nurse should identify involuntary muscle spasms as an indication of hypoparathyroidism, which can occur if the parathyroid glands are damaged or removed during a thyroidectomy. Muscle twitching and paresthesias can result due to decreased parathyroid hormone levels and calcium deficiency. Cold intolerance The nurse should identify cold intolerance as an indication of hypothyroidism. Weight loss The nurse should identify weight loss as an indication of hyperthyroidism. RN Learning System Medical-Surgical: Final Quiz

A nurse in an emergency department is assessing a client who has extensive burns, including on her face. Which of the following assessments should the nurse perform first? Estimation of burn injury Characteristics of the cough and sputum Extent of peripheral edema Amount of urine output

Estimation of burn injury The nurse should determine the percentage of the client's total body surface area that is burned to ensure proper care and estimation of prognosis; however, there is another assessment that the nurse should perform first. ✅Characteristics of the cough and sputum The nurse should apply the ABC priority-setting framework. This framework emphasizes the basic core of human functioning: having an open airway, being able to breathe in adequate amounts of oxygen, and circulating oxygen to the body's organs via the blood. An alteration in any of these can indicate a threat to life and is the nurse's priority concern. When applying the ABC priority-setting framework, airway is always the highest priority because the airway must be clear and open for oxygen exchange to occur. Breathing is the second-highest priority in the ABC priority-setting framework because adequate ventilatory effort is essential in order for oxygen exchange to occur. Circulation is the third-highest priority in the ABC priority-setting framework because delivery of oxygen to critical organs only occurs if the heart and blood vessels are capable of efficiently carrying oxygen to them. Therefore, the nurse's priority assessment is the client's cough characteristics. A client who has burns to the face is at risk for pulmonary injury and the development of a brassy cough can indicate impending loss of airway. Extent of peripheral edema The nurse should assess the extent of the client's edema to determine the effects of the injury on the client's cardiovascular status; however, there is another assessment that the nurse should perform first. Amount of urine output The nurse should accurately monitor the client's urine output to assess the client's kidney function; however, there is another assessment that the nurse should perform first. RN Learning System Medical-Surgical: Final Quiz

A nurse is preparing to transfuse a unit of packed red blood cells (RBCs) to a client who has anemia. Which of the following actions should the nurse take first? Hang an IV infusion of 0.9% sodium chloride with the blood. Check the client's identification number with the number on the blood. Witness the informed consent. Obtain pretransfusion vital signs.

Hang an IV infusion of 0.9% sodium chloride with the blood. The nurse should hang an IV infusion of 0.9% sodium chloride with the blood to dilute the blood and maintain the IV infusion line. However, the nurse should take a less invasive intervention first. Check the client's identification number with the number on the blood. The nurse should check the client's identification number with the number on the blood to ensure the client receives the correct unit of blood. With another nurse, the nurse should check the provider prescription, the identity of the blood product, the client, and the compatibility (blood type and Rh factor) of the blood and the client. However, the nurse should take a less invasive intervention first. ✅Witness the informed consent. The nurse should apply the least invasive priority-setting framework. This framework assigns priority to nursing interventions that are least invasive to the client, as long as those interventions do not jeopardize client safety. The nurse should take interventions that are not invasive to the client before interventions that are invasive; therefore, as witnessing the informed consent is the least invasive, it is the action that should be performed first. Unless it is an emergency, informed consent should be obtained prior to initiating a blood transfusion to a client. Obtain pretransfusion vital signs. The nurse should obtain the client's pretransfusion vital signs prior to infusing the packed RBCs. However, the nurse should take a less invasive intervention first. RN Learning System Medical-Surgical: Cardiovascular and Hematology Practice Quiz

A nurse is evaluating the laboratory values of a client who is in the resuscitation phase following a major burn. Which of the following laboratory findings should the nurse expect? Hemoglobin 10 g/dL Sodium 132 mEq/L Albumin 3.6 g/dL Potassium 4.0 mEq/dL

Hemoglobin 10 g/dL This laboratory value is below the expected reference range. The nurse should anticipate an elevated hemoglobin level during the resuscitation phase due to loss of fluid volume. ✅Sodium 132 mEq/L This laboratory finding is below the expected reference range. The nurse should anticipate a low sodium level because sodium is trapped in interstitial space. Albumin 3.6 g/dL This laboratory finding is within the expected reference range. The nurse should anticipate a low albumin level during the resuscitation phase. Potassium 4.0 mEq/dL This laboratory finding is within the expected reference range. The nurse should anticipate an elevated potassium level during the resuscitation phase. RN Learning System Medical-Surgical: Dermatological Practice Quiz

A nurse is providing teaching to a client who has anemia and a new prescription for epoetin alfa. Which of the following information should the nurse include in the teaching? Hospitalization is required when administering each treatment. The maximum effect of the medication will occur in 6 months. Hypertension is a common adverse effect of this medication. Blood transfusions are needed with each treatment.

Hospitalization is required when administering each treatment. The nurse should teach the client that epoetin alfa can be self-administered at home. The maximum effect of the medication will occur in 6 months. The nurse should teach that the maximum effect of epoetin alfa will occur in 2 to 3 months. ✅Hypertension is a common adverse effect of this medication. The nurse should teach that a common adverse effect of epoetin alfa is hypertension because of the rise in the production of erythrocytes and other blood cell types. Epoetin alfa is a synthetic version of human erythropoietin. Epoetin alfa is used to treat anemia associated with kidney disease or medication therapy. It increases and maintains the red blood cell level. Blood transfusions are needed with each treatment. The nurse should teach that epoetin alfa is administered to decrease the need for periodic blood transfusions. RN Learning System Medical-Surgical: Cardiovascular and Hematology Practice Quiz

A nurse is caring for a client who has a prescription for silver sulfadiazine cream to be applied to her burn wounds. The nurse should evaluate the client for which of the following laboratory findings? Hyponatremia Leukopenia Hyperchloremia Elevated BUN

Hyponatremia Silver sulfadiazine does not cause electrolyte imbalance. ✅Leukopenia Transient leukopenia is an adverse effect of silver sulfadiazine. Hyperchloremia Hyperchloremia and other electrolyte imbalances can be adverse effects of mafenide acetate solution or cream. Elevated BUN Impaired kidney function is an adverse effect of gentamicin. RN Learning System Medical-Surgical: Dermatological Practice Quiz

A nurse is preparing to administer eye drops for a client who has glaucoma. When instilling the medication, which of the following actions should the nurse take? Instruct the client to blink several times after instilling the medication. Ask the client to look straight ahead during instillation of the medication. Apply pressure to the puncta after instilling the medication. Place each drop of the medication directly on to the client's cornea.

Instruct the client to blink several times after instilling the medication. The nurse should instruct the client to gently close the eyes and to avoid blinking after instillation to prevent loss of the medication out of the eye and promote absorption. Ask the client to look straight ahead during instillation of the medication. The nurse should instruct the client to look upward toward the ceiling during instillation of the medication to allow for proper placement of the medication and to suppress the client's blink reflex. ✅Apply pressure to the puncta after instilling the medication. The nurse should instill the medication into the conjunctival sac and apply pressure to the puncta for 1 to 2 min afterwards to prevent systemic absorption of the medication. Place each drop of the medication directly on to the client's cornea. The nurse should instill the medication into the client's conjunctival sac and should take measures to protect the client's cornea during administration of the medication. RN Learning System Medical-Surgical: Final Quiz

A nurse is administering a unit of packed red blood cells (RBCs) to a client who is postoperative. The client reports itching and has hives 30 min after the infusion begins. Which of the following actions should the nurse take first? Maintain the IV access with 0.9% sodium chloride. Stop the infusion of blood. Send the blood container and tubing to the blood bank. Obtain a urine sample.

Maintain the IV access with 0.9% sodium chloride. Maintaining IV access by initiating an infusion of 0.9% sodium chloride solution using a new IV administration set is important. However, there is another action that is the nurse's priority. ✅Stop the infusion of blood. The nurse should apply the urgent vs. nonurgent priority-setting framework. Using this framework, the nurse should consider urgent needs the priority because they pose more of a threat to the client. The nurse might also need to use Maslow's hierarchy of needs, the ABC priority-setting framework, or nursing knowledge to identify which finding is the most urgent. The nurse should stop the infusion of blood because the client has manifestations of an allergic reaction. Send the blood container and tubing to the blood bank. The nurse should send the blood container and tubing to the blood bank for a repeat typing and culture. However, there is another action that is the nurse's priority. Obtain a urine sample. The nurse should obtain a urine sample from the client to determine if hemoglobin is in the urine. However, there is another action that is the nurse's priority. RN Learning System Medical-Surgical: Cardiovascular and Hematology Practice Quiz

A nurse is teaching a client about transmission prevention of hepatitis A. The nurse should identify that hepatitis A is transmitted by which of the following routes? Maternal-fetal Fecal-oral contamination Genital sexual contact Blood to blood

Maternal-fetal The nurse should identify that hepatitis B is transmitted by several routes, including the maternal-fetal route. ✅Fecal-oral contamination The nurse should identify that hepatitis A is most commonly transmitted by the fecal-oral route. Transmission occurs by ingesting food or liquid that has been infected with the virus. Outbreaks due to contaminated food are usually due to poor hygiene practices by food handlers or shellfish from contaminated water. Genital sexual contact The nurse should identify that hepatitis B, C, and D are transmitted by several routes, including unprotected sexual intercourse. Blood to blood The nurse should identify that hepatitis B and C are transmitted by several routes, including blood to blood exposure. RN Learning System Medical-Surgical: Final Quiz

A nurse is caring for a client who has hemophilia. The client reports pain and swelling in a joint following an injury. Which of the following actions should the nurse take? Obtain blood samples to test platelet function. Prepare for replacement of the missing clotting factor. Administer aspirin for the client's pain. Place the bleeding joint in the dependent position.

Obtain blood samples to test platelet function. Coagulation tests that measure platelet function, such as bleeding time, are used to diagnose, not treat, hemophilia. ✅Prepare for replacement of the missing clotting factor. Hemophilia is a hereditary bleeding disorder in which blood clots slowly and abnormal bleeding occurs. It is caused by a deficiency in the most common clotting factor, factor VIII (hemophilia A). Aggressive factor replacement is initiated to prevent hemarthrosis that can result in long-term loss of range of motion in repeatedly affected joints. Administer aspirin for the client's pain. Medications that interfere with clotting function, such as aspirin, NSAIDS, and some herbal supplements, should be avoided. Place the bleeding joint in the dependent position. The affected joint should be elevated to allow the blood to drain away from the joint. RN Learning System Medical-Surgical: Cardiovascular and Hematology Practice Quiz

A nurse is planning dietary teaching for a client who has diabetes mellitus. Which of the following actions should the nurse plan to take first? Obtain sample menus from the dietitian to give to the client. Ask the client to identify the types of foods she prefers. Identify the recommended range for the client's blood glucose level. Discuss long-term complications that can result from nonadherence to the dietary plan.

Obtain sample menus from the dietitian to give to the client. The nurse should work with a registered dietitian to provide the client with appropriate materials to use during the dietary teaching. Sample menus can be helpful in providing the client with ideas for new foods or exchanges; however, there is another action that the nurse should take first. ✅Ask the client to identify the types of foods she prefers. The nurse should apply the nursing process priority-setting framework. The nurse can use the nursing process to plan client care and prioritize nursing actions. Each step of the nursing process builds on the previous step, beginning with assessment or data collection. Before the nurse can formulate a plan of action, implement a nursing intervention, or notify the provider of a change in the client's status, he must first collect adequate data from the client. Assessing or collecting additional data will provide the nurse with knowledge to make an appropriate decision. Therefore, the nurse should first ask the client about individual food preferences to provide an opportunity for the nurse to include these foods in her diet. Involving the client in the planning will help promote her adherence to the dietary plan. Identify the recommended range for the client's blood glucose level. The nurse should identify the recommended blood glucose range that the client should maintain through diet, medication, and lifestyle changes; however, there is another action that the nurse should take first. Discuss long-term complications that can result from nonadherence to the dietary plan. The nurse should identify the long-term complications so the client understands the importance of adherence to the dietary plan; however, there is another action that the nurse should take first. RN Learning System Medical-Surgical: Final Quiz

A nurse is caring for a client who has a stage III pressure ulcer on the heel. When preparing to irrigate the wound, which of the following actions should the nurse take first? Obtain the prescribed irrigation solution. Don personal protective equipment. Check the client's pain level. Place a waterproof pad under the client's extremity.

Obtain the prescribed irrigation solution. The nurse should obtain the prescribed irrigation solution prior to performing the procedure; however, there is another action the nurse should take first. Don personal protective equipment. The nurse should don personal protective equipment prior to performing the procedure to prevent exposure to blood or body fluids from the client's wound; however, there is another action the nurse should take first. ✅Check the client's pain level. The nurse should apply the nursing process priority-setting framework. The nurse can use the nursing process to plan client care and prioritize nursing actions. Each step of the nursing process builds on the previous step, beginning with assessment or data collection. Before the nurse can formulate a plan of action, implement a nursing intervention, or notify the provider of a change in the client's status, she must first collect adequate data from the client. Assessing or collecting additional data will provide the nurse with knowledge to make an appropriate decision. Therefore, the nurse should determine the client's level of pain prior to the procedure to determine the need for administration of an analgesic. Medicating the client approximately 30 min prior to wound care will decrease pain and increase her comfort. Place a waterproof pad under the client's extremity. The nurse should place a waterproof pad under the client's extremity to protect the linens from moisture and contamination during the irrigation; however, there is another action the nurse should take first. RN Learning System Medical-Surgical: Final Quiz

A nurse is caring for a client who has continuous bladder irrigation following a transurethral resection of the prostate (TURP). Which of the following findings should the nurse report to the provider? Output equal to the instilled irrigant Report of bladder spasms Viscous urinary output with clots Report of a strong urge to urinate

Output equal to the instilled irrigant The nurse should recognize that the amount of output in the urinary drainage bag should equal the amount of instilled irrigant. The nurse should monitor this amount to ensure the client does not experience overdistention of the bladder due to an obstruction. Report of bladder spasms The nurse should recognize that bladder spasms are an expected manifestation after a TURP and are usually controlled by a smooth muscle relaxant such as oxybutynin. ✅Viscous urinary output with clots The nurse should report urine output that is bright red with clots or urine that resembles ketchup to the provider because this is an indication of arterial bleeding. Report of a strong urge to urinate The nurse should recognize that a client sensation of a strong urge to urinate is an expected manifestation after a TURP due to the pressure of the balloon on the bladder sphincter. RN Learning System Medical-Surgical: Final Quiz

A nurse is caring for a client who has encephalitis due to West Nile virus. Which of the following actions should the nurse take? (Select all that apply.) Place the client on respiratory isolation. Monitor vital signs every 2 hr. Assess neurological status every 4 hr. Maintain the client in a modified Trendelenburg position. Keep the client's room darkened.

Place the client on respiratory isolation is incorrect. West Nile virus is an arbovirus that is transmitted to humans after a person is bitten by an infected organism, such as a mosquito. The nurse should follow standard precautions when caring for a client who has encephalitis due to West Nile virus. ✅Monitor vital signs every 2 hr is correct. The nurse should monitor the client's vital signs to assess for changes consistent with increased intracranial pressure. ✅Assess neurological status every 4 hr is correct. The nurse should monitor the client's neurological status at least every 4 hr, or more frequently if the client's status indicates. The course of encephalitis is unpredictable, so the client should be monitored closely for any indications of deteriorating neurological functioning. Maintain the client in a modified Trendelenburg position is incorrect. The client who has encephalitis is at risk for increased intracranial pressure; therefore, the nurse should maintain the head of the client's bed at 30 to 45 degrees. ✅Keep the client's room darkened is correct. The nurse should provide the client with a low-stimulation environment to promote comfort and decrease agitation. RN Learning System Medical-Surgical: Final Quiz

American Nurses Association (ANA) Standards of Practice: Standard 1, Assessment

The registered nurse collects comprehensive data pertinent to the health care consumer's health and/or the situation. ⏺Measurement Criteria The registered nurse: ❑Collects comprehensive data including but not limited to demographics, social determinants of health, health disparities, and physical, functional, psychosocial, emotional, cognitive, sexual, cultural, age-related, environmental, spiritual/transpersonal, and economic assessments in a systematic and ongoing process with compassion and respect for the inherent dignity, worth, and unique attributes of every person. ❑Recognizes the importance of the assessment parameters identified by World Health Organization (WHO), Healthy People 2020, or other organizations that influence nursing practice. ❑Integrates knowledge from global and environmental factors into the assessment process. ❑Elicits the health care consumer's values, preferences, expressed needs, and knowledge of the health care situation. ❑Recognizes the impact of his or her own personal attitudes, values, and beliefs on the assessment process. ❑Identifies barriers to effective communication based on psychosocial, literacy, financial, and cultural considerations. ❑Assesses the impact of family dynamics on health care consumer health and wellness. ❑Engages the health care consumer and other interprofessional team members in holistic, culturally sensitive data collection. ❑Prioritizes this collection based on the health care consumer's immediate condition or the anticipated needs of the health care consumer or situation. ❑Uses evidence-based assessment techniques, instruments, tools, available data, information, and knowledge relevant to the situation to identify patterns and variances. ❑Applies ethical, legal, and privacy guidelines and policies to the collection, maintenance, use, and dissemination of data and information. ❑Recognizes the health care consumer as the authority on her or his own health by honoring the person's care preferences. ❑Documents relevant data accurately and in a manner accessible to the interprofessional team. Jarvis CHAPTER 14 Assessing

6. The nurse collects objective and subjective data when conducting patient assessments. Which patient situations are examples of subjective data? Select all that apply. A patient tells the nurse that she is feeling nauseous. A patient's ankles are swollen. A patient tells the nurse that she is nervous about her test results. A patient complains that the skin on her arms is tingling. A patient rates his pain as a 7 on a scale of 1 to 10. A patient vomits after eating supper.

a, c, d, e. Subjective data are information perceived only by the affected person; these data cannot be perceived or verified by another person. Examples of subjective data are feeling nervous, nauseated, tingling, and experiencing pain. Objective data are observable and measurable data that can be seen, heard, or felt by someone other than the person experiencing them. Examples of objective data are an elevated temperature reading (e.g., 101°F), edema, and vomiting. Jarvis CHAPTER 14 Assessing

5.The nurse uses blended competencies when caring for patients in a rehabilitation facility. Which examples of interventions involve cognitive skills? Select all that apply. A. The nurse uses critical thinking skills to plan care for a patient. B. The nurse correctly administers IV saline to a patient who is dehydrated. C. The nurse assists a patient to fill out an informed consent form. D. The nurse learns the correct dosages for patient pain medications. E. The nurse comforts a mother whose baby was born with Down syndrome. F. The nurse uses the proper procedure to catheterize a female patient.

a, d. Using critical thinking and learning medication dosages are cognitive competencies. Performing procedures correctly is a technical skill, helping a patient with an informed consent form is a legal/ethical issue, and comforting a patient is an interpersonal skill. Jarvis CHAPTER 13 Blended Competencies, Clinical Reasoning, and Processes of Person-Centered Care

7. When a nurse enters the patient's room to begin a nursing history, the patient's wife is there. After introducing herself to the patient and his wife, what should the nurse do? Thank the wife for being present. Ask the wife if she wants to remain. Ask the wife to leave. Ask the patient if he would like the wife to stay.

d. The patient has the right to indicate whom he would like to be present for the nursing history and exam. The nurse should neither presume that he wants his wife there nor that he does not want her there. Similarly, the choice belongs to the patient, not the wife. Jarvis CHAPTER 14 Assessing

3. The nurse is admitting a 35-year-old pregnant woman to the hospital for treatment of preeclampsia. The patient asks the nurse: "Why are you doing a history and physical exam when the doctor just did one?" Which statements best explain the primary reasons a nursing assessment is performed? Select all that apply. "The nursing assessment will allow us to plan and deliver individualized, holistic nursing care that draws on your strengths." "It's hospital policy. I know it must be tiresome, but I will try to make this quick!" "I'm a student nurse and need to develop the skill of assessing your health status and need for nursing care." "We want to make sure that your responses to the medical exam are consistent and that all our data are accurate." "We need to check your health status and see what kind of nursing care you may need." "We need to see if you require a referral to a physician or other health care professional."

a, e, f. Medical assessments target data pointing to pathologic conditions, whereas nursing assessments focus on the patient's responses to health problems. The initial comprehensive nursing assessment results in baseline data that enable the nurse to make a judgment about a patient's health status, the ability to manage his or her own health care and the need for nursing. It also helps nurses plan and deliver individualized, holistic nursing care that draws on the patient's strengths and promotes optimum functioning, independence, and well-being, and enables the nurse to refer the patient to a physician or other health care professional, if indicated. The fact that this is hospital policy is a secondary reason, and although it may be true that a nurse may need to develop assessment skills, it is not the chief reason the nurse performs a nursing history and exam. The assessment is not performed to check the accuracy of the medical examination. Jarvis CHAPTER 14 Assessing

8. A nurse is assessing a patient who is diagnosed with anorexia. Following the assessment, the nurse recommends that the patient meet with a nutritionist. This action best exemplifies the use of: A. Clinical judgment B. Clinical reasoning C. Critical thinking D. Blended competencies

a. Although all the options refer to the skills used by nurses in practice, the best choice is clinical judgment as it refers to the result or outcome of critical thinking or clinical reasoning—in this case, the recommendation to meet with a nutritionist. Clinical reasoning usually refers to ways of thinking about patient care issues (determining, preventing, and managing patient problems). Critical thinking is a broad term that includes reasoning both outside and inside of the clinical setting. Blended competencies are the cognitive, technical, interpersonal, and ethical and legal skills combined with the willingness to use them creatively and critically when working with patients. Jarvis CHAPTER 13 Blended Competencies, Clinical Reasoning, and Processes of Person-Centered Care

4. An experienced nurse tells a beginning nurse not to bother studying too hard, since most clinical reasoning becomes "second nature" and "intuitive" once you start practicing. What thinking below should underlie the beginning nurse's response? A. Intuitive problem solving comes with years of practice and observation, and novice nurses should base their care on scientific problem solving. B. For nursing to remain a science, nurses must continue to be vigilant about stamping out intuitive reasoning. C. The emphasis on logical, scientific, evidence-based reasoning has held nursing back for years; it is time to champion intuitive, creative thinking! D. It is simply a matter of preference; some nurses are logical, scientific thinkers, and some are intuitive, creative thinkers.

a. Beginning nurses must use nursing knowledge and scientific problem solving as the basis of care they give; intuitive problem solving comes with years of practice and observation. If the beginning nurse has an intuition about a patient, that information should be discussed with the faculty member, preceptor, or supervisor. Answer b is incorrect because there is a place for intuitive reasoning in nursing, but it will never replace logical, scientific reasoning. Critical thinking is contextual and changes depending on the circumstances, not on personal preference. Jarvis CHAPTER 13 Blended Competencies, Clinical Reasoning, and Processes of Person-Centered Care

10. A student nurse tells the instructor that a patient is fine and has "no complaints." What would be the instructor's best response? "You made an inference that she is fine because she has no complaints. How did you validate this?" "She probably just doesn't trust you enough to share what she is feeling. I'd work on developing a trusting relationship." "Sometimes everyone gets lucky. Why don't you try to help another patient?" "Maybe you should reassess the patient. She has to have a problem—why else would she be here?"

a. The instructor is most likely to challenge the inference that the patient is "fine" simply because she is telling you that she has no problems. It is appropriate for the instructor to ask how the student nurse validated this inference. Jumping to the conclusion that the patient does not trust the student nurse is premature and is an invalidated inference. Answer c is wrong because it accepts the invalidated inference. Answer d is wrong because it is possible that the condition is resolving. Jarvis CHAPTER 14 Assessing

6. A nurse uses critical thinking skills to focus on the care plan of an older adult who has dementia and needs placement in a long-term care facility. Which statements describe characteristics of this type of critical thinking applied to clinical reasoning? Select all that apply. A. It functions independently of nursing standards, ethics, and state practice acts. B. It is based on the principles of the nursing process, problem solving, and the scientific method. C. It is driven by patient, family, and community needs as well as nurses' needs to give competent, efficient care. D. It is not designed to compensate for problems created by human nature, such as medication errors. E. It is constantly re-evaluating, self-correcting, and striving for improvement. F. It focuses on the big picture rather than identifying the key problems, issues, and risks involved with patient care.

b, c, e. Critical thinking applied to clinical reasoning and judgment in nursing practice is guided by standards, policies and procedures, and ethics codes. It is based on principles of nursing process, problem solving, and the scientific method. It carefully identifies the key problems, issues, and risks involved, and is driven by patient, family, and community needs, as well as nurses' needs to give competent, efficient care. It also calls for strategies that make the most of human potential and compensate for problems created by human nature. It is constantly re-evaluating, self-correcting, and striving to improve Jarvis CHAPTER 13 Blended Competencies, Clinical Reasoning, and Processes of Person-Centered Care

10. A nurse working in a community health clinic writes nursing diagnoses for patients and their families. Which nursing diagnoses are correctly written as three-part nursing diagnoses? 1. Disabled Family Coping related to lack of knowledge about home care of child on ventilator 2. Imbalanced Nutrition: Less Than Body Requirements related to inadequate caloric intake while striving to excel in gymnastics as evidenced by 20-lb weight loss since beginning the gymnastic program, and greatly less than ideal body weight when compared to standard height-weight charts 3. Need to learn how to care for child on ventilator at home related to unexpected discharge of daughter after 3-month hospital stay as evidenced by repeated comments "I cannot do this," "I know I'll harm her because I'm not a nurse," and "I can't do medical things" 4. Spiritual Distress related to inability to accept diagnosis of terminal illness as evidenced by multiple comments such as "How could God do this to me?" "I don't deserve this," "I don't understand. I've tried to live my life well," and "How could God make me suffer this way?" 5. Caregiver Role Strain related to failure of home health aides to appropriately diagnose needs of family caregivers and initiate a plan to facilitate coping as evidenced by caregiver's loss of weight and clinical depression (1) and (3) (2) and (4) (1), (2), and (3) (1), (2), (3), (4), and (5)

b. (1) is a two-part diagnosis, (3) is written in terms of needs and not an unhealthy response, and (5) is a legally inadvisable statement which blames home health aides for the patient's problem. Statements that may be interpreted as libel or that imply nursing negligence are legally hazardous to all the nurses caring for the patient. Assigning blame in the written record is problematic. CHAPTER 15 Diagnosing

8. A nurse is performing an initial comprehensive assessment of a patient admitted to a long-term care facility from home. The nurse begins the assessment by asking the patient, "How would you describe your health status and well-being?" The nurse also asks the patient, "What do you do to keep yourself healthy?" Which model for organizing data is this nurse following? Maslow's human needs Gordon's functional health patterns Human response patterns Body system model

b. Gordon's functional health patterns begin with the patient's perception of health and well-being and progress to data about nutritional-metabolic patterns, elimination patterns, activity, sleep/rest, self-perception, role relationship, sexuality, coping, and values/beliefs. Maslow's model is based on the human needs hierarchy. Human responses include exchanging, communicating, relating, valuing, choosing, moving, perceiving, knowing, and feeling. The body system model is based on the functioning of the major body systems. Jarvis CHAPTER 14 Assessing

3. The nursing process ensures that nurses are person centered rather than task centered. Rather than simply approaching a patient to take vital signs, the nurse thinks, "How is Mrs. Barclay today? Are our nursing actions helping her to achieve her goals? How can we better help her?" This demonstrates which characteristic of the nursing process? A. Systematic B. Interpersonal C. Dynamic D. Universally applicable in nursing situations

b. Interpersonal. All of the other options are characteristics of the nursing process, but the conversation and thinking quoted best illustrates the interpersonal dimension of the nursing process. Jarvis CHAPTER 13 Blended Competencies, Clinical Reasoning, and Processes of Person-Centered Care

5. A student nurse attempts to perform a nursing history for the first time. The student nurse asks the instructor how anyone ever learns all the questions the nurse must ask to get good baseline data. What would be the instructor's best reply? "There's a lot to learn at first, but once it becomes part of you, you just keep asking the same questions over and over in each situation until you can do it in your sleep!" "You make the basic questions a part of you and then learn to modify them for each unique situation, asking yourself how much you need to know to plan good care." "No one ever really learns how to do this well because each history is different! I often feel like I'm starting afresh with each new patient." "Don't worry about learning all of the questions to ask. Every facility has its own assessment form you must use."

b. Once a nurse learns what constitutes the minimum data set, it can be adapted to any patient situation. It is not true that each assessment is the same even when using the same minimum data set, nor is it true that each assessment is uniquely different. Nurses committed to thoughtful, person-centered practice tailor their questions to the uniqueness of each patient and situation. Answer d is incorrect because relying solely on standard facility assessment tools does not allow for individualized patient care or critical thinking. Jarvis CHAPTER 14 Assessing

10. The nurse practices using critical thinking indicators (CTIs) when caring for patients in the hospital setting. The best description of CTIs is: A. Evidence-based descriptions of behaviors that demonstrate the knowledge that promotes critical thinking in clinical practice B. Evidence-based descriptions of behaviors that demonstrate the knowledge and skills that promote critical thinking in clinical practice C. Evidence-based descriptions of behaviors that demonstrate the knowledge, characteristics, and skills that promote critical thinking in clinical practice D. Evidence-based descriptions of behaviors that demonstrate the knowledge, characteristics, standards, and skills that promote critical thinking in clinical practice

c. Evidence-based descriptions of behaviors that demonstrate the knowledge, characteristics, and skills that promote critical thinking in clinical practice. Jarvis CHAPTER 13 Blended Competencies, Clinical Reasoning, and Processes of Person-Centered Care

7. A nurse is caring for a patient who has complications related to type 2 diabetes mellitus. The nurse researches new procedures to care for foot ulcers when developing a care plan for this patient. Which QSEN competency does this action represent? A.Patient-centered care B. Evidence-based practice C. Quality improvement D. Informatics

c. Quality improvement involves routinely updating nursing policies and procedures. Providing patient-centered care involves listening to the patient and demonstrating respect and compassion. Evidence-based practice is used when adhering to internal policies and standardized skills. The nurse is employing informatics by using information and technology to communicate, manage knowledge, and support decision making. Jarvis CHAPTER 13 Blended Competencies, Clinical Reasoning, and Processes of Person-Centered Care

2. A female patient who is receiving chemotherapy for breast cancer tells the nurse, "The treatment for this cancer is worse than the disease itself. I'm not going to come for my therapy anymore." The nurse responds by using critical thinking skills to address this patient problem. Which action is the first step the nurse would take in this process? A. The nurse judges whether the patient database is adequate to address the problem. B. The nurse considers whether or not to suggest a counseling session for the patient. C. The nurse reassesses the patient and decides how best to intervene in her care. D. The nurse identifies several options for intervening in the patient's care and critiques the merit of each option.

c. The first step when thinking critically about a situation is to identify the purpose or goal of your thinking. Reassessing the patient helps to discipline thinking by directing all thoughts toward the goal. Once the problem is addressed, it is important for the nurse to judge the adequacy of the knowledge, identify potential problems, use helpful resources, and critique the decision. Jarvis CHAPTER 13 Blended Competencies, Clinical Reasoning, and Processes of Person-Centered Care

9. The nurse is surprised to detect an elevated temperature (102°F) in a patient scheduled for surgery. The patient has been afebrile and shows no other signs of being febrile. What is the priority nursing action? Inform the charge nurse. Inform the surgeon. Validate the finding. Document the finding.

c. The nurse should first validate the finding if it is unusual, deviates from normal, and is unsupported by other data. Should the initial recording prove to be in error, it would have been premature to notify the charge nurse or the surgeon. The nurse should be sure that all data recorded are accurate; thus, all data should be validated before documentation if there are any doubts about accuracy. Jarvis CHAPTER 14 Assessing

9.A nurse is writing nursing diagnoses for patients in a psychiatrist's office. Which nursing diagnoses are correctly written as two-part nursing diagnoses? Ineffective Coping related to inability to maintain marriage Defensive Coping related to loss of job and economic security Altered Thought Processes related to panic state Decisional Conflict related to placement of parent in a long-term care facility (1) and (2) (3) and (4) (1), (2), and (3) (1), (2), (3), and (4)

d. Each of the four diagnoses is a correctly written two-part diagnostic statement that includes the problem or diagnostic label and the etiology or cause. CHAPTER 15 Diagnosing

4. A nurse notes that a shift report states that a patient has no special skin care needs. The nurse is surprised to observe reddened areas over bony prominences during the patient bath. What nursing action is appropriate? Correct the initial assessment form. Redo the initial assessment and document current findings. Conduct and document an emergency assessment. Perform and document a focused assessment of skin integrity.

d. Perform and document a focused assessment on skin integrity since this is a newly identified problem. The initial assessment stands as is and cannot be redone or corrected. This is not a life-threatening event; therefore, there is no need for an emergency assessment. Jarvis CHAPTER 14 Assessing

2. The nurse practitioner is performing a short assessment of a newborn who is displaying signs of jaundice. The nurse observes the infant's skin color and orders a test for bilirubin levels to report to the primary care provider. What type of assessment has this nurse performed? Comprehensive Initial Time-lapsed Quick priority

d. Quick priority assessments (QPAs) are short, focused, prioritized assessments nurses do to gain the most important information they need to have first. The comprehensive initial assessment is performed shortly after the patient is admitted to a health care facility or service. The time-lapsed assessment is scheduled to compare a patient's current status to baseline data obtained earlier. Jarvis CHAPTER 14 Assessing

9. A nurse working in a long-term care facility bases patient care on five caring processes: knowing, being with, doing for, enabling, and maintaining belief. This approach to patient care best describes whose theory? Travelbee's Watson's Benner's Swanson's

d. Swanson (1991) identifies five caring processes and defines caring as "a nurturing way of relating to a valued other toward whom one feels a personal sense of commitment and responsibility." Travelbee (1971), an early nurse theorist, developed the Human-to-Human Relationship Model, and defined nursing as an interpersonal process whereby the professional nurse practitioner assists an individual, family, or community to prevent or cope with the experience of illness and suffering, and if necessary to find meaning in these experiences. Benner and Wrubel (1989) wrote that caring is a basic way of being in the world, and that caring is central to human expertise, curing, and healing. Watson's theory is based on the belief that all humans are to be valued, cared for, respected, nurtured, understood, and assisted. Jarvis CHAPTER 13 Blended Competencies, Clinical Reasoning, and Processes of Person-Centered Care

Theories of Caring BOX 13-2 Reflective Practice: Person-Centered Care

⏺1. All team members are considered caregivers. 🔘Identify the team members you worked with while implementing the plan of care. Everyone in the workforce from housekeeping staff to the CEO is part of the patient's plan of care. 🔘Next to each describe what they contributed to person-centered care for your patient. 🔘Star or highlight those with whom you had meaningful conversations and briefly describe what you learned ⏺2. Care is based on continuous healing relationships. 🔘Describe what you did to make your interactions with the patient/family qualify as a healing relationship versus a mere business transaction or provision of a service. 🔘This principle reinforces a focus on the continuum of care. In what ways did your care anticipate and prepare your patient and family for the days ahead. ⏺3. Care is customized and reflects patient needs, values, and choices. 🔘Describe what you learned about this patient/family as unique individuals (strengths, needs, values, and preferences) that allowed you to creatively individualize the care plan. 🔘What strategies did you use to elicit the patient's needs, values, and choices? 🔘In what specific ways is your care plan different from any standardized care plan? ⏺4. Knowledge and information are freely shared between and among patients, care partners, health care providers, and caregivers. 🔘Describe one way you shared information with the patient/family and one way you shared information with the health care team that enhanced the partnership and the care plan. 🔘What did you learn from other members of the team that enhanced your ability to care for this patient/family? Jarvis CHAPTER 13 Blended Competencies, Clinical Reasoning, and Processes of Person-Centered Care

THE PROFESSIONAL NURSE TABLE 13-1 Comparison of Benner's The Helping Role of the Nurse With Swanson's Caring Process and Watson's Carative Factors

⏺BENNER: THE HELPING ROLE OF THE NURSE • Creating a climate for establishing a commitment to healing • Providing comfort measures and preserving personhood in the face of pain and extreme breakdown • Presencing • Maximizing the patient's participation and control in the patient's recovery ⏺SWANSON'S CARING PROCESS • Knowing: avoiding assumptions, centering on the one cared for, assessing thoroughly, seeking cues, engaging the self of both • Being with: being there, conveying ability, sharing feelings, not burdening • Doing for: comforting, anticipating, performing competently/skillfully, protecting, preserving dignity • Enabling: informing/explaining, supporting/allowing, focusing, generating alternatives/thinking it through, validating/giving feedback • Maintaining belief: believing in/holding in esteem, maintaining a hope-filled attitude, offering realistic optimism, "going the distance" ⏺WATSON'S CARATIVE FACTORS • Humanistic altruistic system of values • Instillation of faith/hope • Sensitivity to self and others • Helping/trusting human care relationship • Expressing positive and negative feelings • Creating problem-solving caring process • Transpersonal teaching/learning • Supportive, protective, and/or corrective mental, physical, societal, and spiritual environment • Human needs assistance • Existential-phenomenologic-spiritual forces Jarvis CHAPTER 13 Blended Competencies, Clinical Reasoning, and Processes of Person-Centered Care

1.

✅ CHAPTER 16 Outcome Identification and Planning

10.

✅ CHAPTER 16 Outcome Identification and Planning

2.

✅ CHAPTER 16 Outcome Identification and Planning

3.

✅ CHAPTER 16 Outcome Identification and Planning

4.

✅ CHAPTER 16 Outcome Identification and Planning

5.

✅ CHAPTER 16 Outcome Identification and Planning

6.

✅ CHAPTER 16 Outcome Identification and Planning

7.

✅ CHAPTER 16 Outcome Identification and Planning

8.

✅ CHAPTER 16 Outcome Identification and Planning

9.

✅ CHAPTER 16 Outcome Identification and Planning

A nurse is providing teaching to a client who has cervical cancer and is scheduled to receive brachytherapy in an ambulatory care clinic. Which of the following statements by the client indicates an understanding of the teaching? "I need to lie still in bed during my brachytherapy treatment." "I will have an implant placed once a month during my brachytherapy treatment." "I must stay at least 3 feet away from others between brachytherapy treatments." "I should expect some blood in my urine after each brachytherapy treatment."

✅"I need to lie still in bed during my brachytherapy treatment." The nurse should confirm that the client understands the need to remain on bed rest with limited movement while the radioactive implant is in place to prevent dislodgment. "I will have an implant placed once a month during my brachytherapy treatment." The nurse should teach the client that the provider often prescribes brachytherapy treatments 1 to 2 times per week. "I must stay at least 3 feet away from others between brachytherapy treatments." The nurse should teach the client that she does not emit any radiation between treatments; therefore, there are no restrictions regarding contact with others. "I should expect some blood in my urine after each brachytherapy treatment." The nurse should teach the client that blood in the urine is not expected after brachytherapy treatment. The client should notify the provider immediately if she develops this manifestation. RN Learning System Medical-Surgical: Final Quiz

A nurse is providing discharge teaching to a client who has a new permanent pacemaker. Which of the following statements by the client indicates an understanding of the teaching? "I should check my heart rate at the same time each day." "I don't have to take my antihypertensive medications now that I have a pacemaker." "I should keep a pressure dressing over the generator until the incision is healed." "I cannot stand in front of our new microwave oven when it is on."

✅"I should check my heart rate at the same time each day." The nurse should instruct the client to check his heart rate at the same time each day and to document the rate in a log for communication with the provider. "I don't have to take my antihypertensive medications now that I have a pacemaker." The nurse should instruct the client that a pacemaker maintains a regular heart rate but is not intended to lower blood pressure or control hypertension. "I should keep a pressure dressing over the generator until the incision is healed." The nurse should instruct the client to avoid applying pressure over the generator. "I cannot stand in front of our new microwave oven when it is on." The nurse should instruct the client that new microwaves are equipped with shielding that protects a person who has a pacemaker from interference. Standing in front of a new microwave oven is not contraindicated. The client should avoid being in close proximity to older microwaves that do not have this shielding. RN Learning System Medical-Surgical: Final Quiz

A nurse is caring for a client who has burn injuries to his trunk. The nurse is explaining what to expect from the prescribed hydrotherapy. Which of the following statements by the client indicates an understanding of the teaching? "I will be on a special shower table." "The water temperature will be very cool to ease my pain." "The nurse will use a firm-bristled brush to remove loose skin." "The nurse will use scissors to open small blisters."

✅"I will be on a special shower table." The special shower table facilitates examination and debridement of the wound during hydrotherapy. An advantage of using the showering technique as opposed to a tub bath is that the water can be kept at a constant temperature and there is a lower risk of wound infection. "The water temperature will be very cool to ease my pain." The nurse should use warm water during the hydrotherapy treatment to help the client maintain adequate body temperature. "The nurse will use a firm-bristled brush to remove loose skin." The nurse should use soft washcloths or gauze to gently scrub and debride the wounds. "The nurse will use scissors to open small blisters." The nurse should leave small blisters intact; however, she will open large blisters. RN Learning System Medical-Surgical: Dermatological Practice Quiz

A nurse is providing teaching to a client who is scheduled for a sigmoid colon resection with colostomy. Which of the following statements by the client indicates a need for further teaching? "Because most of my colon is still intact and functioning, my stool will be formed." "My stoma will appear large at first, but it will shrink over the next several weeks." "My colostomy will begin to function 2 to 6 days after surgery." "My diet will have to change to a soft diet after surgery."

✅"My diet will have to change to a soft diet after surgery." The nurse should identify that this statement requires further teaching. After surgery the client's diet quickly returns to a regular diet and there are not any food restrictions, unless the client chooses to decrease intake of foods that increase gas or odor. "Because most of my colon is still intact and functioning, my stool will be formed." The nurse should inform the client that a colostomy placed at the descending or sigmoid colon produces stool that is fairly solid and resembles that which is normally expelled from the rectum; therefore, this statement does not require further teaching. "My stoma will appear large at first, but it will shrink over the next several weeks." The nurse should inform the client that the stoma is edematous at first because of trauma from surgery and manipulation of the colon, but that it will shrink within 6 to 8 weeks after surgery as the edema decreases; therefore, this statement does not require further teaching. "My colostomy will begin to function 2 to 6 days after surgery." The nurse should inform the client that because of the lack of bowel peristalsis after surgery and the client's NPO status, it is not unusual to see only mucus drain from the ostomy until 2 to 6 days after surgery; therefore, this statement does not require further teaching. RN Learning System Medical-Surgical: Final Quiz

A nurse in a provider's office is assessing a client's skin lesions. The nurse notes that the lesions are 0.5 cm (0.20 in) in size, elevated, and solid, with very distinct borders. The nurse should document the findings as which of the following skin lesions? Papules Macules Wheals Vesicles

✅Papules A papule is a small, solid, elevated lesion with distinct borders. It is usually smaller than 10 mm in diameter. Papules are common lesions of warts and elevated moles. Macules A macule is flat, variably shaped, discolored, and small, typically smaller than 10 mm in diameter. A macule is a change in the color of the skin. Freckles and the rash associated with rubella are types of macules. Wheals Wheals, also known as hives, are transient, elevated, irregularly shaped lesions caused by localized edema. Wheals are a common manifestation of an allergic reaction. Vesicles A vesicle is a circumscribed, elevated lesion or blister containing serous fluid. Vesicles typically arise with herpes simplex, poison ivy, and chickenpox. RN Learning System Medical-Surgical: Dermatological Practice Quiz

A nurse is caring for a client who is in hypovolemic shock. While waiting for a unit of blood, the nurse should administer which of the following IV solutions? 0.45% sodium chloride Dextrose 5% in 0.9% sodium chloride Dextrose 10% in water 0.9% sodium chloride

✅0.9% sodium chloride Solutions of 0.9% sodium chloride, as well as Lactated Ringer's solution, are used for fluid volume replacement. Sodium chloride, a crystalloid, is a physiologic isotonic solution that replaces lost volume in the blood stream and is the only solution to use when infusing blood products. 0.45% sodium chloride The solution 0.45% sodium chloride is a hypotonic solution and should not be used for fluid replacement. This solution can cause lysis of red blood cells because it has fewer solutes than the cell, and osmotic pressure pulls the fluid into the few cells remaining. Dextrose 5% in 0.9% sodium chloride The solution of dextrose 5% in 0.9% sodium chloride is a hypertonic solution and should not be used for fluid replacement. This solution will diffuse into the cells of the tissue, having no effect on circulating volume. When the fluid surrounding the cells is hypertonic or has more solutes than the cells, osmotic pressure pulls the fluid from the cells. Dextrose 10% in water The solution of dextrose 10% in water is a hypertonic solution and should not be used for fluid replacement. This solution will diffuse into the cells of the tissue, having no effect on circulating volume. When the fluid surrounding the cells is hypertonic or has more solutes than the cells, osmotic pressure pulls the fluid from the cells. RN Learning System Medical-Surgical: Cardiovascular and Hematology Practice Quiz

A nurse is caring for a client who has an abdominal aortic aneurysm and is scheduled for surgery. The client's vital signs are blood pressure 160/98 mm Hg, heart rate 102/min, respirations 22/min, and SpO2 95%. Which of the following actions should the nurse take? Administer antihypertensive medication for blood pressure. Monitor that urinary output is 20 mL/hr. Withhold pain medication to prepare for surgery. Take vital signs every 2 hr.

✅Administer antihypertensive medication for blood pressure. The nurse should administer antihypertensive medication for the elevated blood pressure because hypertension can cause a sudden rupture of the aneurysm due to pressure on the arterial wall. Monitor that urinary output is 20 mL/hr. The nurse should monitor that the client has adequate kidney profusion determined by urinary output of at least 30 mL/hr. Oliguria can indicate a rupture of the aneurysm. Withhold pain medication to prepare for surgery. The nurse should administer pain medication because pain occurs due to pressure from the aneurysm on the lumbar nerves. Pain can also cause hypertension. Take vital signs every 2 hr. The nurse should take the client's vital signs at least every 15 min in order to monitor for a sudden drop in blood pressure, which can indicate a rupture of the aneurysm. RN Learning System Medical-Surgical: Cardiovascular and Hematology Practice Quiz

A nurse is caring for a client who has thrombocytopenia and develops epistaxis. Which of the following actions should the nurse take? Have the client gently blow clots from the nose every 5 min. Instruct the client to sit with his head hyperextended. Apply ice compresses to the back of the client's neck. Apply lateral pressure to the client's nose for 10 min.

✅Apply lateral pressure to the client's nose for 10 min. The nurse should apply direct, lateral pressure to the nose for 10 min to control epistaxis. If after 10 min the epistaxis continues, the client might require nasal packing or other interventions. Have the client gently blow clots from the nose every 5 min. The nurse should instruct the client to refrain from blowing his nose for 24 hr after the epistaxis stops. The formation of clots is the mechanism that will terminate the nosebleed. Having the client blow his nose will dislodge any clots that do form and cause the bleeding to continue. Instruct the client to sit with his head hyperextended. The nurse should place the client in a sitting position, leaning forward. If the client positions his head and neck backward, it will allow blood to drain into the stomach, causing nausea and vomiting. Apply ice compresses to the back of the client's neck. The nurse should apply an ice pack or cool compress to the client's nose and face to help control epistaxis. RN Learning System Medical-Surgical: Final Quiz

A nurse in an emergency department is caring for a client who reports developing severe right eye pain with a gritty sensation while sawing wood. Which of the following actions should the nurse take first? Instill proparacaine hydrochloride eyedrops. Perform ocular irrigation of the right eye. Place the client in a supine position with the head turned toward the affected side. Ask the client about first aid performed at the scene.

✅Ask the client about first aid performed at the scene. The nurse should apply the nursing process priority-setting framework. The nurse can use the nursing process to plan client care and prioritize nursing actions. Each step of the nursing process builds on the previous step, beginning with assessment or data collection. Before the nurse can formulate a plan of action, implement a nursing intervention, or notify the provider of a change in the client's status, she must first collect adequate data from the client. Assessing or collecting additional data will provide the nurse with knowledge to make an appropriate decision. Therefore, the first action the nurse should take is to assess the first aid that was performed at the scene to determine if eye irrigation was administered. Instill proparacaine hydrochloride eyedrops. The nurse should instill proparacaine hydrochloride eyedrops, after assessing for client allergies, to promote relief of eye pain; however, there is another action that the nurse should take first. Perform ocular irrigation of the right eye. The nurse should prepare for and quickly perform ocular irrigation when a foreign body in the eye is suspected; however, there is another action that the nurse should take first. Place the client in a supine position with the head turned toward the affected side. The nurse should place the client in a supine position with the head turned toward the affected eye to promote drainage of irrigation fluid during ocular irrigation; however, there is another action that the nurse should take first. RN Learning System Medical-Surgical: Final Quiz

A nurse in the emergency department is caring for a client who has fruity breath odor, dry mouth, and extreme thirst. Which of the following assessments should the nurse make? Blood glucose level Pupillary reaction to light Deep tendon reflexes Liver function tests

✅Blood glucose level The nurse should identify that these findings are indications of hyperglycemia and diabetic ketoacidosis. The nurse should therefore check the client's blood glucose level. The nurse should also assess the client's respiratory status, vital signs, level of consciousness, and hydration status, including a laboratory assessment of his electrolyte levels. Pupillary reaction to light The nurse should identify the probable cause of the client's manifestations and perform a focused assessment of the affected system; therefore, it is not necessary for the nurse to assess the client's pupillary reaction to light. Deep tendon reflexes The nurse should identify the probable cause of the client's manifestations and perform a focused assessment of the affected system; therefore, it is not necessary for the nurse to assess the client's deep tendon reflexes. Liver function tests The nurse should identify the probable cause of the client's manifestations and perform a focused assessment of the affected system; therefore, it is not necessary for the nurse to assess the client's liver function. RN Learning System Medical-Surgical: Final Quiz

A nurse is completing an assessment for a client who has a history of unstable angina. Which of the following findings should the nurse expect? Chest pain is relieved soon after resting. Nitroglycerin relieves chest pain. Physical exertion does not precipitate chest pain. Chest pain lasts longer than 15 min.

✅Chest pain lasts longer than 15 min. The client who has unstable angina will have chest pain lasting longer than 15 min. This is due to the reduced blood flow in a coronary artery due to atherosclerotic plaque and thrombus formation causing partial arterial obstruction, or from an artery spasm. Chest pain is relieved soon after resting. The client who has unstable angina will have chest pain even while resting because of insufficient blood flow to the coronary arteries and decreased oxygen supply. Chest pain at rest is a condition called variant (Prinzmetal's) angina, caused by an artery spasm. Nitroglycerin relieves chest pain. The client who has unstable angina will have minimal, if any, relief of chest pain from nitroglycerin. This is due to the reduced blood flow in a coronary artery due to atherosclerotic plaque and thrombus formation causing partial arterial obstruction. Physical exertion does not precipitate chest pain. The client who has unstable angina will report chest pain or discomfort with exertion, which can limit the client's activity. This is due to the reduced blood flow in a coronary artery due to atherosclerotic plaque and thrombus formation causing partial arterial obstruction. RN Learning System Medical-Surgical: Cardiovascular and Hematology Practice Quiz

A nurse is teaching about a low-cholesterol diet to a client who had a myocardial infarction. Which of the following meal selections by the client indicates an understanding of the teaching? Chicken breast and corn on the cob Shrimp and rice Cheese omelet and turkey bacon Liver and onions

✅Chicken breast and corn on the cob The nurse should identify that chicken breast is low in cholesterol and all vegetables, including corn, are cholesterol free; therefore, this food selection by the client indicates an understanding of the teaching. Shrimp and rice The nurse should inform the client that shrimp are high in cholesterol and should be eaten in moderation; therefore, this food selection does not indicate an understanding of a low-cholesterol diet. Cheese omelet and turkey bacon The nurse should inform the client that eggs and cheese are high in cholesterol; therefore, this food selection does not indicate an understanding of a low-cholesterol diet. Liver and onions The nurse should inform the client that liver and other organ meats are high in cholesterol; therefore, this food selection does not indicate an understanding of a low-cholesterol diet. RN Learning System Medical-Surgical: Final Quiz

5. A nurse is counseling a patient who refuses to look at or care for a new colostomy. The patient tells the nurse, "I don't care what I look like anymore, I don't even feel like washing my hair, let alone changing this bag." The nurse diagnoses Altered Health Maintenance. This is an example of what type of problem? Collaborative problem Interdisciplinary problem Medical problem Nursing problem

✅D. Altered Health Maintenance is a nursing problem, because the diagnosis describes a problem that can be treated by nurses within the scope of independent nursing practice. Collaborative and interdisciplinary problems require a teamwork approach with other health care professionals to resolve the problem. A medical problem is a traumatic or disease condition validated by medical diagnostic studies. CHAPTER 15 Diagnosing

A nurse is providing postoperative care for a client who has two chest tubes in place following a lobectomy. The client asks the nurse the reason for having two chest tubes. The nurse should inform the client that the lower chest tube is placed for which of the following reasons? Removing air from the pleural space Creating access for irrigating the chest cavity Evacuating secretions from the bronchioles and alveoli Draining blood and fluid from the pleural space

✅Draining blood and fluid from the pleural space The nurse should inform the client that blood and fluids tend to accumulate in the bases and posterior areas of the pleural cavity following a lobectomy. For this reason, the lower chest tube primarily drains blood and fluid from the pleural space Removing air from the pleural space The nurse should inform the client that the upper chest tube is the one that removes air from the pleural space. Creating access for irrigating the chest cavity The nurse should be aware that the chest tubes are not used for irrigation following a lobectomy. Evacuating secretions from the bronchioles and alveoli The nurse should be aware that secretions are removed from the airways via tracheal suctioning rather than chest tubes. RN Learning System Medical-Surgical: Final Quiz

A nurse is planning care for a client during a sickle cell crisis. Which of the following interventions should the nurse include in the client's plan of care? Maintain the client's knees and hips in a flexed position. Apply cold compresses to painful joints. Withhold opioids until the crisis is resolved. Encourage increased fluid intake.

✅Encourage increased fluid intake. The nurse should encourage increased fluid intake to promote hydration because dehydration increases the viscosity of the blood, which can increase sickling and client discomfort. Maintain the client's knees and hips in a flexed position. The nurse should avoid flexion of the client's knees and hips during a sickle cell crisis to promote adequate perfusion to all areas of the client's body, which can decrease pain. Apply cold compresses to painful joints. The nurse should keep the room warm during a sickle cell crisis and apply warm, moist compresses to painful joints. The application of cold compresses causes vasoconstriction, which in turn increases sickling. Withhold opioids until the crisis is resolved. The nurse should administer opioids, including morphine and hydromorphone, on a routine schedule during a crisis to manage the client's pain. RN Learning System Medical-Surgical: Final Quiz

A nurse is assessing a client for manifestations of aplastic anemia. Which of the following findings should the nurse expect? Plethoric appearance of facial skin Glossitis and weight loss Jaundice with an enlarged liver Petechiae and ecchymosis

✅Petechiae and ecchymosis The client who has aplastic anemia will have manifestations of petechiae and ecchymosis. Dyspnea on exertion also can be present. In aplastic anemia, all three major blood components (red blood cells, white blood cells, and platelets) are reduced or absent, which is known as pancytopenia. Manifestations usually develop gradually. Plethoric appearance of facial skin The client who has polycythemia vera will have a plethoric (dark, flushed) manifestation of the facial skin and mucous membranes. Glossitis and weight loss The client who has pernicious anemia will have manifestation of glossitis (smooth, beefy-red tongue) and weight loss. Jaundice with an enlarged liver The client who has sickle cell anemia will have manifestations of jaundice with an enlarged liver and spleen. RN Learning System Medical-Surgical: Cardiovascular and Hematology Practice Quiz

A nurse is assesing a client who is bedridden and was admitted from home. The nurse notes a shallow crater in the epidermis of the client's sacral area. The nurse should document that the client has a pressure ulcer at which of the following stages? IV I III II

✅II With a stage II pressure ulcer, there is partial-thickness skin loss involving the epidermis and the dermis. The ulcer is visible and superficial and can appear as an abrasion, blister, or shallow crater. Edema persists, and the ulcer might become infected. The client might report pain, and there might be a small amount of drainage. IV With a stage IV pressure ulcer, the client has full-thickness tissue loss, with destruction, tissue necrosis, and visible damage to muscle, bone, or supporting structures. Sinus tracts, deep pockets of infection, tunneling, and undermining can occur. I With a stage I pressure ulcer, the skin is intact with an area of persistent, nonblanchable redness, usually over a bony prominence, that might feel warm or cool when touched. The tissue is swollen and congested, and the client might report discomfort at the site. With darker skin tones, the ulcer can appear blue or purple and different from other skin areas. III With a stage III pressure ulcer, there is full-thickness tissue loss with damage to or necrosis of subcutaneous tissue. The ulcer might extend down to, but not through, underlying fascia. The ulcer appears as a deep crater with or without undermining of adjacent tissue and without exposed muscle or bone. Drainage and infection are common. RN Learning System Medical-Surgical: Dermatological Practice Quiz

A nurse in the emergency department is caring for a client who has a snakebite on her arm. Which of the following interventions should the nurse implement? Immobilize the limb at the level of the heart. Apply a tourniquet to the affected limb. Use a sterile scapula to incise the wound. Apply ice to the skin over the snakebite wound.

✅Immobilize the limb at the level of the heart. The emergency management of a client who has a snakebite focuses on limiting the spread of venom. Any constrictive clothing or jewelry should be removed before swelling worsens, and the affected limb should be immobilized at the level of the heart. Apply a tourniquet to the affected limb. Although the use of tourniquets was recommended in the past for the emergency management of snakebites, this is no longer an accepted treatment and is now contraindicated. Use a sterile scapula to incise the wound. Although incising the wound was recommended in the past for the emergency management of snakebites, this is no longer an accepted treatment and is now contraindicated. Apply ice to the skin over the snakebite wound. Although the use of ice was recommended in the past for the emergency management of snakebites, this is no longer an accepted treatment and is now contraindicated. RN Learning System Medical-Surgical: Dermatological Practice Quiz

A nurse is assessing a client who has a lesion on his skin. Which of the following findings is a clinical manifestation of a malignant melanoma? Rough, dry, scaly lesion Firm nodule with crust Pearly papule with ulcerated center Irregularly shaped lesion with blue tones

✅Irregularly shaped lesion with blue tones Malignant melanomas are irregularly shaped and can be blue, red, or white in tone. They often occur on the client's upper back and lower legs. Rough, dry, scaly lesion This finding is a clinical manifestation of actinic keratosis. Firm nodule with crust This finding is a clinical manifestation of squamous cell carcinoma. Pearly papule with ulcerated center This finding is a clinical manifestation of basal cell carcinoma. RN Learning System Medical-Surgical: Dermatological Practice Quiz

A nurse is assessing a client who has fluid volume overload from a cardiovascular disorder. Which of the following manifestations should the nurse expect? (Select all that apply.) Jugular vein distension Moist crackles Postural hypotension Increased heart rate Fever

✅Jugular vein distension is correct. The increase in venous pressure due to excessive circulating blood volume results in neck vein distension. ✅Moist crackles is correct. This is an indicator of pulmonary edema that can quickly lead to death. Postural hypotension is incorrect. Fluid volume excess, or hypervolemia, is an expansion of fluid volume in the extracellular fluid compartment. This results in hypertension and tachycardia. ✅Increased heart rate is correct. Fluid volume excess, or hypervolemia, is an expansion of fluid volume in the extracellular fluid compartment. This results in increased heart rate and bounding pulses. Fever is incorrect. Fever is common in clients who are experiencing dehydration, not fluid volume excess. RN Learning System Medical-Surgical: Cardiovascular and Hematology Practice Quiz

A nurse is assessing a client who has an abdominal aortic aneurysm. Which of the following manifestations should the nurse expect? Midsternal chest pain Thrill Pitting edema in lower extremities Lower back discomfort

✅Lower back discomfort Abdominal aortic aneurysm involves a widening, stretching, or ballooning of the aorta. Back and abdominal pain indicate that the aneurysm is extending downward and pressing on lumbar spinal nerve roots, causing pain. Midsternal chest pain The nurse should assess for mid or lower abdominal pain to the left of the midline because of the enlarged artery mass. Thrill The nurse should auscultate for a bruit heard over the location of the mass. Pitting edema in lower extremities Pitting edema is a manifestation of heart failure. This is not an assessment the nurse should find with an abdominal aortic aneurysm. RN Learning System Medical-Surgical: Cardiovascular and Hematology Practice Quiz

A nurse is planning care for a client who had a stroke. The client has hemiplegia and occasional urinary incontinence. Which of the following actions should the nurse include in the client's plan of care? Offer the client a bedpan every 2 hr. Limit the client's daily fluid intake until he is no longer incontinent. Request a prescription for an indwelling urinary catheter from the client's provider. Ambulate the client to the bathroom every 30 min.

✅Offer the client a bedpan every 2 hr. Following a stroke, the client might have bladder incontinence due to confusion, impaired sensation in response to bladder fullness, and decreased sphincter control. The nurse should encourage and assist the client to void every 2 hr while awake to promote bladder control. By offering a bedpan, the nurse promotes client safety. Limit the client's daily fluid intake until he is no longer incontinent. The nurse should encourage adequate fluid intake of 2 to 3 L per day to promote urine production. The nurse should also plan actions that promote bladder training. Request a prescription for an indwelling urinary catheter from the client's provider. The nurse should avoid inserting an indwelling urinary catheter due to the increased risk for a health care-associated urinary tract infection. The nurse should instead plan actions that promote bladder training. Ambulate the client to the bathroom every 30 min. The nurse should promote client safety and plan actions that promote bladder training. Ambulating a client who has hemiplegia every 30 min increases the risk for falls. RN Learning System Medical-Surgical: Final Quiz

A nurse is caring for a client who has heart failure and whose telemetry reading displays a flattening of the T wave. Which of the following laboratory results should the nurse anticipate as the cause of this ECG change? Potassium 2.8 mEq/L Digoxin level 0.7 ng/mL Hemoglobin 9.8 g/dL Calcium 8.0 mg

✅Potassium 2.8 mEq/L A flattened T wave or the development of U waves is indicative of a low potassium level. Digoxin level 0.7 ng/mL The client has a digoxin level within the therapeutic range of 0.5 to 0.8 ng/mL. Atrioventricular block, ventricular fibrillation, and ventricular tachycardia are a few of the dysrhythmias occurring with toxic digoxin levels. Hemoglobin 9.8 g/dL The client who has low hemoglobin will manifest tachycardia on the ECG rhythm because of the compensatory mechanism that provides oxygen to vital organs. The ECG pattern anticipated with low hemoglobin is tachycardia. Calcium 8.0 mg The client who has hypocalcemia can have a prolonged S-T interval and a prolonged Q-T interval, but not a flattened T wave. RN Learning System Medical-Surgical: Cardiovascular and Hematology Practice Quiz

A nurse is reviewing a client's laboratory report. The client's ABG levels are pH 7.5, PaCO2 32 mm HG, and HCO3- 24 mEq/L. The nurse should determine that the client has which of the following acid-base imbalances? Respiratory alkalosis Metabolic acidosis Respiratory acidosis Metabolic alkalosis

✅Respiratory alkalosis The nurse should identify that the client's pH is elevated above the expected reference range of 7.35 to 7.45, indicating alkalosis. The nurse should then identify that the client's PaCO2 is low, below the expected reference range of 35 to 45 mm Hg, which indicates a respiratory origin. The nurse should conclude that the client's elevated pH and decreased PaCO2 indicate respiratory alkalosis. Metabolic acidosis The nurse should recognize that ABGs are drawn to determine acid-base balance in the arterial blood. Acidosis is determined by measuring a pH lower than the expected reference range of 7.35 to 7.45. This client has a pH of 7.5 and therefore does not have acidosis. Respiratory acidosis The nurse should recognize that ABGs are drawn to determine acid-base balance in the arterial blood. The client's pH is elevated above the expected reference range of 7.35 to 7.45. Acidosis is presented by a lower pH, usually below 7.35. Metabolic alkalosis The nurse should recognize that ABGs are drawn to determine acid-base balance in the arterial blood. This client does not have a metabolic origin for abnormal ABGs. Metabolic origin is determined by examining the HCO3- levels. The client's bicarbonate is within the expected reference range of 22 to 26 mEq/L. RN Learning System Medical-Surgical: Final Quiz

A nurse is planning care for a client who has been admitted for treatment of a malignant melanoma of the upper leg without metastasis. The nurse should plan to prepare the client for which of the following procedures? Curettage External radiation therapy Regional chemotherapy Surgical excision

✅Surgical excision The therapeutic approach to malignant melanoma depends on the level of invasion and the depth of the lesion. Surgical excision is the treatment of choice for small, superficial lesions. Deeper lesions require wide local excision, followed by skin grafting. Curettage Curettage is used for small lesions that are not melanomas. External radiation therapy Melanoma is resistant to radiation therapy. However, radiation along with corticosteroids might be helpful to clients who have metastatic disease. Regional chemotherapy Regional, or topical, chemotherapy is the treatment of choice for localized tumors and superficial basal cell carcinomas, but not for malignant melanoma, which has an extremely high incidence of metastasis. RN Learning System Medical-Surgical: Dermatological Practice Quiz

A nurse is providing teaching to a client who has gout and urolithiasis. The client asks how to prevent future uric acid stones. Which of the following suggestions should the nurse make? (Select all that apply.) Take allopurinol as prescribed. Exercise several times a week. Limit intake of foods high in purine. Decrease daily fluid intake. Avoid citrus juices.

✅Take allopurinol as prescribed is correct. The nurse should inform the client that allopurinol is an antigout medication that reduces levels of uric acid, which helps prevent uric acid stone formation. ✅Exercise several times a week is correct. The nurse should inform the client that immobility is a risk factor for stone formation; therefore, the client should maintain a healthy lifestyle, including regular exercise, to help prevent stone formation. ✅Limit intake of foods high in purine is correct. The nurse should inform the client that purine increases the risk for uric acid stone formation. The nurse should identify that organ meats, poultry, fish, red wine, and gravies are high in purine. Decrease daily fluid intake is incorrect. The nurse should inform the client that adequate fluid intake of 2 to 3 L per day reduces the risk of stone formation. Avoid citrus juices is incorrect. The nurse should inform the client that citrus juices help to alkalinize the urine, which helps prevent uric acid stone formation. RN Learning System Medical-Surgical: Final Quiz

A nurse is assessing a client who sustained superficial partial-thickness and deep partial-thickness burns 72 hr ago. Which of the following findings should the nurse report to the provider? Edema in the burned extremities Severe pain at the burn sites Urine output of 30 mL/hr Temperature of 39.1° C (102.4° F)

✅Temperature of 39.1° C (102.4° F) An elevated temperature is an indication of infection and the nurse should report this finding to the provider. Sepsis is a critical finding following a major burn injury. Initially, burn wounds are relatively pathogen-free. On approximately the third day following the injury, early colonization of the wound surface by gram-negative organisms changes to predominantly gram-positive opportunistic organisms. Edema in the burned extremities Significant edema is expected when fluid shifts after a burn injury. Severe pain at the burn sites Superficial partial-thickness and deep partial-thickness burns are painful throughout burn therapy. Urine output of 30 mL/hr A urinary output of 30 mL/hr is within the expected reference range. A decrease in urine output is expected with edema and fluid shifts on or about the fourth day following a major burn injury. RN Learning System Medical-Surgical: Dermatological Practice Quiz

Models for Organizing or Clustering Data

🔘HOLISTIC ⏺Human Needs (Maslow) •Physiologic (Survival) Needs: Food, fluids, oxygen, elimination, warmth, physical comfort •Safety and Security Needs: Things necessary for physical safety (e.g., a cane) and psychological security (e.g., a child's favorite toy) •Love and Belonging Needs: Family and significant others •Self-Esteem Needs: Things that make people feel good about themselves and confident in their abilities (e.g., being well groomed, having accomplishments recognized) •Self-Actualization Needs: Need to grow, change, and accomplish goals ⏺Functional Health Patterns (Gordon) •Health Perception/Health Management: Perception of general health status and well-being; adherence to preventive health practices •Nutritional-Metabolic: Patterns of food and fluid intake, fluid and electrolyte balance, general ability to heal •Elimination: Patterns of excretory function (bowel, bladder, skin) and patient's perception •Activity/Exercise: Pattern of exercise, activity, leisure, recreation, and ADLs; factors that interfere with desired to expected individual pattern •Cognitive-Perceptual: Adequacy of sensory modes such as vision, hearing, taste, touch, smell, pain perception, cognitive functional abilities •Sleep/Rest: Patterns of sleep and rest-relaxation periods during a 24-hour day, as well as quality and quantity •Self-Perception/Self-Concept: Attitudes about self, perception of abilities, body image, identity, general sense of worth, and emotional patterns •Role/Relationship: Perception of major roles and responsibilities in current life situation •Sexuality and Reproductive: Perceived satisfaction or dissatisfaction with sexuality, reproductive state, and pattern •Coping/Stress Tolerance: General coping pattern, stress tolerance, support systems, perceived ability to control and manage situations •Value Belief: Values, goals, or beliefs that guide choices or decisions 🔘MEDICAL ⏺Human Response Patterns (Unitary Person) •Exchanging: Nutritional status, temperature, elimination, oxygenation, circulation, fluid balance, skin, and mucous membranes, risk for injury •Communicating: Ability to express thoughts verbally; orientation, speech impairments, language barriers Relating: Establishing bonds, social interaction, support systems, role performance (including parenting, occupation, and sexual role) •Valuing: Religious and cultural preference and practices, relationship with deity, perception of suffering; acceptance of illness •Choosing: Ability to accept help and make decisions, adjustment to health status, desire for independence/dependence, denial of problem, adherence to therapies Moving: Activity tolerance, ability for self-care, sleep patterns, diversional activities, disability history, safety needs, breastfeeding Perceiving: Body image, self-esteem, ability to use all five senses, amount of hopefulness, perception of ability to control current situation Knowing: Knowledge about current illness or therapies; previous illnesses; risk factors, expectations of therapy, cognitive abilities; readiness to learn, orientation, memory Feeling: Pain, grieving, risk for violence, anxiety level, emotional integrity ⏺Body System Model Neurologic Cardiovascular Respiratory Gastrointestinal Musculoskeletal Genitourinary Psychosocial Jarvis CHAPTER 14 Assessing

A nurse is assessing a client who has late-stage heart failure and is experiencing fluid volume overload. Which of the following findings should the nurse expect? Weight gain 1 kg (2.2 lb) in 1 day Pitting edema +1 Client report of nocturnal cough B-Type Natriuretic Peptide (BNP) level of 100 pg/m

✅Weight gain 1 kg (2.2 lb) in 1 day A weight gain of 1 kg (2.2 lb) in 1 day alerts the nurse that the client is retaining fluid and is at risk of fluid volume overload. This is an indication that the client's heart failure is worsening. Pitting edema +1 Pitting edema, a visible finger indentation after application of pressure, alerts the nurse that the client has retained fluid and demonstrates that there is fluid in the client's tissues. Pitting edema is rated on a scale of mild (+1) to severe (+3). Pitting edema of +3 is an indication that the client has developed fluid volume overload and the heart failure is worsening. Client report of nocturnal cough The client who is in the early stages of heart failure might report a cough that is irritating, occurs at night, and is nonproductive. B-Type Natriuretic Peptide (BNP) level of 100 pg/mL BNP levels increase as the result of the ventricular hypertrophy that occurs in heart failure. A BNP level above 100 pg/mL is indicative of heart failure. Levels continue to increase with the severity of the heart failure. RN Learning System Medical-Surgical: Cardiovascular and Hematology Practice Quiz

6. The nurse records a patient's blood pressure as 148/100. What is the priority action of the nurse when determining the significance of this reading? Compare this reading to standards. Check the taxonomy of nursing diagnoses for a pertinent label. Check a medical text for the signs and symptoms of high blood pressure. Consult with colleagues

✅a. A standard, or a norm, is a generally accepted rule, measure, pattern, or model to which data can be compared in the same class or category. For example, when determining the significance of a patient's blood pressure reading, appropriate standards include normative values for the patient's age group, race, and illness category. Deviation from an appropriate norm may be the basis for writing a diagnosis. CHAPTER 15 Diagnosing

7. When the initial nursing assessment revealed that a patient had not had a bowel movement for 2 days, the student nurse wrote the diagnostic label "constipation." What would be the instructor's BEST response to this student's diagnosis? "Was this diagnosis derived from a cluster of significant data or a single clue?" "This early diagnosis will help us manage the problem before it becomes more acute." "Have you determined if this is an actual or a possible diagnosis?" "This condition is a medical problem that should not have a nursing diagnosis."

✅a. Nursing diagnoses should always be derived from clusters of significant data rather than from a single cue. A data cluster is a grouping of patient data or cues that point to the existence of a patient health problem. There may be a reason for the lack of a bowel movement for 2 days, or it might be this person's normal pattern. CHAPTER 15 Diagnosing

2. A nurse is caring for a patient who presents with labored respirations, productive cough, and fever. What would be appropriate nursing diagnoses for this patient? Select all that apply. Bronchial pneumonia Impaired gas exchange Ineffective airway clearance Potential complication: sepsis Infection related to pneumonia Risk for septic shock

✅b, c, f. Nursing diagnoses are actual or potential health problems that can be prevented or resolved by independent nursing interventions, such as impaired gas exchange, ineffective airway clearance, or risk for septic shock. Bronchial pneumonia and infection are medical diagnoses, and "potential complication: sepsis" is a collaborative problem CHAPTER 15 Diagnosing

1. A registered nurse is writing a diagnosis for a patient who is in traction because of multiple fractures from a motor vehicle accident. Which nursing actions are related to this step in the nursing process? Select all that apply. The nurse uses the nursing interview to collect patient data. The nurse analyzes data collected in the nursing assessment. The nurse develops a care plan for the patient. The nurse points out the patient's strengths. The nurse assesses the patient's mental status. The nurse identifies community resources to help his family cope.

✅b, d, f. The purposes of diagnosing are to identify how an individual, group, or community responds to actual or potential health and life processes; identify factors that contribute to or cause health problems (etiologies); and identify resources or strengths the individual, group, or community can draw on to prevent or resolve problems. In the diagnosing step of the nursing process, the nurse interprets and analyzes data gathered from the nursing assessment, identifies patient strengths, and identifies resources the patient can use to resolve problems. The nurse assesses and collects patient data in the assessment step and develops a care plan in the planning step of the nursing process. CHAPTER 15 Diagnosing

4. A nurse assesses a patient and formulates the following nursing diagnosis: Risk for Impaired Skin Integrity related to prescribed bed rest as evidenced by reddened areas of skin on the heels and back. Which phrase represents the etiology of this diagnostic statement? Risk for Impaired Skin Integrity Related to prescribed bed rest As evidenced by As evidenced by reddened areas of skin on the heels and back

✅b. "Related to prescribed bed rest" is the etiology of the statement. The etiology identifies the contributing or causative factors of the problem. "Risk for Impaired Skin Integrity" is the problem, and "as evidenced by reddened areas of skin on the heels and back" are the defining characteristics of the problem CHAPTER 15 Diagnosing

3. After assessing a patient who is recovering from a stroke in a rehabilitation facility, a nurse interprets and analyzes the patient data. Which of the four basic conclusions has the nurse reached when identifying the need to collect more data to confirm a diagnosis of situational low self-esteem? No problem Possible problem Actual nursing diagnosis Clinical problem other than nursing diagnosis

✅b. When a possible problem exists, such as situational low self-esteem related to effects of stroke, the nurse must collect more data to confirm or disprove the suspected problem. The conclusion "no problem" means no nursing response is indicated. When an actual problem exists, the nurse begins planning, implementing, and evaluating care to prevent, reduce, or resolve the problem. A clinical problem other than nursing diagnosis requires that the nurse consult with the appropriate health care professional to work collaboratively on the problem. CHAPTER 15 Diagnosing

HELP mnemonic to ensure systematic person-centered observation

❑H = Help: Observe the first signs patient may need help. Look for signs of distress (pallor, pain, labored breathing). ❑E = Environmental equipment: Look for safety hazards; ensure that all equipment is working (IVs, oxygen, catheter). ❑L = Look: Examine patient thoroughly. ❑P = People: Who are the people in the room? What are they doing? Jarvis CHAPTER 14 Assessing


Set pelajaran terkait

RNSG 2201 Care of Children and Families Ch 14 Evolve

View Set

Chapter 6 - The Legal And Regulatory Environment of Business

View Set

Ch 1 Strategic Management and Strategic Competitiveness

View Set

Lab Manual Exercise 11 Post-lab Quiz Question 5

View Set

AP Chem Unit 4.7: Types of Chemical Reactions

View Set

CH 10 CLOUD AND VIRTUALIZATION SECURITY

View Set

Principles of RE I: Texas Real Estate License Act

View Set

AH 1 Exam 1: Care of Perioperative Pts

View Set